Download as pdf or txt
Download as pdf or txt
You are on page 1of 220

Engineering Mathematics I Study Guide

Eng. Jose Eduardo dos Santos Campus


Faculty of Engineering & IT
University of Namibia

Compiled by: Mr. Naftali N Indongo


Revised by: Dr. Enock koga

April 2020
ii

PLAGIARISM DECLARATION

1. Plagiarism is the use of ideas, material and other intellectual property of another’s work and
to present it as my own.

2. I agree that plagiarism is a punishable offence because it constitutes theft.

3. Accordingly, all quotations and contributions from any source whatsoever (including the in-
ternet) have been cited fully. I understand that the reproduction of text without quotation
marks (even when the source is cited) is plagiarism.

4. I also understand that direct translations are plagiarism.

5. I declare that the work contained in this assignment, except otherwise stated, is my original
work and that I have not previously (in its entirety or in part) submitted it for grading in
this assignment or another assignment.

Copyright © 2020 University of Namibia


All rights reserved
iii

One author
iv

ACKNOWLEDGEMENTS

The Department of Statistics and Actuarial Science (the Department) wishes to acknowledge David
Rodwell for generously creating a template based off the USB (University of Stellenbosch Business
School) guidelines which have been adapted for the purposes of the department.
v

ABSTRACT

Insert an abstract of not more than 500 words here

Key words:
Technical guidelines; Chapter outline; Examples
vi

OPSOMMING

Sluit ’n Afrikaanse opsomming in

Sleutelwoorde:
Tegniese vereistes, Voorgestelde Hoofstukke, Voorbeelde.
vii

TABLE OF CONTENTS

PLAGIARISM DECLARATION ii

ACKNOWLEDGEMENTS iv

ABSTRACT v

OPSOMMING vi

LIST OF FIGURES xiii

LIST OF TABLES xiv

LIST OF APPENDICES xv

LIST OF ABBREVIATIONS AND/OR ACRONYMS xvi

1 VECTORS IN EUCLIDEAN SPACE 1


1.1 Introduction . . . . . . . . . . . . . . . . . . . . . . . . . . . . . . . . . . . . . . . . . 1
1.2 Representation of Vectors in terms of Unit vectors î,ĵ and k̂ . . . . . . . . . . . . . . 2
1.3 Vector Algebra . . . . . . . . . . . . . . . . . . . . . . . . . . . . . . . . . . . . . . . 3
1.4 Vector Dot or Scalar Product . . . . . . . . . . . . . . . . . . . . . . . . . . . . . . . 4
1.5 The Cross or Vector Product . . . . . . . . . . . . . . . . . . . . . . . . . . . . . . . 5

2 LINES AND PLANES 10


2.1 Lines . . . . . . . . . . . . . . . . . . . . . . . . . . . . . . . . . . . . . . . . . . . . . 10
2.1.1 Line through a point, parallel to a vector . . . . . . . . . . . . . . . . . . . . 10
2.1.2 Line through two points . . . . . . . . . . . . . . . . . . . . . . . . . . . . . . 12
2.1.3 Distance between a point and a line . . . . . . . . . . . . . . . . . . . . . . . 14
2.2 Planes . . . . . . . . . . . . . . . . . . . . . . . . . . . . . . . . . . . . . . . . . . . . 17
2.2.1 Plane through a point, perpendicular to a vector . . . . . . . . . . . . . . . . 17
2.2.2 Plane containing three non-collinear points . . . . . . . . . . . . . . . . . . . 18
2.2.3 Distance between a point and a plane . . . . . . . . . . . . . . . . . . . . . . 19
2.2.4 Distance between two parallel planes . . . . . . . . . . . . . . . . . . . . . . . 20
viii

2.2.5 Line of intersection of two planes . . . . . . . . . . . . . . . . . . . . . . . . . 21

3 MATRICES AND SYSTEMS OF LINEAR EQUATIONS 24


3.1 Introduction . . . . . . . . . . . . . . . . . . . . . . . . . . . . . . . . . . . . . . . . . 24
3.2 Matrix Algebra . . . . . . . . . . . . . . . . . . . . . . . . . . . . . . . . . . . . . . . 25
3.2.1 Equality of matrices . . . . . . . . . . . . . . . . . . . . . . . . . . . . . . . . 25
3.2.2 Matrix Addition or Subtraction . . . . . . . . . . . . . . . . . . . . . . . . . . 25
3.2.3 Matrix Transpose . . . . . . . . . . . . . . . . . . . . . . . . . . . . . . . . . . 26
3.2.4 Matrix Scalar Multiplication . . . . . . . . . . . . . . . . . . . . . . . . . . . 26
3.2.5 Matrix Multiplication . . . . . . . . . . . . . . . . . . . . . . . . . . . . . . . 26
3.3 Special matrices . . . . . . . . . . . . . . . . . . . . . . . . . . . . . . . . . . . . . . 29
3.4 Determinants . . . . . . . . . . . . . . . . . . . . . . . . . . . . . . . . . . . . . . . . 31
3.4.1 Determinant of a 2 × 2 matrix . . . . . . . . . . . . . . . . . . . . . . . . . . 32
3.5 Solution to systems of linear equations by matrices . . . . . . . . . . . . . . . . . . . 35
3.5.1 Cramer’s rule . . . . . . . . . . . . . . . . . . . . . . . . . . . . . . . . . . . . 35
3.5.2 Gaussion Elimination . . . . . . . . . . . . . . . . . . . . . . . . . . . . . . . 36
3.5.3 Gauss-Jordan Elimination . . . . . . . . . . . . . . . . . . . . . . . . . . . . . 41
3.5.4 Inconsistent and Dependent System . . . . . . . . . . . . . . . . . . . . . . . 42
3.6 The inverse matrices . . . . . . . . . . . . . . . . . . . . . . . . . . . . . . . . . . . . 43
3.6.1 Finding the inverse of a matrix by elementary row operations . . . . . . . . 44
3.6.2 Finding the inverse of a matrix via the adjoint matrix . . . . . . . . . . . . . 45
3.6.3 Solving to systems of linear equations by matrix inversion . . . . . . . . . . . 46

4 SEQUENCES AND SERIES 49


4.1 Sequences . . . . . . . . . . . . . . . . . . . . . . . . . . . . . . . . . . . . . . . . . . 49
4.1.1 Recursively defined sequences . . . . . . . . . . . . . . . . . . . . . . . . . . . 51
4.1.2 Limits of Sequences . . . . . . . . . . . . . . . . . . . . . . . . . . . . . . . . 53
4.2 Partial sums of a sequence . . . . . . . . . . . . . . . . . . . . . . . . . . . . . . . . . 55
4.2.1 Convergence/Divergence of Series . . . . . . . . . . . . . . . . . . . . . . . . . 58
4.3 special series . . . . . . . . . . . . . . . . . . . . . . . . . . . . . . . . . . . . . . . . 60
4.3.1 Geometric Series . . . . . . . . . . . . . . . . . . . . . . . . . . . . . . . . . . 60
ix

4.3.2 Telescoping Series . . . . . . . . . . . . . . . . . . . . . . . . . . . . . . . . . 62


4.3.3 Harmonic Series . . . . . . . . . . . . . . . . . . . . . . . . . . . . . . . . . . 63
4.4 Series with non-negative terms . . . . . . . . . . . . . . . . . . . . . . . . . . . . . . 64

5 FUNCTIONS 68
5.1 Introduction . . . . . . . . . . . . . . . . . . . . . . . . . . . . . . . . . . . . . . . . . 68
5.2 Types of functions . . . . . . . . . . . . . . . . . . . . . . . . . . . . . . . . . . . . . 70
5.3 Trigonometric Functions . . . . . . . . . . . . . . . . . . . . . . . . . . . . . . . . . . 75
5.3.1 Special angles and special values of their trigonometric functions . . . . . . . 77
5.3.2 Signs of trigonometric functions . . . . . . . . . . . . . . . . . . . . . . . . . . 78
5.3.3 Trigonometric Identities and Formulas . . . . . . . . . . . . . . . . . . . . . . 79
5.4 Exponential and logarithmic functions . . . . . . . . . . . . . . . . . . . . . . . . . . 83
5.5 Hyperbolic functions . . . . . . . . . . . . . . . . . . . . . . . . . . . . . . . . . . . . 88
5.6 INVERSES OF TRIGONOMETRIC AND HYPERBOLIC FUNCTIONS . . . . . . 92
5.6.1 INVERSE TRIGONOMETRIC FUNCTIONS . . . . . . . . . . . . . . . . . 92
5.6.2 INVERSE HYPERBOLIC FUNCTIONS . . . . . . . . . . . . . . . . . . . . 92
5.7 RATIONAL FUNCTIONS . . . . . . . . . . . . . . . . . . . . . . . . . . . . . . . . 94
5.7.1 rules for partial fraction decomposition . . . . . . . . . . . . . . . . . . . . . . 94
5.8 Limits and continuity of Functions . . . . . . . . . . . . . . . . . . . . . . . . . . . . 105
5.8.1 Infinite Limits . . . . . . . . . . . . . . . . . . . . . . . . . . . . . . . . . . . 116
5.8.2 Limits to Infinity . . . . . . . . . . . . . . . . . . . . . . . . . . . . . . . . . . 119
5.8.3 Limits of Rational Functions as x → ±∞ . . . . . . . . . . . . . . . . . . . . 121
5.8.4 Continuous Functions . . . . . . . . . . . . . . . . . . . . . . . . . . . . . . . 124

6 DIFFERENTIATION 128
6.1 DERIVATIVES AND RATES OF CHANGE . . . . . . . . . . . . . . . . . . . . . . 128
6.1.1 TANGENTS . . . . . . . . . . . . . . . . . . . . . . . . . . . . . . . . . . . . 128
6.1.2 VELOCITIES . . . . . . . . . . . . . . . . . . . . . . . . . . . . . . . . . . . 131
6.1.3 DERIVATIVES . . . . . . . . . . . . . . . . . . . . . . . . . . . . . . . . . . . 133
6.1.4 RATES OF CHANGE . . . . . . . . . . . . . . . . . . . . . . . . . . . . . . . 136
6.1.5 HIGHER DERIVATIVES . . . . . . . . . . . . . . . . . . . . . . . . . . . . . 143
x

6.2 DIFFERENTIATION RULES . . . . . . . . . . . . . . . . . . . . . . . . . . . . . . . 146


6.2.1 DERIVATIVES OF POLYNOMIALS AND EXPONENTIAL FUNCTIONS . 146
6.2.2 DERIVATIVES OF TRIGONOMETRIC FUNCTIONS . . . . . . . . . . . . 154
6.2.3 THE CHAIN RULE . . . . . . . . . . . . . . . . . . . . . . . . . . . . . . . . 157
6.2.4 DERIVATIVES OF HYPERBOLIC FUNCTIONS . . . . . . . . . . . . . . . 160
6.3 IMPLICIT DIFFERENTIATION . . . . . . . . . . . . . . . . . . . . . . . . . . . . . 162
6.3.1 DERIVATIVES OF INVERSE TRIGONOMETRIC FUNCTIONS . . . . . . 163
6.3.2 DERIVATIVES OF LOGARITHMIC FUNCTIONS . . . . . . . . . . . . . . 165
6.4 APPLICATIONS OF DIFFERENTIATION . . . . . . . . . . . . . . . . . . . . . . . 169
6.4.1 EXTREME VALUES AND FIRST AND SECOND DERIVATIVE TESTS . 169
6.4.2 INDETERMINATE FORMS AND L’HOSPITAL’S RULE . . . . . . . . . . 179
6.4.3 PRACTICAL APPLICATIONS OF DIFFERENTIATION . . . . . . . . . . 185

REFERENCES 203

APPENDIX A QUESTIONNAIRE 204

APPENDIX B USB STYLE SHEETS 205


xi

LIST OF FIGURES

1.1 Units base vectors . . . . . . . . . . . . . . . . . . . . . . . . . . . . . . . . . . . . . 2


1.2 Vector v in terms of unit vectors . . . . . . . . . . . . . . . . . . . . . . . . . . . . . 2
1.3 A geometric interpretation of the cross product . . . . . . . . . . . . . . . . . . . . . 7
1.4 Geometrical area of the parallelogram . . . . . . . . . . . . . . . . . . . . . . . . . . 7
1.5 Geometrical area of the parallelogram . . . . . . . . . . . . . . . . . . . . . . . . . . 8

2.1 Line through a point parallel to a vector. . . . . . . . . . . . . . . . . . . . . . . . . 10


2.2 Line through two points. . . . . . . . . . . . . . . . . . . . . . . . . . . . . . . . . . . 12
2.3 Distance between a point and a line. . . . . . . . . . . . . . . . . . . . . . . . . . . . 14
2.4 Plane π through a point, ⊥ to a vector. . . . . . . . . . . . . . . . . . . . . . . . . . 17
2.5 Plane π, containing noncollinear points P , Q and R. . . . . . . . . . . . . . . . . . . 18
2.6 Distance between a plane π and a point P . . . . . . . . . . . . . . . . . . . . . . . . 20
2.7 Distance D between planes π1 and π2 . . . . . . . . . . . . . . . . . . . . . . . . . . . 20
2.8 Line of intersection between planes π1 and π2 . . . . . . . . . . . . . . . . . . . . . . . 21

n+1
4.1 limn→∞ n2
. . . . . . . . . . . . . . . . . . . . . . . . . . . . . . . . . . . . . . . . . 53

5.1 Definition of a function . . . . . . . . . . . . . . . . . . . . . . . . . . . . . . . . . . 68


5.2 Graph of f (x) . . . . . . . . . . . . . . . . . . . . . . . . . . . . . . . . . . . . . . . . 72
5.3 Graph of g(x) . . . . . . . . . . . . . . . . . . . . . . . . . . . . . . . . . . . . . . . . 73
5.4 Graph of a circle (x − h)2 + (y − k)2 = r2 . . . . . . . . . . . . . . . . . . . . . . . . 73
5.5 Radian measure of angle θ . . . . . . . . . . . . . . . . . . . . . . . . . . . . . . . . . 75
5.6 Graph of y = sin θ . . . . . . . . . . . . . . . . . . . . . . . . . . . . . . . . . . . . . 76
5.7 Graph of y = cos θ . . . . . . . . . . . . . . . . . . . . . . . . . . . . . . . . . . . . . 76
5.8 Graph of y = tan θ . . . . . . . . . . . . . . . . . . . . . . . . . . . . . . . . . . . . . 77
5.9 Special Triangles . . . . . . . . . . . . . . . . . . . . . . . . . . . . . . . . . . . . . . 78
5.10 Cast Diagram . . . . . . . . . . . . . . . . . . . . . . . . . . . . . . . . . . . . . . . . 78
 x
1
5.11 Graph of f (x) = 2 . . . . . . . . . . . . . . . . . . . . . . . . . . . . . . . . . . . 83
 x
1
5.12 Graph of f (x) = 2 . . . . . . . . . . . . . . . . . . . . . . . . . . . . . . . . . . . 84
5.13 Graph of f (x) = ex and g(x) = e−x . . . . . . . . . . . . . . . . . . . . . . . . . . . . 84
xii

5.14 Graph of f (x) = log 1 x . . . . . . . . . . . . . . . . . . . . . . . . . . . . . . . . . . . 85


2

5.15 Graph of f (x) = log2 x . . . . . . . . . . . . . . . . . . . . . . . . . . . . . . . . . . . 86


5.16 Graph of f (x) = ex and g(x) = loge x = ln x . . . . . . . . . . . . . . . . . . . . . . . 86
 x
1
5.17 Graph of f (x) = 2 and g(x) = log 1 x . . . . . . . . . . . . . . . . . . . . . . . . . 87
2

5.18 sinh x . . . . . . . . . . . . . . . . . . . . . . . . . . . . . . . . . . . . . . . . . . . . 88
5.19 cosh x . . . . . . . . . . . . . . . . . . . . . . . . . . . . . . . . . . . . . . . . . . . . 88
5.20 tanh . . . . . . . . . . . . . . . . . . . . . . . . . . . . . . . . . . . . . . . . . . . . . 88
5.21 A catenary y = c + a cosh x . . . . . . . . . . . . . . . . . . . . . . . . . . . . . . . . 89
5.22 Idealized ocean wave . . . . . . . . . . . . . . . . . . . . . . . . . . . . . . . . . . . . 89
5.23 sinh−1 x domain= R, range= R . . . . . . . . . . . . . . . . . . . . . . . . . . . . . . 93
5.24 cosh−1 x domain= [1, ∞], range= [0, ∞] . . . . . . . . . . . . . . . . . . . . . . . . . 93
5.25 tanh−1 domain= (−1, 1), range= R . . . . . . . . . . . . . . . . . . . . . . . . . . . . 93
5.26 limx→a f (x) = L in all three cases . . . . . . . . . . . . . . . . . . . . . . . . . . . . . 106
5.27 . . . . . . . . . . . . . . . . . . . . . . . . . . . . . . . . . . . . . . . . . . . . . . . . 107
5.28 . . . . . . . . . . . . . . . . . . . . . . . . . . . . . . . . . . . . . . . . . . . . . . . . 107
5.29 limx→a− f (x) = L . . . . . . . . . . . . . . . . . . . . . . . . . . . . . . . . . . . . . . 108
5.30 limx→a+ f (x) = L . . . . . . . . . . . . . . . . . . . . . . . . . . . . . . . . . . . . . . 108
5.31 Graph of f (x) . . . . . . . . . . . . . . . . . . . . . . . . . . . . . . . . . . . . . . . . 109
1
5.32 limx→0 x2
. . . . . . . . . . . . . . . . . . . . . . . . . . . . . . . . . . . . . . . . . . 117
5.33 One-sided infinite limits . . . . . . . . . . . . . . . . . . . . . . . . . . . . . . . . . . 118
1
5.34 limx→±∞ x . . . . . . . . . . . . . . . . . . . . . . . . . . . . . . . . . . . . . . . . . 120

6.1 Slope of the secant line P Q. . . . . . . . . . . . . . . . . . . . . . . . . . . . . . . . . 129


6.2 Tangent to the curve C : y = f (x). . . . . . . . . . . . . . . . . . . . . . . . . . . . . 129
6.3 Tangent to the curve C : y = f (x) where h = x − a . . . . . . . . . . . . . . . . . . . 130
3
6.4 Tangent to the curve y = x at the point P (3, 1). . . . . . . . . . . . . . . . . . . . . 131
6.5 Position change from f (a) to f (a + h). . . . . . . . . . . . . . . . . . . . . . . . . . . 132
f (a+h)−f (a)
6.6 mP Q = h = average velocity . . . . . . . . . . . . . . . . . . . . . . . . . . 132
6.7 Tangent to the curve y = x2 − 8x + 9 at the point P (3, −6). . . . . . . . . . . . . . . 136
6.8 average rate of change mP Q instantaneous rate of change = slope of tangent at P . . 137
6.9 Graph of f (x) = |x|. . . . . . . . . . . . . . . . . . . . . . . . . . . . . . . . . . . . . 141
xiii

6.10 Graph of f 0 (x). . . . . . . . . . . . . . . . . . . . . . . . . . . . . . . . . . . . . . . . 141


6.11 Three ways for y = f (x) not to be differentiable at x = a. . . . . . . . . . . . . . . . 142
6.12 . . . . . . . . . . . . . . . . . . . . . . . . . . . . . . . . . . . . . . . . . . . . . . . . 150
6.13 . . . . . . . . . . . . . . . . . . . . . . . . . . . . . . . . . . . . . . . . . . . . . . . . 150
6.14 Minimum value 0, no maximum. . . . . . . . . . . . . . . . . . . . . . . . . . . . . . 170
6.15 No minimum, no maximum. . . . . . . . . . . . . . . . . . . . . . . . . . . . . . . . . 170
6.16 Illustration of the Extreme Value Theorem . . . . . . . . . . . . . . . . . . . . . . . 170
6.17 Illustration of the Increasing/Decreasing Test . . . . . . . . . . . . . . . . . . . . . . 174
6.18 Illustration of the First Derivative Test . . . . . . . . . . . . . . . . . . . . . . . . . . 175
6.19 concave upward . . . . . . . . . . . . . . . . . . . . . . . . . . . . . . . . . . . . . . . 176
6.20 concave downward . . . . . . . . . . . . . . . . . . . . . . . . . . . . . . . . . . . . . 176
6.21 Sketch of the curve y = x4 − 4x3 . . . . . . . . . . . . . . . . . . . . . . . . . . . . . 178
6.22 A cylindrical tank with fluid. . . . . . . . . . . . . . . . . . . . . . . . . . . . . . . . 185
6.23 A spherical balloon, with radius r. . . . . . . . . . . . . . . . . . . . . . . . . . . . . 186
6.24 . . . . . . . . . . . . . . . . . . . . . . . . . . . . . . . . . . . . . . . . . . . . . . . . 188
6.25 A sliding ladder problem . . . . . . . . . . . . . . . . . . . . . . . . . . . . . . . . . . 189
6.26 An inverted circular cone shaped water tank. . . . . . . . . . . . . . . . . . . . . . . 190
6.27 Similar triangles . . . . . . . . . . . . . . . . . . . . . . . . . . . . . . . . . . . . . . 191
6.28 Two cars approaching an intersection. . . . . . . . . . . . . . . . . . . . . . . . . . . 192
6.29 Area of a rectangular field with dimensions x and y. . . . . . . . . . . . . . . . . . . 195
6.30 A cylindrical can. . . . . . . . . . . . . . . . . . . . . . . . . . . . . . . . . . . . . . . 196
6.31 Total Surface Area of a cylindrical can. . . . . . . . . . . . . . . . . . . . . . . . . . 196
6.32 . . . . . . . . . . . . . . . . . . . . . . . . . . . . . . . . . . . . . . . . . . . . . . . . 198
6.33 A 1125 m3 rectangular tank . . . . . . . . . . . . . . . . . . . . . . . . . . . . . . . . 200
xiv

LIST OF TABLES

4.1 Finding the terms of the sequences . . . . . . . . . . . . . . . . . . . . . . . . . . . . 51

5.1 Special angles and values of their trigonometric function. . . . . . . . . . . . . . . . . 77


5.2 Quadrants and the signs of the trigonometric functions. . . . . . . . . . . . . . . . . 78
5.3 Guessing limx→2 f (x). . . . . . . . . . . . . . . . . . . . . . . . . . . . . . . . . . . . 105
5.4 Guessing limx→1 f (x) . . . . . . . . . . . . . . . . . . . . . . . . . . . . . . . . . . . . 107
5.5 Guessing limt→0 f (t) . . . . . . . . . . . . . . . . . . . . . . . . . . . . . . . . . . . . 108
5.6 Guessing limx→0 f (x) . . . . . . . . . . . . . . . . . . . . . . . . . . . . . . . . . . . . 113
5.7 Guessing limx→0 f (x) . . . . . . . . . . . . . . . . . . . . . . . . . . . . . . . . . . . . 117

ah −1
6.1 f 0 (0) = limh→0 h . . . . . . . . . . . . . . . . . . . . . . . . . . . . . . . . . . . . . 149
6.2 Behavior of f (x). . . . . . . . . . . . . . . . . . . . . . . . . . . . . . . . . . . . . . . 174
6.3 Fist Derivative Test. . . . . . . . . . . . . . . . . . . . . . . . . . . . . . . . . . . . . 177
6.4 Concavity Test. . . . . . . . . . . . . . . . . . . . . . . . . . . . . . . . . . . . . . . . 177
xv

LIST OF APPENDICES

APPENDIX A QUESTIONNAIRE
APPENDIX B USB STYLE SHEET
xvi

LIST OF ABBREVIATIONS AND/OR ACRONYMS

USB University of Stellenbosch Business School


Etc Insert Acronym Here
1

CHAPTER 1

VECTORS IN EUCLIDEAN SPACE

1.1 INTRODUCTION

Definition 1.1.1. A vector quantity v in a Euclidean space R3 is an ordered triple v = (v1 , v2 , v3 )


in which v1 , v3 and v3 are real numbers.

Note. The numbers v1 , v3 and v3 are called the first, second and third components of vector v,
respectively. Equivalently, v1 , v3 and v3 are the x, y and z components of vector v.

Definition 1.1.2. Null vector


The null (zero) vector, denoted by 0, has neither magnitude no direction and it is an ordered triple
0 = (0, 0, 0).

Definition 1.1.3. Equality of Vectors


Two nonzero vectors u = (u1 , u2 , u3 ) and v = (v1 , v2 , v3 ) are said to be equal if and only if they
have the same magnitude and direction. i.e. u1 = v1 , u2 = v2 and u3 = v3 .

Example 1.1. If u = (a1 , −5, 6) and v = (3, b2 , b3 ), what the values of a1 , b2 and b3 such that
u = v?

Solution 1.1. a1 = b1 = 3, b2 = a2 = −5 and b3 = a3 = 6.

Theorem 1.1.4. Norm or Magnitude of a vector


For a vector v = (v1 , v2 , v3 ) in R3 , the norm or magnitude of v, denoted by ||v|| is given by:

q
||v|| = v12 + v22 + v32 (1.1)

Geometrically, ||v|| is the length of vector v. Thus for a zero vector ||0|| = 0.
2

Example 1.2. Calculate the magnitude of the vector v = (2, −1, 2).
p √
Solution 1.2. ||v|| = 22 + (−1)2 + 22 = 9=3

1.2 ˆ Jˆ
REPRESENTATION OF VECTORS IN TERMS OF UNIT VECTORS I,
AND K̂

A vector v = aî + bĵ + ck̂ defines a point in the Euclidean space R3 , where î, ĵ and k̂ are called
unit base vectors whose magnitudes are all equal to 1 (see Figure 1.1). i.e ||î|| = ||ĵ|| = ||k̂|| = 1,
as shown in Figure 1.2.

NB: unit base vectors are of the same


magnitude (1)

Figure 1.1: Units base vectors Figure 1.2: Vector v in terms of unit vectors

From Figure ??, we have that the position vector relative to the origin is v = aî + bĵ + ck̂ and its

norm or magnitude is ||v|| = a2 + b2 + c2 .

Definition 1.2.1. Unit Vector A unit vector is vector whose magnitude is equals to 1.

Theorem 1.2.2. For any arbitrary vector v = v1 î + v2 ĵ + v3 k̂, a unit vector in the direction of v,
denoted by v̂ is defined by:
v v1 v2 v3
v̂ = = î + ĵ + k̂ (1.2)
||v|| ||v|| ||v|| ||v||
3

Note. v = ||v||v̂ and ||v̂|| = 1

Example 1.3. Find a unit vector in the direction of v = 3î + 2ĵ + 5k̂.

Solution 1.3.

p
||v|| = 32 + 22 + 52

= 40

So,

1
v̂ = √ (3î + 2ĵ + 5k̂)
40
3 2 5
= √ î + √ ĵ + √ k̂
40 40 40

1.3 VECTOR ALGEBRA

Now that we know what vectors are, we can then perform some of the common algebraic operations
such as: addition, subtraction, scalar multiplication e.t.c on them. We begin by introducing the
notion of a scalar.

Definition 1.3.1. A scalar is a quantity that can be represented by a single number.

Definition 1.3.2. Scalar multiplication of a vector


For an arbitrary scalar λ and a nonzero vector v the scalar multiplication of v = (v1 , v2 , v3 ) by λ
is defined as a vector
λv = (λv1 , λv2 , λv3 ) (1.3)

Whose magnitude is given by


||λv|| = |λ||v|| (1.4)

Consider the following case for λv


4

Case 1: If λ > 0, then λv points in the same direction as v.


Case 2: If λ < 0, then λv points in the opposite direction as v.
Case 3: If λ = 0, then λv = 0.

Definition 1.3.3. Sum of two vectors


If u = (u1 , u2 , u3 ) and v = (v1 , v2 , v3 ) be two nonzeros vectors. The sum of u and v is a vector
defined by:
u + v = (u1 + v1 , u2 + v2 , u3 + v3 ) (1.5)

Example 1.4. Let u = (1, 2, −5) and v = (−2, 2, 4) then u + v = (1 + (−2), 2 + 2, (−5) + 4) =
(−1, 4, −1)

1.4 VECTOR DOT OR SCALAR PRODUCT

In the previous section on vector algebra, we defined the multiplication of a vector by a scalar.
However, we did not define multiplication of a vector by another vector. In this section we consider
one type of multiplication of vectors called the dot product.

Definition 1.4.1. Let u = (u1 , u2 , u3 ) and v = (v1 , v2 , v3 ) be vectors in R3 . The dot product of u
and v , denoted by u · v is given by

u · v = u1 v1 + u2 v2 + u3 v3 (1.6)

Theorem 1.4.2. Properties of the dot product


Let u, v and w be any vectors and λ ∈ R be a scalar, then

(a) u · v = v · u Commutative Law

(b) (λu) · v = v · (λu) = λ(u · v) Associative Law

(c) v · 0 = 0 = 0 · v

(d) u·(v+w) Distributive Law


5

Theorem 1.4.3. Angle between two vectors


Let u and v be two nonzero vectors, then the angle between u and v is given by

 u·v 
θ = cos−1 (1.7)
||u||||v||

NB: u · v = ||u||||v|| cos θ

Example 1.5. Find the angle between the vectors u = (2, 1, −1) and v = (3, −4, 1)
√ √
Solution 1.4. Since u · v = 6 − 4 − 1 = 1, ||u|| = 6 and ||v|| = 26, then
cos θ = u·v
||u||||v|| = √ 1

6 26
≈ 0.08 =⇒ θ = cos−1 (0.08) = 85.41

Corollary 1.4.4. Two nonzero vectors u and v are perpendicular if and only if u · v = 0 i.e.
θ = π2 . We write u ⊥ v to indicate that u and v are perpendicular.

Corollary 1.4.5. Two nonzero vectors u and v are parallel if and only if u · v = ||u||||v|| i.e.
θ = 0. We write u k v to indicate that u and v are parallel.

Theorem 1.4.6. Dot product of unit vectors


If û and v̂ are unit vectors, then û · v̂ = ||u||||v|| cos θ = cos θ, 0 ≤ θ ≤ π.

1.5 THE CROSS OR VECTOR PRODUCT

In Section 1.4, we defined the dot product, which gave a way of multiplying two vectors, however,
the resulting product was a scalar. In this section we will define a product of two mothers called the
Cross product that will result into another vector. The cross product is only defined for vectors
in R3 .

Definition 1.5.1. Let u = (u1 , u2 , u3 ) and v = (v1 , v2 , v3 ) be vectors in R2 . The cross product of
6

u and v, denoted by u × v is a vector in R3 given by

i j k
u × v = u1 u2 u3
v1 v2 v3

u2 u3 u1 u3 u1 u2
= î − ĵ + k̂
v2 v3 v1 v3 v1 v2

= (u2 v3 − u3 v2 )î − (u1 v3 − u3 v1 )ĵ + (u1 v2 − u2 v1 )k̂

Example 1.6. Find î × ĵ

Solution 1.5.

i j k
u×v= 1 0 0
0 1 0

0 0 1 0 1 0
= î − ĵ + k̂
1 0 0 0 0 1

= 0î − 0ĵ + k̂ = k̂

Theorem 1.5.2. If the cross product u × v of two nonzero vectors u and v is also a nonzero
vector, then it is perpendicular to both u and v.

Definition 1.5.3. The Cross Product ( A geometric interpretation)


Let u and v be nonzero vectors in R3 , such that u and v are not parallel. The span of u and v
forms a plane P so that u × v is perpendicular to both u and v as shown on Figure ??. There are
two possible directions for u × v, one in the opposite of the other. The direction of u × v is given
by the right-hand rule. i.e. The vectors u, v and u × v forms the right-hand system.
7

Figure 1.3: A geometric interpretation of the cross product

Let n̂ be the unit normal to the plane P, then the cross product of vectors u and v, written as u × v
is defined as the vector
u × v = ||u||||v|| sin θn̂ (1.8)

Where θ is the angle between u and v. Thus, the cross product can be interpreted as a vector area

u × v = An̂ (1.9)

where A = ||u||||v|| sin θ is the geometrical area of the parallelogram shown below:

Figure 1.4: Geometrical area of the parallelogram


8

and the unit vector n̂ is normal to the area A.

Theorem 1.5.4. Areas of Tringles and Parallelogram

(a) The area of the triangle with adjacent sides u and v (As vectors in R3 ) is:

1
A = ||u × v|| (1.10)
2

(b) The area of the parallelogram with adjacent sides u and v (As vectors in R3 ) is:

A = ||u × v|| (1.11)

Example 1.7. Calculate the area of the triangle 4P QR where P = (2, 4, −7), Q = (3, 7, 18) and
R = (−5, 12, 8).
−−→ −→
Solution 1.6. Let u = P Q and v = P R as shown on Figure 1.5. Then u = (3, 7, 18) − (2, 4, −7) =
(1, 3, 25) and v = (−5, 12, 8) − (2, 4, −7) = (−7, 8, 15)

Figure 1.5: Geometrical area of the parallelogram


9

The area of the triangle 4P QR is given by:

1
A = ||u × v||
2
1
= ||(1, 3, 25) × (−7, 8, 15)||
2
1
= ||(−155, −190, 29)||
2
1q
= (−155)2 + (−190)2 + 292
2
1√
= 60966
2
= 123.46 units2

Theorem 1.5.5. Properties of the cross product


For any vectors u, v and w in R3 and λ ∈ R, we have that

(a) u × v = −v × u Anti-commutative law

(b) u × (v + w) = u × v + u × w Distributive law

(c) (λu) × v = u × (λv) = λ(u × v) Associative law

(d) u × 0 = 0 = 0 × u

(e) u × u = 0

(f) u × v = 0, if and only if u k v. i.e. θ = 0


π
(g) u × v = ||u||||v||n̂|| if and only if u ⊥ v. i.e. θ = 2

(h) u × v = sin θn̂ if and only if u and v are unit vectors.

Note. î × î = ĵ × ĵ = k̂ × k̂ = 0, î × ĵ = k̂, î × k̂ = −ĵ and ĵ × k̂ = î.


start here 27/01/2021

10

CHAPTER 2

LINES AND PLANES

In Chapter 1, we have looked at some operations on vectors. In this chapter, our focus will be on
some familiar geometric objects such as lines and planes in terms of vector calculus. Using vectors
makes it easier for us study objects in a 3-dimensional Euclidean space (R3 ).

2.1 LINES

We first consider lines, were we look at an equation of the line through a point, parallel to a vector
and an equation of the line through two points.

2.1.1 Line through a point, parallel to a vector

Let P = (x0 , y0 , z0 ) be a point in R3 , v = (a, b, c) be a nonzero vector and L be a line through the
point P which is parallel to the vector v as shown on Figure 2.1 below.

Figure 2.1: Line through a point parallel to a vector.

Let r0 = (x0 , y0 , z0 ) be the position vector relative to the origin 0 (i.e. r0 is the vector pointing
from the origin to the point P ). Multiplying v by a scalar λ lengthens or shrinks v while preserving
its direction if λ > 0 and reversing its direction if λ < 0. Thus, from Figure 2.1 every point of the
line can be obtained by adding the vector λv to r0 for some λ ∈ R. Therefore as λ varies over R,
11

the vector r0 + λv will point to every point on the line L.

In summary, the vector representation of the equation of the line L through the point P (x0 , y0 , z0 )
parallel to the vector v = (a, b, c) is given by

r = r0 + λv; λ∈R (2.1)

where r0 = (x0 , y0 , z0 ) is the vector pointing to P from the origin. Furthermore, we have that
r = (x, y, z), thus

(x, y, z) = (x0 , y0 , z0 ) + λ(a, b, c) = (x0 + λa, y0 + λb, z0 + λc); λ∈R (2.2)

So, the parametric representation of the equation of the line L with parameter λ, through the
point P and parallel to the vector v is given by

x = x0 + λa, x = y0 + λb and x = z0 + λc; λ∈R (2.3)

If a, b, c 6= 0, then we can solve for the parameter λ in terms of x, y and z respectively to get
the symmetric representaion of the equation of the line L through the point P (x0 , y0 , z0 ) and
parallel to the vector v as:
x − x0 y − y0 z − z0
= = (2.4)
a b c

Example 2.1. Find the equation of the line through the point P (2, 3, 4) and parallel to the vector
v = (4, −1, 6) in

(a) Vector form

(b) Parammetric form

(c) Symmetric form

Hence, find any two distinct points of the line L from P .


12

Solution 2.1. (a) Let r0 = (2, 3, 5), thus the vector equation of L is given by:

v = r0 + λv

= (2, 3, 5) + λ(4, −1, 6)

(b) We have that

(x, y, x) = (2, 3, 5) + λ(4, −1, 6) = (2 + 4λ, 3 − λ, 5 + 6λ)

∴ x = 2 + 4λ, y = 3 − λ and z = 5 + 6λ are the parametric equations of the line L.


x−2 y−3 z−5
(c) λ = 4 = −1 = 6 is the symmetric equation of L.

Let λ1 = 1 and λ2 = 2, we have P1 (6, 2, 11) and P2 (10, 1, 17) as the points on L

2.1.2 Line through two points

Let P1 (x1 , y1 , z1 ) and P2 (x2 , y2 , z2 ) be distinct points in R3 and let L be the line through P1 and P2 .
Let r1 = (x1 , y1 , z1 ) and r2 = (x2 , y2 , z2 ) be vectors pointing to P1 and P2 respectively as shown
on Figure 2.2.

Figure 2.2: Line through two points.

From Figure 2.2, we observe that r2 − r1 is the vector from points P1 to P2 . Thus for any λ ∈ R,
13

r1 + λr2 − r1 will give us the whole line L as λ varies across R.

In summary, the different representations of the equation of the line L through the points P1 (x1 , y1 , z1 )
and P2 (x2 , y2 , z2 ) are given by:

(1.) Vector Representation:


r = r1 + λ(r2 − r1 ); λ∈R (2.5)

(2.) Parametric Representation:

x = x1 + λ(x2 − x1 ), y = y1 + λ(y2 − y1 ); and z = z1 + λ(z2 − z1 ); λ∈R (2.6)

(3.) Symmetric Representation:

x − x1 y − y1 z − z1
= = (2.7)
x2 − x1 y2 − y1 z2 − z1

Example 2.2. Find the equation of the line L through the points P1 (−3, 1, −4) and P2 (4, 4, −6)
in:

(a) Vector form

(b) Parametric form

(c) Symmetric form


−−−→
Solution 2.2. The vector v parallel to L is given by v = P1 P2 = (7, 3, −2)

(a) The vector for equation of L is given by

r = (x, y, z) = r1 + λv = (−3, 1, −4) + λ(7, 3, −2); λ∈R

(b) The parametric equation of L is given by:

x = −3 + 7λ, y = 1 + 3λ, z = −4 − 2λ; λ∈R


14

(c) The symmetric equation of L is given by:

x+3 y−1 z+4


= =
7 3 −2

2.1.3 Distance between a point and a line

Let L be a line in R3 in vector form r0 + λv, λ ∈ R and let Q be a point not on L. The distance
d from Q to L is the length of the line segment from Q to L which is perpendicular to L as shown
on Figure 2.3 below.

Figure 2.3: Distance between a point and a line.

Pick a point P on L and let u be a vector from P to Q. If θ is the angle between u and v, then
d = ||u|| sin θ. Since ||u × v|| = ||u||||v|| sin θ and u 6= 0, then

||u × v||
d= (2.8)
||u||

Example 2.3. Find the distance d from the point Q(1, 1, 1) to the line x = −3 + 7λ, y = 1 + 3λ
and z = −4 − 2λ; λ ∈ R.

Solution 2.3. L can be represented as r = r0 + λv. Thus r0 = (−3, 1, −4) and v = (7, 3, −2). Since
−−→
the point P (−3, 1, −4) is on L then u = P Q = (4, 0, 5) so that
15

i j k
u × v = 7 3 −2 = (15, −43, −12)
4 0 5

q √
||u × v|| = 152 + (−43)2 + (−12)2 = 2218 ≈ 47.1 units and
q √
||u|| = 72 + 32 + (−2)2 = 62 ≈ 7.87 units

||u × v||
∴d=
||u||

2218
= √
62
= 5.98 units

Theorem 2.1.1. Let L1 and L2 be lines with vectors equations r1 + λv1 and r2 + λv2 respectively,
then

(1.) L1 k L2 iff v1 k v2 .

(2.) L1 ⊥ L2 iff v1 ⊥ v2 .

(3.) If L1 and L2 intersect then we can use their parametric representations to find theirc point of
intersection.

Example 2.4. Find the point of intersection (if any) of the following lines revise

x+1 y−2 z−1


L1 : = =
3 2 −1
y−8 z+3
L2 : x+3= =
−3 2
16

Solution 2.4. We first write the lines in parametric forms

L1 : x = −1 + 3λ1 , y = 2 + 2λ1 and z = 1 − λ1 ; λ1 ∈ R

L2 : x = −3 + λ2 , y = 8 − 3λ2 and z = −3 + λ2 ; λ2 ∈ R

The lines L1 and L2 intersect when (−1 + 3λ1 , 2 + 2λ1 , 1 − λ1 ) = (−3 + λ2 , 8 − 3λ2 , −3 + λ2 ), were
λ1 , λ2 ∈ R. Thus

−1 + 3λ1 = −3 + λ2 =⇒ λ2 = 2 + 3λ1

2 + 2λ1 = 8 − 3λ2 =⇒ 2 + 2λ1 = 8 − 3(2 + 3λ1 ) = 2 − 9λ1

=⇒ 11λ1 = 0 =⇒ λ1 = 0 and λ2 = 2 + 2(0) = 2

Check: 1 − λ1 = −3 + 2λ2 =⇒ 1 − 0 = 1 = −3 + 2(2)

By letting λ1 = 0 in the equation of L1 or letting λ2 = 2 in the equation of L2 we will get the point
P (−1, 2, 1) as the point of intersection.
17

2.2 PLANES

In this section, we will look at equations of planes in the Euclidean space R3 .

2.2.1 Plane through a point, perpendicular to a vector

Let π be a plane in R3 , containing a point P0 (x0 , y0 , z0 ). Let n = (a, b, c) be a non-zero vector


perpendicular to π. n is called a normal vector to the plane. Let P (x, y, z) be a point in the
plane π. The vector r = (x − x0 , y − y0 , z − z0 ) lies in the plane as shown on Figure 2.4

If r 6= 0, then r ⊥ n and hence n · r = 0. If r = 0, still n · r = 0.

Figure 2.4: Plane π through a point, ⊥ to a vector.

Definition 2.2.1. Let π be a plane in R3 , P0 (x0 , y0 , z0 ) be a point in π, and let n = (a, b, c) be a


non-zero vector perpendicular to pi. Then π consists of all points P (x, y, z) satisfying the vector
equation.
n·r=0 (2.9)

where r = (x − x0 , y − y0 , z − z0 ), or equivalently:

a(x − x0 ) + b(y − y0 ) + c(z − z0 ) = 0 (2.10)

Equation (2.10) is called the point-normal form equation of the plane π.


18

Example 2.5. Find the equation of the plane π containing the point P0 (−3, 1, 3) and perpendic-
ular to the vector n = (2, 4, 8).

Solution 2.5. By Equation (2.10) we have the plane π containing all point P (x, y, z) such that:

(2, 4, 8) · (x + 3, y + 1, z − 3) = 2(x + 3) + 4(y − 1) + 8(z − 3) = 0

By multiplying the terms in Equation (2.10) and add the constant we will get the equation of the
plane in normal form:
ax + by + cz + d = 0 (2.11)

For example, the norm al form of the plane in Example 2.5 above is

2x + 4y + 8z − 22 = 0

2.2.2 Plane containing three non-collinear points


−−→ −→
If P , Q and R are non-collinear points in R3 , then P Q and P R are no-zero vectors which are not
−−→ −→ −−→ −→ −−→ −→
parallel. So, the cross product P Q × P R is perpendicular to both P Q and P R. So, P Q and P R
−−→ −→
lie in the plane π through the point P with the normal vector n = P Q × P R as shown on Figure
2.5 below.

Figure 2.5: Plane π, containing noncollinear points P , Q and R.

Example 2.6. Find the equation of the plane π containing the points P (2, 1, 3), Q(1, −1, 2) and
19

R(3, 2, 1).
−−→ −→
Solution 2.6. We have vectors P Q = Q − P = (−1, −2, −1) and P R = R − P = (1, 1, −2). The
plane π has a normal vector

−−→ −→
n = PQ × PR

= (−1, −2, −1) × (1, 1, −2)

= (5, −3, 1)

So, the plane π consists of all points (x, y, z) such that:

5(x − 2) − 3(y − 1) + (z − 3) = 0

In point-normal form, and

5x − 3y + z − 10 = 0

In normal form.

2.2.3 Distance between a point and a plane

The distance between a point P and a plane π in R3 is the length of the line segment from the
point P to the plane π which is perpendicular to the plane as shown on Figure 2.6.

Definition 2.2.2. Let P (x0 , y0 , z0 ) be a point in R3 and π be a plane with normal form ax + by +
cz + d = 0 that does not contain P . The distance D from P to π is given by:

|ax0 + by0 + cz0 + d|


D= √ (2.12)
a2 + b2 + c2
20

Figure 2.6: Distance between a plane π and a point P .

Example 2.7. Find the distance D from the point P (2, 4, −5) to the plane π: 5x − 3y + z − 10 = 0

Solution 2.7.

|5(2) − 3(4) + (−5) − 10|


D= p
52 + (−3)2 + 12
17
=√
35
= 2.87 units

2.2.4 Distance between two parallel planes

Let π1 and π2 be planes such that π1 k π2 , then the distance between π1 andπ2 is the distance
between then plane π1 and any point on the plane π2 or vice-versa.

Figure 2.7: Distance D between planes π1 and π2 .


21

Example 2.8. Find the distance D between the planes

π1 : 2x − 3y + 4z + 10 = 0

π2 : 4x − 6y + 8z − 12 = 0

Solution 2.8. Since P (3, 0, 0) is the point on π2 , then the distance between π1 and π2 is the distance
between π1 and the point P . Thus

|2(3) − 3(0) + 4(0) + 10|


D= p
22 + (−3)2 + 42
16
=√
29
= 2.97 units

2.2.5 Line of intersection of two planes

If two planes intersect they do so in a line as shown in Figure 2.8 below.

Figure 2.8: Line of intersection between planes π1 and π2 .

Suppose two planes π1 and π2 with normals n1 and n2 , respectively, intersect in a line L. Since
n1 × n2 ⊥ n1 , n2 then n1 × n2 k π1 , π2 . Thus n1 × n2 is parallel to the line of intersection of π1
and π2 . i.e. n1 × n2 k L. So, the vector form equation of L is given by:

L: r0 + λ(n1 × n2 ) : λ∈R (2.13)


22

Where r0 is any vector pointing to the point belonging to both π1 and π2 . To find a point belong-
ing to both π1 and π2 , we find a common solution (x, y, z) to the normal form equation of the planes.

Example 2.9. Find the line of intersection L of the planes

π1 : 5x − 3y + z − 10 = 0

π2 : 2x + 4y − z + 3 = 0

Solution 2.9. The plane π1 has a normal n1 = (5, −3, 1) and π2 has a normal n2 = (2, 4, −1). Since
n1 and n2 are not scalar multiple of each other then n1 and n2 are not parallel. Hence they can
intersect. P (x, y, z) on both planes will satisfy the following system of equations:

5x−3y + z = 10

2x+4y − z = −3

Let x = 0, then

−3y + z = 10

4y − z = −3

y=7

Thus z = 31, so P (0, 7, 31) is on L. Furthermore

n1 × n2 = (5, −3, 1) × (2, 4, −1)

= (−1, 7, 26)

The equation of L is given by:


23

L: (0, 7, 31) + λ(−1, 7, 26) : λ∈R

Corollary 2.2.3. Let π1 and π2 be planes in R3 , with normals n1 and n2 respectively. Then

(a) π1 k π2 if and only if n1 k n2

(b) π1 ⊥ π2 if and only if n1 ⊥ n2

In vector form. Or in parametric form as

x = −λ, y = 7 + 7λ and z = 31 + 26λ; λ∈R


24

CHAPTER 3

MATRICES AND SYSTEMS OF LINEAR EQUATIONS

3.1 INTRODUCTION

A matrix is simply a rectangular array of numbers. Matrices are often used to organise information
into categories corresponding to the rows and colums of the matrix.

Definition 3.1.1. An m × n matrix A is a rectangular array of numbers with m rows and n


columns, denoted by A = [aij ]. Thus

 
a11 a12 ... a1n
 
 
 a21 a22 ... a2n 
A = [aij ] =  . (3.1)
 
 . .. .. .. 
 . . . . 

 
am1 am2 . . . amn

We say that matrix A has m × n dimensions. The number aij are called the entriesor element of
matrix A. Thus aij is the entry at the ith row and j th column.

Example 3.1. Here are some examples of matrices.


 
1 5 0 3
 
2 −1 4 3 ,
A=  is a 3 × 4 matrix.
 
7 2 1 6
 
11
B =  , is a 2 × 1 matrix (column vector).
9
 
C= 2 7 3 , is a 1 × 3 matrix (row vetor), and

D = [3], is a 1 × 1 matrix.
25

Definition 3.1.2. Square matrix


A matrix in which the number of rows are equals to the number of columns is called a square matrix
and it is denoted by An×n .

3.2 MATRIX ALGEBRA

Now that we know what matrices are, we can then perform some of the common algebraic operations
such as: addition, subtraction, matrix transpose, scalar multiplication and multiplication on them.
We begin by describing the equality of matrices.

3.2.1 Equality of matrices

Let A = [aij ] be an m × n matrix and B = [bij ] be a p × q matrix. Then matrices A and B are said
to be equal; written as A = B if and only if:

(a) A and B have the same number of rows and colums. i.e. m = p and n = q, and

(b) aij = bij for each i and j.


 
 2 3 9
   
2 3 a 2 3 9  
Example 3.2. If A =  , B =   and C = 
−3 6 1, then A = B if and only
    
b 6 1 −3 6 1  
0 5 2
if a = 9 and b = −3. However, A 6= C and B 6= C.

3.2.2 Matrix Addition or Subtraction

Let A = [aij ] amd B = [bj ] be m × n matrices, the sum or difference of A and B denoted by A ± B
is the m × n matrix whose entry at the ith row and j th column is aij ± bij for each i and j such
that:
A + B = [aij ± bij ] (3.2)

Note. Matrices that can be added and subtracted are said to be confirmable for addition and
subtraction. A + B = B + A. i.e matrix addition is commutative.
26

3.2.3 Matrix Transpose

Let A = [aij ] be an m × n matrix, then the transpose of A, denoted by AT is an n × m matrix


obtained by interchanging the rows and columns. Thus

AT = [aij ]T = [aji ] (3.3)

   
2 1 7
 
2 6 3
 
, B = 7 3 2 , then AT =  T
   
Example 3.3. If A =  6 0 and B = 3.
    
1 0 4    
3 4 2

3.2.4 Matrix Scalar Multiplication

Let A = [aij ] be an m × n matrix and λ be a scalar (real or complex). The scalar multiplication
of A by λ, written as λA can be obtained by multiplyinge very elementv of A by λ, resulting in an
m × n matrix defined by:
λA = [λaij ] (3.4)
 
2 −6 7
Example 3.4. Let A =  , λ1 = −1 and λ2 = 2, then
1 4 15
   
 2 6 −7  4 −12 14
λ1 A = −A =   and λ2 A = 2A =  .
−1 −4 −15 2 8 30

3.2.5 Matrix Multiplication

Let A = [aij ] be an m×n matrix, whose r th row is the row vector ar , and let B = [bij ] be a p×q ma-
trix whose s th column is the column vector bs such that m = q or n = p. The product of A and B
denoted by AB is the m×q matrix whose element at the r th row and s th column is defined as ar ·bs

If crs = ar · bs = ar1 b1s + ar2 b2s + · · · + arn bns , then


27

 
a1 · b1 a1 · b2 ... a1 · bq
 
 
 a2 · b1 a2 · b2 ... a2 · bq 
AB = [crs ] =  . (3.5)
 
.. .. .. .. 

 . . . 

 
am · b1 am · b2 . . . am · bq

Note. AB 6= BA. i.e. Matrix multiplication is not commutative.

 
4 1
 
1 4 −3  
Example 3.5. Given A =  2 6. Find AB and BA.
 and B = 
  
2 5 4  
0 3
 
    4
 
Solution 3.1. a1 = 1 4 −3 and a2 = 2 5 4 are the row vectors of A and b1 = 
2 and

 
0
 
1
 
6 are the column vectors of B. So,
b2 =  
 
3

 
a1 · b1 a1 · b2 
AB =  
a 2 · b1 a 2 · b2
 
12 16
= 
18 44

 
1
     
Simmilarly, b1 = 4 1 , b2 = 2 6 , b3 = 0 3 are the row vectors of B and a1 =  ,
2
   
4 −3
a2 =   and a3 =   are the column vectors of A. So,
5 4
28

 
b1 · a1 b1 · a2 b1 · a3 
 
BA = 
b2 · a1 b2 · a2 b2 · a3 

 
b3 · a1 b3 · a2 b3 · a3
 
21 −8
6
 
=
14 38 18 

 
6 15 12

Theorem 3.2.1. Properties of matrices


Let A, B and C be matrices confirmed under the previous operations, and let λ be a scalar. Then

(a) If AB and BA are both defined, in general AB 6= BA anti-commutative law

(b) A(BC)=(AB)C Associative law

(c) (λA)B = A(λB) Associative law

(d) A(B +C) = AB +AC Distribustive law

(e) (A+B)C = AC +BC Distribustive law

Theorem 3.2.2. Transposition of a product


Let A and B be matrices confirmed under multiplication, then

(AB)T = B T AT (3.6)

Theorem 3.2.3. Let A be a square matrix, and m and n be positive integers, then

(a) Am = A n
| · A{z. . . A}, A = A
m n
| · A{z. . . A} and A · A = A
m+n , and

m times n times

(b) (A + B)T = AT + B T and (ABC)T = C T B T AT .

Theorem 3.2.4. Let A = [aij ] be a square matrix, then the elements on the line extending from
the top left corner to the bottom right corner is called the leading (main) diagonal of A, and it
29

contains the n elements a1 , a22 , . . . and ann . Thus

 
a11 a12 . . . a1n
 
 
 a21 a22 ... a2n 
A= . (3.7)
 
 . .. .. .. 
 . . . . 

 
an1 an2 . . . ann

Definition 3.2.5. Let A = [aij ] be a square matrix, then the trace of A denoted by tr(A) is the the
sum of all elements on the main diagonal. Thus

tr(A) = a11 + a22 + · · · + ann (3.8)

3.3 SPECIAL MATRICES

In this section, we will look at some of the special matrices. These matrices are considered to have
special properties and they are often used in Mathematics and Engineering.

Definition 3.3.1. Diagonal Matrix


A square matrix in which all elements away from the main diagonal are zerps, but not every element
on the main diagonal is zero is called a diagonal matrix.

 
1 0 0
 
Example 3.6. A = 
0 2  is a diagonal matrix.
0
 
0 0 3

Definition 3.3.2. Identity Matrix


A square matrix with ones on the main diagonal and zeros off the main diagonal is called an
30

identity matrix. The identity matrix is denoted by In×n or simply just I, and it is given by

 
1 0 ... 0
 
 
0 1 ... 0
In×n =  . . (3.9)
 
. . .. .. 
. . . .

 
0 0 ... 1

Note. Let A be an n × n and I be an n × n identity matrix, then AI = A = IA

Definition 3.3.3. Upper Triangular Matrices


Upper triangular matrices are matrices in which all elements below the main diagonal are zeros.

 
1 3 −1 0
 
 
0 2 −6 1
e.g. U = 
 

0 0

−3 2

 
0 0 0 4

Definition 3.3.4. Lower Triangular Matrices Lower triangular matrices are matrices in which
all elements above the main diagonal are zeros.

 
2 0 0 0
 
 
 1 0 0 0
e.g. L = 
 

 3

−2 5 0

 
−2 4 7 3

Definition 3.3.5. Symmetric Matrices


An n × n matrix A = [aij ] is said to be symmetric if aij = aji for all i and j.

 
 1 5 −3
 
e.g. M = 
 5 4 2

 
−3 2 7
31

Theorem 3.3.6. If A is symetric, then AT = A

Definition 3.3.7. Skew-symmetric matrix


An n × n matrix A = [aij ] is said to be skew-symmetric if and only if aij = −aji for all i and j.

 
0 3 −5 6
 
 
−3 0 2 −4
e.g. S = 
 

 5 −2 0 −1


 
−6 4 1 0

Theorem 3.3.8. If A is skew-symmetric, then AT = −A.

Definition 3.3.9. Orthogonal Matrix


An orthogonal matrix Q is a matrix such that QQT = I = QT Q.

3.4 DETERMINANTS

Every square matrix A = [aij ] with numbers as entries is associated with a single unique number
called the determinant, written as det(A) or |A|.

Definition 3.4.1. Let A be a square matrix, then the determinant of A is given by

a11 a12 . . . a1n


a21 a22 . . . a2n
det(A) = |A| = .. .. .. .. (3.10)
. . . .
an1 an2 . . . ann

The number n is called the order of the determinant of A.


32

3.4.1 Determinant of a 2 × 2 matrix


 
a11 a12 
Let A be a 2 × 2 matrix, i.e.  , then det(A) is given by
a21 a22

det(A) = |A| = a11 a22 − a12 a21 (3.11)

 
3 −1
e.g. For A =  , det(A) = 18 + 2 = 20.
2 6

Definition 3.4.2. Minors of a determinant of order n


The minor Mij associated with element aij of the nth order determninat (3.10) is the determinant
of order n − 1 formed from det(A) by deleting the elements in the i th row and j th column.

 
a11 a12 a13 
 
Example 3.7. The minors of a 3 × 3 matrix A = 
a21  are given by:
a22 a23 
 
a31 a32 a33

a22 a23 a21 a23 a21 a22


M11 = , M12 = , M13 = ,
a32 a33 a31 a33 a31 a32

a12 a13 a11 a13 a11 a12


M21 = , M22 = , M23 = ,
a32 a33 a31 a33 a31 a32

a12 a13 a11 a13 a11 a12


M31 = , M32 = , and M33 =
a22 a23 a21 a23 a21 a22

Definition 3.4.3. Cofactors of determinants of oder n


The cofactor Cij associated with the elements aij of the n th order determinant (3.10) is defined in
terms of the minor Mij as

Cij = (−1)i+j Mij for i, j = 1, 2, . . . , n (3.12)


33

So, the n th order determinant has n2 minors and cofactors.


 
a11 a12 a13 
 
Example 3.8. The cofactors of a 3 × 3 matrix A = 
a21 a22 a23  are given by:

 
a31 a32 a33

C11 = M11 , C12 = −M12 , C13 = M13 ,

C21 = −M21 , C22 = M22 , C23 = −M23 ,

C31 = M31 , C32 = −M32 and M33 = C33 .

2 −3
Example 3.9. Find the minors and cofactors of det(A) =
1 4
Solution 3.2. Minors: M11 = 4, M12 = 1, M21 = −3 and M22 = 2.
Cofactors: C11 = M11 = 4, C12 = −M12 = −1, C21 = −M21 = 3 and C22 = M22 = 2.

Definition 3.4.4. Determinants of order n in terms of cofactors


The n th order determinnat det(A) in which the element aij is associated with the cofactor Cij for
i, j = 1, 2, . . . , n is defined as

a11 a12 . . . a1n


a21 a22 . . . a2n Xn
det(A) = |A| = . .. .. .. = aij Cij (3.13)
.. . . . j=1

an1 an2 . . . ann

where i is any row of your choice.

 
1 4 −1
 
Example 3.10. Find the determinant of the matrix A = 
2 0  in terms of cofactors.
3
 
1 2 1

P3
Solution 3.3. det(A) = j=1 a2j C2j = a21 C21 + a22 C22 + a23 C23
34

4 −1
C21 = (−1)2+1 M21 = − = −6
2 1

1 −1
C22 = (−1)2+2 M22 = =2
1 1

1 4
C23 = (−1)2+3 M23 = − =2
1 2

∴ det(A) = 2(−6) + 0(2) + 3(2) = −12 + 6 = −6

Theorem 3.4.5. Laplace expansion theorem


Let A = [aij ] be an n × n matrix. Then
Pn
i) det(A) = ai1 Ci1 + ai2 Ci2 + · · · + ain Cin = j=1 aij Cij for any fixed row i such that 0 ≤ i ≤ n.
Pn
ii) det(A) = a1j C1j + a2j C2j + · · · + anj Cnj = i=1 aij Cij for any fixed column j such that
0 ≤ j ≤ n.

Theorem 3.4.6. Properties of determinants


Let A = [aij ] be an n × n matrix, then the determinant of A has the following properties:

(1.) If any row or column of A contains zeros, then det(A) = 0.

(2.) det(A) = det(AT ).

(3.) det(λA) = λ det(A) for any scalar λ.

(4.) Interchanging any two rows or columns of A changes the sign of the determinant.

(5.) If any two rows or columns of A are proportional, then det(A) = 0.

(6.) Adding a multiple of a row (or column ) to another row (or column) leaves the value of the
determinant unchanged.

(7.) Let A and B be two n × n matrices, then det(AB) = det(A) det(B).


35

3.5 SOLUTION TO SYSTEMS OF LINEAR EQUATIONS BY MATRICES

In this ection we present nethods for solving systems of linear equations. We cosinder two such
methods: Cramer’s rule and Gaussian Elimination.

3.5.1 Cramer’s rule

The solution to the system of n linear equations in n unknowns x1 , x2 , . . . , xn

a11 x1 + a12 x2 + · · · + a1n xn = b1

a21 x1 + a22 x2 + · · · + a2n xn = b2


(3.14)
..
.

an1 x1 + an2 x2 + · · · + ann xn = bn

is given by
det(Ai )
xi = ; for i = 1, 2, . . . , n (3.15)
det(A)

where det(A) is the detrminant of the coefficient matrix with elements aij , and det(Ai ) is the de-
terminant obtained from the coefficient matrix by replacing its i th column with column containing
the numbers b1 , b2 , . . . , bn .

Example 3.11. Use cramer’s rule to solve the following system of linear equations

2x1 − x2 − x3 = −3

−x1 + x2 + x3 = 4

x1 − x2 + 6x3 = 17
36

   
 2 −1 −1 −3
   
Solution 3.4. The coefficient matrix is given by A = 
−1 1 1 and b =  4 . Thus
 
   
1 −1 6 17

2 −1 −1
det(A) = −1 1 1 =7
1 −1 6

−3 −1 −1
det(A1 ) = 4 1 1 =7
17 −1 6

2 −3 −1
det(A2 ) = −1 4 1 = 14
1 17 6

2 −1 −3
det(A3 ) = −1 1 4 = 21
1 −1 17

Therefore

det(A1 ) 7
x1 = = =1
det(A) 7
det(A2 ) 14
x2 = = =2
det(A) 7
det(A3 ) 21
x3 = = =3
det(A) 7

∴ x1 = 1, x2 = 2 and x3 = 3 is the solution to the given system of equations.

3.5.2 Gaussion Elimination

Definition 3.5.1. The Augmented matrix of a linear system of equations


37

Consider the sytem of n linear equations (3.14). We can write the linear system (3.14) in the
matrix form as follows:
 
a11 a12 . . . a1n b1
 
 
 a21 a22 . . . a2n b2 
A= . (3.16)
 
 . .. .. .. .. 
 . . . . .

 
an1 an2 . . . ann bn

Matrix A is called the Augmented Matrix of the sytem (3.14).

Theorem 3.5.2. Elementary Row Operations


There are three types of elementary row operations used when solving the system of equations (3.14)
by elimination:

(1.) The interchanging of two rows.

(2.) Scaling of row by a nonzero constant.

(3.) Addition of a scalar multiple of a row to another.

Definition 3.5.3. Row equivalence of Matrices


Two m × n matrices A and B are said to be equivalent if one can be obtained from the other by a
sequences of elementary row operations. Row equivalence between matries A and B is denoted by
A ∼ B.

Theorem 3.5.4. Let A, B and C be m × n matrices. Then

i) A ∼ A, B ∼ B and C ∼ C. i.e every matrix is row equivalent to itself. Reflexive property

ii) If A ∼ B then B ∼ A. Symmetric prop-


erty

iii) If A ∼ B and B ∼ C, then A ∼ C. Transitive prop-


erty
38

Definition 3.5.5. Row-Echelon and Reduced Row-Echelon forms of a matrix


A matrix A is said to be in row-echelon form if it satisfies the following conditions:

(1.) The first nonzero number in each row, called the leading entry is 1.

(2.) In any two successive rows i and i + 1 that do not consist entirely of zeros, the leading entry
in the (i + 1) th row lies to the bottom right of the leading entry in the i th row.

(3.) Any rows consisting intirely of zeros lie at the bottom of the matrix.

Matrix A is said to be in reduced row-echelon form if it is in row-echelon form and also


satisfies the following condition:

(4.) Every number above and below the main diagonal is zero.

Example 3.12. The matrices

 

  1 1 1 1 1 1
 
1 0 5 7
0
 0 1 2 0 1

   
0 0 1 1 and A = 
A= are in row-echelon form and matrices
0 0 0 1 5 2
  
  
 
0 0 0 0 0
 0 0 0 1 3

 
0 0 0 0 0 0
   
1 0 5 7 1 0 0 9 2
   
C=
0 0 1 1 and D = 0 1 0 2 3 are in reduced row-echelon form.
  
   
0 0 0 0 0 0 1 1 0

Rules for reducing a matrix to row-echelon form

(1.) Start by obtaining 1 in the top left corner. Then obtain zeros below 1 by adding appropriate
multiples of the 1 th row to the row below it.

(2.) Next, obtain a leading 1 in the next row, and then obtain zeros below it.

(3.) At each stage make sure that every leading entry is to the right of the leading entry in the row
above it. Rearrage the rows if necessary.
39

(4.) Continue the process until you arrive at a matrix in row-echelon form.

This is how the process work for a 3 × 4 matrix

     
1    1    1   
     
0    ⇒ 0 1   ⇒ 0 1  
     
     
0    0 0   0 0 1 

Once the augmented matrix is in row-echelon form, we can solve the corresponding linear system
using Back-Substitution. This technique is called Gaussian Elimination.

Main steps used when solving a system of linear equations by Gausssian Eliminination

(1.) Augmented matrix: Write the augmented matrix of the system.

(2.) Row-mchelon form: Use elementary row operations to change the augmented matrix to
row-echelon form.

(3.) Back-substitution: Write the new system of equations corresponds to the row-echelon form
of the augmented matrix and solve the system by back substitution.

Example 3.13. Solve the following system of linear equations using Gaussian Elimination.

4x + 8y − 4z = 4







3x + 8y + 5z = −11




−2x + y + 12z = −17

Solution 3.5. We first write the augmented matrix of the system and then use elementary row
operations to put it in row-echelon form.
40

 
 4 8 −4 4 
 
(1.) Augmented matrix: 
 3 8 5 −11

 
−2 1 12 −17
     
 4 8 −4 4   1 2 −1 1 1 1 2 −1 1 
    4 R1 → R1  
⇒ 3
−11 −11 ⇒
0 2 8 −14 R2 − 3R1 → R2 ⇒
 3 8 5  8 5 
 
     
−2 1 12 −17 −2 1 12 −17 0 5 10 −15 R3 + 2R1 → R3
     
1 2 −1 1  1 2 −1 1 1 2 −1 1 
 1    
 2 R2 → R2 ⇒ 0 1
−7  −7 ⇒
0 1 4 −7
0 1 4  4 
 
     
0 5 10 −15 0 0 −10 20 R3 − 5R2 → R3 0 0 1 −2 − 1 R3 → R3
10

Note. The blue colour means we need a 1 at that entry and the red one means we need a 0 at
that entry as we carryout elementary row operation on the augmentedc matrix.

 
1 2 −1 1
 
(2.) Row-echelon form: 
0 1 4 −7

 
0 0 1 −2
We now have an augmented matrix in row echelon form, and the corresponding system is:


x + 2y − z = 1







 y + 4z = −7




z = −2

(3.) Back-substitution: We can now use back substitution to solve the system. Thus

z = −2, , y = −7 − 4(−2) = 1 and x = 1 − 2(1) + (−2) = −3

∴ The solution to the corresponding system is (−3, 1, −2).


41

3.5.3 Gauss-Jordan Elimination

Once we put the augmented matrix of a linear system in reduced row-echelon form we do not need
back-substitution to solve the system. To put the matrix in reduced row-echelon form, we use use
the following steps:

(1.) Use elementary row operations to put the matrix in row-echelon form.

(2.) Obtain zeros above each leading entry by adding appropriate multiples of the row containing
that entry to the rows above it. Begin with the last entry and work your way up.

This is how the process work for a 3 × 4 matrix

     
1    1  0  1 0 0 
     
0 1   ⇒ 0 1 0  ⇒ 0 1 0 
     
     
0 0 1  0 0 1  0 0 1 

Using reduced row-echelon form to solve the system of linear equations is called Gauss-Jordan
Elimination.

Example 3.14. Solve the system of linear equations in Example 3.13 by Gauss-Jordan elimination.

Solution 3.6. In Example 3.13 we used Gaussian elimination on the augmented matrix of the system
to arrive at an equivalent matrix in row-echelon form. We continue using elementary riw operations
on the last matrix, working up.
     
−1 R1 + R2 → R1 R1 − 2R2 → R1
1 2 1 1 2 0 −1 1 0 0 −3
     
 ⇒ 0 1 0 1  R2 − 4R1 → R2 ⇒ 0 1 0 1 
−7
0 1 4    

     
0 0 1 −2 0 0 1 −2 0 0 1 −2

We now have the equivalent matrix in reduced row-echelon form, and the corresponding system of
equations is
42


x = −3







 y=1




z = −2

∴ We immediately arrive at the solution (−3, 1, 2).

Exercise 1

Reduce the following matrix to row-echelon and reduced row-echelon forms.


 
0 1 2 0 3
 
 
2 4 8 2 4
 
 
1 2 4 2 2


 
1 3 6 1 5

3.5.4 Inconsistent and Dependent System

A linear system of equation may have exactly one solution, no solution or infinately ,any solutions.
The row echelon form of a system help us to determine which of the following cases applies.

Solution of a linear system in row-echelon form

Suppose the augmented matrix of the system of linear equations has been reduced into row-echelon
form by Gaussian elimination, then exactly one of the following is true:

(1.) No solution: If the row-echelon form contains a row that represents the equation 0 = c,
where c is a nonzero constant, then the sytem has no solution.

(2.) Exactly one solution: If each variable in the row-echelon form is a leading variable, then
the system has exactly one solution, which can be found by back-substitution.

(3.) Infinately many solutions: If the equivalent matrix in row-echelon has a row of zeros then
the system has infinately many solution.

Here is how the three cases are illustrated for a 3 × 4 matrix


43

No solution Exactly one solution Infinately many solution


     
1    1    1   
0 1   0 1   0 1  
     
0 0 0 c 0 0 1 c 0 0 0 0

Example 3.15. Matrices below, all in row-echelon form, illustrate the three cases

No solution Exactly one solution Infinately many solution


     
1 2 5 7 1 6 −1 3 1 2 −3 1
0 1 3 4 0 1 2 −2 0 1 5 −2
     
0 0 0 1 0 0 1 8 0 0 0 0

3.6 THE INVERSE MATRICES

Then operation of division is not defined for matrices. However some matrices have inverses ma-
trices but for a matrix A, A−1 6= 1
A.

Definition 3.6.1. Let A be a n × n matrix, then matrix A is said to be invertible and have an
associated inverse matrix denoted A−1 if

AA−1 = I = A−1 A (3.17)

Definition 3.6.2. An n × n matrix A is said to be nonsingular if and only if its inverse exists, and
to be singular if and only if its inverse does not exist. i.e. If det(A) 6= 0, then A has an inverse,
hence nonsingular, and if det(A) = 0, then A has no inverse, hence singular.

Theorem 3.6.3. Propertise of inverse matrices

i) I −1 = I.

ii) If A is nonsingular, so is A−1 , and (A−1 )−1 = A.

iii) If A is nonsingular, so is AT , and (A−1 )T = (AT )−1 .


44

iv) If A and B are nonsingular matrices, so is AB, and (AB)−1 = B −1 A−1 .

v) If A is nonsingular, then (A−1 )m = (Am )−1

3.6.1 Finding the inverse of a matrix by elementary row operations

Let A be an n × n matrix such that A is nonsingular, to find A−1 , we first write an augmented
matrix [A|I] and make use of elementary row operations to reduce it reduced row-cehelon form and
obtain [I|A−1 ].

 
 1 0 1
Example 3.16. Use elementary row operations to find A−1
 
given that A = 
−1 2 0.

 
0 1 1
Solution 3.7.
   
 1 0 1 1 0 0 1 0 1 1 0 0
   
[A|I] = 
−1 2 0 0 1  = 0 2
0 
 R1 + R2 → R2
1 1 1 0
   
0 1 1 0 0 1 0 1 1 0 0 1
   
1 0 1 1 0 0 1 0 1 1 0 0
 1  
= 1 1 1  R → R = 0 1 1 1 1
0 1 2 2 2 0 2 2 2 2 2 0
2
  
  
0 1 1 0 0 1 0 0 12 − 12 − 12 1 R3 − R2 → R3
   
1 0 1 1 0 0 −2 R1 − R3 → R1
1 0 0 2 1

1 1 1
   1
=
0 1 2 2 2 0
 =
0 1 0 1  R2 − 2 R3 → R2
1 −1
   
0 0 1 −1 −1 2 2R3 → R3 0 0 1 −1 −1 2

= [I|A−1 ]

 
 2 1 −2
A−1
 
∴ =
 1 1 −1

 
−1 −1 2
45

3.6.2 Finding the inverse of a matrix via the adjoint matrix

Definition 3.6.4. Let A be an n × n matrix and C be the associated cofcator matrix, the transpose
of C, C T is called the adjoint of A and it is denoted by Adj(A).

Theorem 3.6.5. Let A be a nonsingular n × n matrix, then the inverse of A is given by

1
A−1 − adj (A) (3.18)
det(A)

 
1 3 0
Example 3.17. Find A−1
 
given that A = 
2 1 1

 
1 0 1

Solution 3.8. The minors of |A| are:

1 1 2 1 2 1
M11 = = 1, M12 = = 1, M13 = = −1
0 1 1 1 1 0

3 0 1 0 1 3
M21 = = 3, M22 = = 1, M23 = = −3
0 1 1 1 1 0

3 0 1 0 1 3
M31 = = 3, M32 = = 1, M33 = = −5
1 1 2 1 2 1

The cofactors are:

C11 = 1, C12 = −1, C13 = −1

C21 = −3, C22 = 1, C23 = 3

C31 = 3, C32 = −1, C33 = −5

 
 1 −1 −1
 
∴C=
−3 1 3
 
3 −1 −5
46

The adjoint matrix is:

3
X
det(A) = a1j C1j
i=1

= a11 C11 + a12 C12 + a13 C13

= 1(1) + 3(−1) + 0

= −2

The inverse matrix is:

1
A−1 = Adj(A)
det(A)
 
 1 −3 3 
1 
=− −1 1 −1
2
 

−1 3 −5
 
1 3
− 2 2 − 32 
 
1
=
 2 − 12 1
2


 
1
2 − 32 5
2

3.6.3 Solving to systems of linear equations by matrix inversion

Consider the sytem of n linear equations (3.14) as

AX = B (3.19)

Where A is the coeffient matrix given by

 
a11 a12 . . . a1n
 
 
 a21 a22 ... a2n 
A= . (3.20)
 
 . .. .. .. 
 . . . . 

 
an1 an2 . . . ann
47

X is the column vector of unknows given by

 
x1
 
 
 x2 
X= .  (3.21)
 
 . 
 . 
 
xn

and B is the column vector given by


 
b1
 
 
 b2 
B=. (3.22)
 
.
.
 
bn

Then the solution to the sytem of equations (3.14) is given by

1
X = A−1 B = adj(A)B (3.23)
det(A)

Where A−1 is the inverse of the coefficient matrix A.

Example 3.18. Solve the following system of linear eqautions by matrix inversion





 x+y+z =6


 2y + 5z = −4




2x + 5y − z = 27

Solution 3.9. The coefficient matrix is given by:


 
1 1 1
 
A=
0 2 5

 
2 5 −1

and the column vector B is given by:


48

 
 6 
 
B=
−4

 
27

Thus, A−1 is given by

   
−27 6 3  1 − 92 − 19 
1   
 10 1 5

−  10
 = − 27
−3 −5 9 27

27 
  


4 1 2
−4 −3 2 27 9 − 27
49

CHAPTER 4

SEQUENCES AND SERIES

4.1 SEQUENCES

A sequence is a list of numbers written in a specific order. For example, the height that a bouncing
tennis ball reaches after each bounce is sequence. Such a sequence has definite pattern, hence de-
scribing the pattern allows us to predict the height that a ball reaches after any number of bounces.
The amount of money in our bank account at the end of each month, mortgage payments, etc. are
also sequences. Our economy is driven by formulas that generate these sequences; thus allowing us
to borrow money to buy our dream houses and cars before retirement. In this section we will study
sequences.

Definition 4.1.1. Mathematically, a sequence is a function whose domain is a set of positive


natural numbers N+ . i.e. f : N+ → R. The terms of the sequence are function values

f (1), f (2), f (3), . . . , f (n), . . .

We usually write {an } instead of the function notation {f (n)}. So, the terms of the sequence are
written as as:
a1 , a2 , a3 , . . . , an , . . .

The number a1 is called the 1st term, a2 is called the 2nd term, a3 is called 3rd term and in
general, an is called the nth term of the sequence.

Example 4.1. Consider the following examples of sequences

a) 5, 10, 15, 20, . . .


∴ The pattern of the sequence above can be described by the formula an = 5n.
50

Thus

a1 = 5 · 1 = 5, a2 = 5 · 2 = 10, a3 = 5 · 3 = 15

a4 = 5 · 4 = 20, a5 = 5 · 5 = 25, a6 = 5 · 6 = 30

To find the 100th term of this sequence, we use n = 100 and get a100 = 500.

b) 2, 4, 6, 8, 10, . . .
∴ The formula for the nth term is given by an = 2n

Thus

a1 = 2 · 1 = 2, a2 = 2 · 2 = 4, a3 = 2 · 3 = 6

a4 = 2 · 4 = 8, a5 = 2 · 5 = 10, a6 = 2 · 6 = 12

To find the 100th term of this sequence, we use n = 100 and get a100 = 200.

Example 4.2. Finding the terms of a sequence


Find the first five terms and the 100th term of the sequences defined by the following formulas:

a) an = 2n − 1

b) bn = n2 − 1
n
c) cn = n+1

(−1)n
d) dn = 2n

Solution 4.1. To find the first five terms , we substitute n = 1, 2, 3, 4 and 5 in the formula for the
n th term. To find the 100th term, we substitute n = 100 in the formula for the n th term. This
gives us
51

nth term 1st five term 100th


2n − 1 1,3,5,7,9 199
n2 − 1 0,3,8,15,24 9999
n 1 2 3 4 5 100
n+1 2,3,4,5,6 101
(−1)n
2n
1 1
− 2 , 4 ,− 18 , 16
1 1
,− 32 1
2100
Table 4.1: Finding the terms of the sequences

Example 4.3. Finding the nth term of a sequence


Find the nth term of the sequnces whose first several terms are given by
1 3 5 7
a) 2, 4, 6, 8, . . .

b) −2, 4, −8, 16, −32, . . .

Solution 4.2. a) We notice that the numerators are odd numbers and the denominators are even
numbers. Thus
2n − 1
an =
2n

b) We notice that these numbers are powers of 2, and the sign alternate, so we have the following
pattern
an = (−1)n 2n

4.1.1 Recursively defined sequences

There are some sequences who do not have simple defining formulas. The nth term of the sequences
may depend on some or all the terms preceeding it. A sequence defined in that way is called recur-
sive. Below are some examples of recursively defined sequences.

Example 4.4. Finding the terms of the recursively defined sequences


52

1.) Find the first five terms of the sequence defined by

a1 = 1 and

an = 3(an−1 + 2)

2.) The Fibonacci sequences 1

Find the first five terms of the sequence defined recursively by

F1 = 1, F2 = 1 and

Fn = Fn−1 + Fn−2

Solution 4.3. 1.) The formula for this sequence is recursive. It allows us to find the nth term, an if
we know the preceeding term, an−1 . Thus

a1 = 1

a2 = 3(a1 + 2) = 3(1 + 2) = 9

a3 = 3(a2 + 2) = 3(9 + 2) = 33

a4 = 3(a3 + 2) = 3(33 + 2) = 105

a5 = 3(a4 + 2) = 3(105 + 2) = 321

2.) To find Fn , we need to first find the two preceeding terms, Fn−1 and Fn−2 . Since are given F1
and F2 , then

F1 = 1

F2 = 1

F3 = F2 + F1 = 1 + 1 = 2

F4 = F3 + F2 = 2 + 1 = 3

F5 = F4 + F3 = 3 + 2 = 5
1
The Finonacci sequences, named after the 13th century Italian mathematician Fibonacci who used to solve
problems on the breeding of rabbits. This sequence also occurs in numerous other applications in nature.
53

4.1.2 Limits of Sequences

Let’s consider graphing a sequence. To graph a sequence {an } we plot the point (n, an ) as n ranges
over all possible values on the graph. For example consider the sequence { n+1
n2
}. The first few
points on the graph are:

3 4 5 6
(1, 2), (2, ), (3, ), (4, ), (5, ), ...
4 9 16 25

The graph for the first 25 terms is shown below.

n+1
Figure 4.1: limn→∞ n2

Notice that as n increases, the terms of the sequence get closer and closer to 0. We can say that 0
is the limit of the sequence. Thus

n+1
lim an = lim =0
n→∞ n→∞ n2

If such a limit exist, we say that the sequence {an } converges, otherwise it diverges.

Definition 4.1.2. Let {an } be a sequence, then

a) limn→∞ an = L, if the values of an approaches L as n approaches infinity.


54

b) limn→∞ an = ∞, if the values of an gets larger and larger without a bound as n approaches
infinity.

c) limn→∞ = −∞, if the values an are negative and get larger and larger without bound as n ap-
proaches infinity.

Theorem 4.1.3. Properties of limits of sequences


If {an } and {bn } are both convergent sequences, then

a) limn→∞ (an ± bn ) = limn→∞ an ± limn→∞ bn .

b) limn→∞ can = c limn→∞ an , for some constant c.

c) limn→∞ (an bn ) = (limn→∞ an )(limn→∞ bn ).


an limn→∞ an
d) limn→∞ bn = limn→∞ bn , provided limn→∞ bn 6= 0.
h ip
e) limn→∞ apn = limn→∞ an , provided that an ≥ 0.

Theorem 4.1.4. If limn→∞ |an | = 0, then limn→∞ an = 0.

Theorem 4.1.5. The sequence {rn } converges if −1 < r ≤ 1 and diverges if |r| > 1. Also,


0,
 if -1<r<1
lim rn = (4.1)
n→∞ 
1,

if r=1

Example 4.5. Determine whether the following series converges or diverges. If the sequence
converges determine its limit.
n o∞
3n2 −1
a) 10n+5n2 n=1
2n ∞
n o
e
b) n n=1
o∞
(−1)n
n
c) n n=1
n o∞
d) (−1)n
n=1
55

Solution 4.4. a)

3n2 − 1
lim an = lim
n→∞ n→∞ 10n + 5n2
3 − 12 )
= lim 2 10n
n→∞ n (
n +5
3 − n12 ) 1 10
= lim , , → 0, as n→∞
n→∞ 10 + 5 n2 n
n
3
=
5
n o∞
3n2 −1
∴ 10n+5n2
converges and its limit is 53 .
n=1

b)

e2n
lim an = lim
n→∞ n→∞ n
(e2n )0
= lim
n→∞ (n)0

2e2n
= lim =∞
n→∞ 1

1
c) limn→∞ |an | = limn→∞ n =0
Since limn→∞ |an | = 0, then limn→∞ an = 0, by Theorem (4.1.4).
o∞
(−1)n
n
∴ n converges and its limit is 0.
n=1

n o∞
d) For the sequence (−1)n , we have r = −1, then by Theorem (4.1.5), the sequence diverges.
n=0

4.2 PARTIAL SUMS OF A SEQUENCE

In calculus, we often interested in adding the terms of a sequence. This yields the following
definition.

Definition 4.2.1. The partial sums of a sequence


For the sequence
a1 , a2 , a3 , . . .
56

the partial sums are:

S1 = a1

S2 = a1 + a2

S3 = a1 + a2 + a3
..
.

Sn = a1 + a2 + a3 + · · · + an

S1 is called the fist partial sum, S2 is called the second partial sum, and in general Sn is called
the nth partial sum. The sequence S1 , S2 , . . . , Sn , . . . is called the sequence of partial sums.

Example 4.6. Find the first four partial sums and the nth partial sum of the following sequences:
1
a) an = 2n

1 1
b) bn = n − n+1

Solution 4.5. a) The terms of the sequence are:

1 1 1 1
, , , , ...
2 4 8 16

The first four partial sums are:

1
S1 = a1 =
2
1 1 3
S2 = a1 + a2 = + =
2 4 4
1 1 1 7
S3 = a1 + a2 + a3 = + + =
2 4 8 8
1 1 1 1 15
S4 = a1 + a2 + a3 + a4 = + + + =
2 4 8 16 16

In each partial sum the denominator is a power of 2 and the numerator is 1 less than the
denominator. So, in general
57

2n − 1 1
Sn = n
=1− n
2 2

b) The terms of the sequence are:

1 1 1 1
, , , , ...
2 6 12 20

The first four partial sums are:

1
S1 = a1 = 1 −
2
 1 1 1 1
S2 = a1 + a2 = 1 − + − =1−
2 2 3 3
 1 1 1 1 1 1
S3 = a1 + a2 + a3 = 1 − + − + − =1−
2 2 3 3 4 4
 1 1 1 1 1 1 1 1
S4 = a1 + a2 + a3 + a4 = 1 − + − + − + − =1−
2 2 3 3 4 4 5 5

∴ The nth partial sum is


1
Sn = 1 −
n+1

Definition 4.2.2. Infinite series


Given a sequence
a1 , a2 , a3 , . . . , an . . .

We can write the nth partial sum using the sigma notation as

n
X
Sn = a1 + a2 + a3 + · · · + an = ak (4.2)
k=1

n o∞
We wamt to take a look at this limit of a sequence of partial sums, Sn . Thus
n=1

n
X ∞
X
lim Sn = lim ak = ak (4.3)
n→∞ n→∞
k=1 k=1
58

P∞
k=1 ak is called an infinite series.

Example 4.7. Here are some examples of series


P∞ 2
a) n=1 n
P∞ 1
b) n=1 n
P∞  1 n
c) n=1 2
P∞ e2n
d) n=1 n!

Theorem 4.2.3. Properties of series


P∞ P∞
If n=1 an and n=1 bn are both convergent series, then
P∞ P∞ P∞
i.) n=1 an , where c ∈ R is also convergent, and n=1 can =c n=1 an .
P∞ P∞ P∞
ii.) n=1 (an ± bn ) is also convergent, and n=1 an ± n=1 bn .

4.2.1 Convergence/Divergence of Series


n o P∞
If the sequence of partial sums Sn is convergent, then the series n=1 an is convergent. Thus,
P∞
limn→∞ Sn = S. So, n=1 an = S. Conversely, if the sequence of partial sums is divergent, then
the series is divergent.

P∞
Example 4.8. a) For the series n=1 n

n
X n(n + 1) n(n + 1)
Sn = i= and lim =∞
i=1
2 n→∞ 2
n o∞
n(n+1) P∞
Thus 2 is divergent, therefore the series Sn = n=1 n is divergent.
n=1
59

P∞ 1 P∞  1 1

1 P∞  1 1

b) For the series n=2 n2 −1 = n=2 2(n−1) − 2(n+1) = 2 n=2 n−1 − n+1

" #
1  1 1 1 1 1 1 1  1 1  1 1 
Sn = 1− + − + − + − + ··· + − + −
2 3 2 4 3 5 4 6 n−2 n n−1 n+1
" #
1 3 1 1
= − −
2 2 n n+1
3 1 1
= − −
4 2n 2n + 2

So, !
3 1 1 3
lim Sn = lim − − =
n→∞ n→∞ 4 2n 2n + 2 4

n o∞ n o∞ P∞
3 1 1 1
Therefore, Sn = 4 − 2n − 2n+2 n=2 is convergent, thus the series n=2 n2 −1 is convergent.
n=2

Exercise 4.1. Determine whether the following series converge or diverge. If the series converges
determine its partial sum.
P∞ 1
a) n=1 3n−1
P∞ n
b) n=1 (−1)

c)

Theorem 4.2.4. nth term test for convergence of series


Consider the series

X
an
n=1

then

a) The series converges if limn→∞ an = 0.

b) The series diverges if limn→∞ an 6= 0.

P∞ 1 P∞ 1
Example 4.9. For the series n=1 n and n=1 n2 , limn→∞ an = 0, however in general the series
P∞ 1 P∞ 1
n=1 n diverges and n=1 n2 converges.

NB: The nth term test is not sufficient enough to test for convergence.
60

Exercise 4.2. Determine whether the following series converges or diverges.


P∞ 4n2 −n3
a) n=0 10+2n3
P∞ n2 +2n+1
b) n=0 n3 +1

Definition 4.2.5. The series



X
an
n=0

converges absolutely, if the series



X
|an |
n=1
P∞ P∞
is convergent. An absolutely convergent series is convergent. Furthermore, if n=0 an and n=0 |an |
P∞
diverges, we say that the series n=0 an is conditionally convergent.

4.3 SPECIAL SERIES

In this section, we will take a brief look at some of the special series. In general, determining the
value of the series is very difficult. For most of the series we will be studying in this section, we
will not determine their values, however who should be able to determine if their convergent or
divergent.

4.3.1 Geometric Series

A geometric series is any series that can be written in the form


X
arn−1 (4.4)
n=1

This series is derived from a geometric sequence, which a sequence of the form

a, ar, ar2 , ar3 , ... (r 6= 0) (4.5)

To find the formula for the nth partial sum of the geometric sequence Sn , we multiply Sn by r ans
61

subtract it from Sn . Thus

Sn = a + ar + ar2 + ar3 + . . . arn−1

rSn = ar + ar2 + ar3 + ar4 + . . . arn−1 + arn

Sn − rSn = a − arn

Sn (1 − r) = a(1 − rn )

So,
a(1 − rn )
Sn = (4.6)
1−r


There for the nth partial sum of the geometric sequence arn−1

n=1
is given by

For the geometric series (4.4), we have that



∞  a , |r| < 1

X 
1−r if
lim Sn = arn = (4.7)
n→∞ 
n=1 Diverges,

if |r| ≥ 1

Example 4.10. Determine whether the following series converges or diverges. If the series con-
verges find its value.
P∞ −n+2 4n+1
a) n=1 9
P∞ (−4)3n
b) n=0 5n−1
62

Solution 4.6. a)

∞ ∞
9−n+2 4n+1 = 9−(n−2) 4n+1
X X

n=1 n=1

X 4n+1
=
n=1
9n−2

X 42 · 4n−1
=
n=1
9−1 · 9n−1

!n−1
X 4
= (16)(9)
n=1
9

!n−1
X 4
= 144
n=1
9

4
So, this is a geometric series with a = 144 and r = 9 < 1. Since |r| < 1, then the series
P∞ −n+2 4n+1 144 1296
n=1 9 is convergent and its value is 1− 49
= 5 .

b)

∞ ∞ n
X (−4)3n X (−4)3
=
n=0
5n−1 n=1
5−1 · 5n

!n
X −64
= 5
n=1
5

Is a geometric series with a = 5 and r = − 64


5 . Since |r| =
64
5 > 1, then the series diverges.

4.3.2 Telescoping Series

Here we are looking at another special type of series called Telescoping series. The names comes
from what happens with the partial sums. We will show this with an example.

Example 4.11. Determine if the following series converges or diverges. If it converges find its value.


X 1
n=1
n2 + 4n + 3
63

Solution 4.7. Here we make use of partial fractions.

∞ ∞
X 1 X 1
2
=
n=1
n + 4n + 3 n=1 (n + 1)(n + 3)

" #
X 1 1
= −
n=1
2(n + 1) 2(n + 3)

So,

n
" #
X 1 1
Sn = −
n=1
2(i + 1) 2(i + 3)
" ! ! ! ! !#
1 1 1 1 1 1 1 1 1 1 1
= − + − + − + ··· + − + −
2 2 4 3 5 4 6 n n+2 n+1 n+3
" #
1 5 1 1
= − −
2 6 n+2 n+3

Therefore, " #
1 5 1 1 5
lim Sn = − − =
n→∞ 2 6 n+2 n+3 12

P∞ 1 5
∴ n=1 n2 +4n+3 = 12

4.3.3 Harmonic Series

The harmonic series is a series



X 1
(4.8)
n=1
n

and it is divergent.

Example 4.12. Show that the following series are divergent.


P∞ 5
a) n=1 n
P∞ 1
b) n=4 n

P∞ 5 P∞ 1 P∞ 1 P∞ 5
Solution 4.8. a) n=1 n =5 n=1 n . Since n=1 n is divergent, then n=1 n is also divergent.
64

P∞ P∞ P∞ P 
1 1 1 1 1 ∞ 1 11
b) Since n=1 n =1+ 2 + 3 + n=4 n then n=4 n = n=1 n − 6 which is divergent.

4.4 SERIES WITH NON-NEGATIVE TERMS

In this section we will look at series whose terms are positive. We will consider some other tests
for convergence of series such as the ratio test and the nth root test.

Theorem 4.4.1. The Ratio Test


P∞ an+1
Let n=1 an be a series such that an ≥ 0 and suppose that limn→∞ an = ρ, then

I. The series converges if ρ < 1.

II. The series diverges if ρ > 1.

III. If ρ = 1, the series may converge or diverge. i.e the test is inconclusive.

Note. If ρ = 1, the ratio test cannot be used to determine the converge or divergence of the series.

Example 4.13. Investigate the convergence of the followings series.


P∞ 2n +5
a) n=1 3n
P∞ n!n!
b) n=0 (2n)!
P∞ (2n)!
c) n=0 n!n!

2n +5 2n+1 +5
Solution 4.9. a) an = 3n and an+1 = 3n+1
, so

an+1 2n+1 + 5 2n
lim = lim ×
n→∞ an n→∞ 3n+1 2n + 5
n
2·2 +5 2n
= lim ×
n→∞ 3 · 3n 2n + 5
n
2·2 +5
= lim
n→∞ 2(2n + 5)

2 + 25n 5
= lim , → ∞ as n→∞
n→∞ 3(1 + 5n ) 2 n
2
2
=
3
65

an+1 2 P∞ 2n +5
∴ limn→∞ an = 3 < 1. So, n=1 3n is convergent.

n!n! (n+1)!
b) an = (2n)! and an+1 = (2n+2)! .

Note. Since n! = 1 × 2 × 3 × · · · × n, then (n + 1)! = 1 × 2 × 3 × · · · × n × (n + 1) = n!(n + 1).


Since (2n)! = 1 · 2 · 3 · · · · · n · · · · · (2n − 1) · 2n, then (2n + 2)! = 1 × 2 × 3 × · · · × n × · · · × (2n −
1) × 2n × (2n + 1) × (2n + 2) = (2n)!(2n + 1)(2n + 2).

an+1 (n + 1)!(n + 1)! (2n)!


lim = lim ×
n→∞ an n→∞ (2n + 2)! n!n!
n!(n + 1)n!(n + 1) (2n)!
= lim ×
n→∞ (2n)!(2n + 1)(2n + 2) n!n!
(n + 1) 2
= lim
n→∞ (2n + 1)(2n + 2)

n2 + 2n + 1
= lim
n→∞ 4n2 + 6n + 2
1 + n2 + n12 2 1 6 2
lim , , 2 , 2 → ∞ as n→∞
n→∞ 4 + 6 + 22 n n n n
n n
1
=
4

an+1 1 P∞ n!n!
∴ limn→∞ an = 4 < 1. So, n=0 (2n)! is convergent.

(2n)! (2n+2)!
c) an = n!n! and an+1 = (n+1)!(n+1)! . So,

an+1 (2n + 2)! n!n!


lim = lim ×
n→∞ an n→∞ (n + 1)!(n + 1)! (2n)!
(2n)!(2n + 1)(2n + 2) n!n!
= lim ×
n→∞ n!(n + 1)n!(n + 1) (2n)!
2
4n + 6n + 2
= lim 2
n→∞ n + 2n + 1
4 + n6 + n22
= lim
n→∞ 1 + 2 + 12
n n

=4
66

an+1 P∞ (2n)!
∴ limn→∞ an = 4 > 1, then n=0 n!n! is divergent.

Theorem 4.4.2. The nth Root Test


P∞ √
Let n=1 an be a series such that an ≥ 0 and suppose that limn→∞ n an = ρ, then

I. The series converges if ρ < 1.

II. The series diverges if ρ > 1.

III. If ρ = 1, the series may converge or diverge. i.e the test is inconclusive.

Example 4.14. Investigate the convergence of the followings series.


P∞ n2
a) n=0 2n
P∞ 2n
b) n=0 3n
P∞ 2n
c) n=1 n3

n2
Solution 4.10. a) an = 2n . So

! n1
√ n2
lim n
an = lim
n→∞ n→∞ 2n
1
(n n )2 1
= lim , since n n → 1 as n → ∞
n→∞ 2
1 2
= lim lim n n
n→∞ 2 n→∞
(2 ln n)0
1 2 2
limn→∞ ln n n limn→∞ (n)0 2
= ( lim n = e
n =e = elimn→∞ n = e0 = 1)
2 n→∞

√ 1 P∞ n2
∴ limn→∞ n an = 2 < 1, thus n=0 2n is convergent.
67

2n
b) an = 3n . So,

! n1
√ 2n
lim n
an = lim
n→∞ n→∞ 3n
2
=
5

√ 2 P∞ 2n
∴ limn→∞ n an = 3 < 1, thus n=1 n3 is convergent.

2n
c) an = n3
. So,

! n1
√ 2n
lim n
an = lim
n→∞ n→∞ n3
2
= lim 3
n→∞
3n
1
= 2 lim 3
n→∞
nn
3
= 2 since n n → 1 as n→∞

√ P∞ 2n
∴ limn→∞ n an = 2 > 1, thus n=1 n3 is divergent.

Exercise 4.3. Investigate the convergence of the following series


P∞ n2
a) n=0 2n3 +n2 +1
P∞ n
b) n=0 n!e
P∞ 3n
c) n=1 n3 2n
P∞ n!
d) n=1 10n
P∞ (−2)n
e) n=1 5n
68

CHAPTER 5

FUNCTIONS

In this chapter we will explore the idea of functions and give the mathematical definition of func-
tions. Furthermore, we will explore different types of functions and study the notion of limits of
functions and continuity.

5.1 INTRODUCTION

In almost every physical phenomenon we observe that one quantity depends on another. For
example, your height depends on your age, the temperature depends on the date, the cost of
mailing the package with Nampost Currier depends on its weight. We use a function to describe
the dependence of one quantity on another. Thus,

• Height is a function of age.

• temperature is a function of date.

• The cost of mailing a package is a function of weight.

We can think of many other functions. Here are some more examples:

 The area of a circle is a function of its radius.

 The number of Coronavirus in culture is a function of time.

 The price a commodity (such as bread, sugar, rice, etc.) is a function of its demand.

Definition 5.1.1. Definition of a function


A function from a set A to a set B is a rule that assign to each element x in set A exactly one
element y = f (x) as shown in Figure 5.1.

Figure 5.1: Definition of a function


69

y = f (x) is called the image of x under f while x is called the pre-image of x under f . The sets
A and B are called Domain and Codomain of the function f respectively; often denoted by Df
and Cf . The set of all possible values of f as x varies throughout the domain is called the range
of the function f , often denoted by Rf and it is given by

Rf = {f (x|x ∈ A)} (5.1)

Since y = f (x), the x is the independent variable and y is the dependent variable.

Example 5.1. Consider the function given by

A(r) = πr2

Solution 5.1. The above function gives the area of a circle as a function of its radius. The area of
a circle of radius 2 is A(2) = 4π.
70

5.2 TYPES OF FUNCTIONS

In this section we will explore different types of functions.

Definition 5.2.1. Real-valued functions


Real-valued functions are functions whose domains and ranges are sets of real numbers.

Types of intervals

Here are different types of interval used to describe ranges and domains of functions:

a) (a, b) = {x|a < x < b} Open interval

b) [a, b] = {x|a ≤ x ≤ b} Closed interval

c) [a, b) = {x|a ≤ x < b} Left-Closed, Right-Open interval

d) (a, b] = {x|a < x ≤ b} Left-Open, Right-Closed interval

e) (a, +∞) = {x|x > a} Left-bounded and Right-unbounded (Left-Open) interval

f) [a, +∞) = {x|x ≥ a} Left-bounded and Right-unbounded (Left-Closed) interval

g) (−∞, b) = {x|x < b} Left-unbounded and Right-bounded (Right-Open) interval

h) (−∞, b] = {x|x < b} Left-unbounded and Right-bounded (Right-Closed) interval

i) (−∞, +∞) = R Unbounded at both ends

Example 5.2. Find the ranges and domains of the following functions

a) f (x) = x2

b) g(x) = 1 − x2
1
c) h(x) = x

Solution 5.2. (a) Since f is defined for all real values of x, then Df = R. The output of f is
71

any real number y such that y ≥ 0, then Rf = [0, +∞).

(b) Since g is only defined for real values of x such that −1 ≤ x ≤ 1, then Dg = [−1, 1]. The
output of g is any real number y such that −1 ≤ y ≤ 1, then Rg = [−1, 1].

(c) Since h is defined for all real values of x such that x 6= 0, then Dh = R/{0}. The output
is any real number y such that y 6= 0, then Rh = R/{0}.

Definition 5.2.2. Even and Odd functions


A function y = f (x) is said to be even if f (−x) = f (x) for all x ∈ Df , and it is said to be odd if
f (−x) = −f (x) for all x ∈ Df .

Example 5.3. Determine whether the following functions are even, odd or neither.

a) f (x) = x2

b) g(x) = 1 − x2

c) h(x) = x3

d) i(x) = x3 + x2 − 1

Solution 5.3. a) f (x) = x2 is even since f (−x) = x2 = f (x).


√ p √
b) g(x) = 1 − x2 is even since g(−x) = 1 − (−x2 ) = 1 − x2 = g(x).

c) h(x) = x3 is odd since f (−x) = (−x)3 = −x3 = −f (x).

d) i(x) = x3 + x2 − 1 is neither odd nor even since i(−x) = −x3 + x2 − 1 6= i(x) 6= −i(x).

Definition 5.2.3. Composition of functions (Composite functions)


Let f : A → B and g : B → C, then the composition of f and g is a function h : A → B defined by

h(x) = (f ◦ g)(x) = f (g(x)) for all x in A (5.2)

For example, the function h(x) = (x − 7)3 is the composition of the functions f (x) = x3 and
72

g(x) = x − 7. Thus h = f (g(x)) = f (x − 7) = (x − 7)3 .

Exercise 5.1. Find f (g(x)) and g(f (x)) where f (x) = x2 and g(x) = x + 1. Hence find f (g(2))
and g(f (2)). What does that tells you about f (g(x)) and g(f (x))?

Definition 5.2.4. Piece-wise defined functions


A piece-wise defined function is a function which is defined by two or more equations, where each
equation is valid for some interval.

Example 5.4. Sketch the graphs of the following piece-wise defined functions.

−x,




 for x<0


a) f (x) = 2
x , for 0 ≤ x ≤ 1





1, for x > 1






 x + 2, for x < 0


b) g(x) = 2, for 0 ≤ x ≤ 1





−x + 3, for x > 1

Solution 5.4. a) f (x) is given by the equation y = −x on the interval (−∞, 0), y = x2 on the
interval [0, 1] and y = 1 on the interval (1, +∞). The graph of f (x) is shown on Figure 5.2
below.

Figure 5.2: Graph of f (x)


73

b) g(x) is given by the equation y = x + 2 on the interval (−∞, 0), y = 2 on the interval [0, 1] and
y = −x + 3 on the interval (1, +∞). The graph of g(x) is shown on Figure 5.3 below.

Figure 5.3: Graph of g(x)

Definition 5.2.5. Absolute Values


The absolute value function y = |x| is defined by


x≥0

x,

|x| = (5.3)

−x,

x<0

Definition 5.2.6. Equations of circles


(x − h)2 + (y − k)2 = r or (x − h)2 + (y − k)2 = r2 is called the equation of the
p
The equation
circle centered at (h, k) and having a radius r as shown in Figure 5.4 below.

Figure 5.4: Graph of a circle (x − h)2 + (y − k)2 = r2


74

Example 5.5. Find the center and radius of the following circles.

a) (x − 1)2 + (y + 5)2 = 3

b) x2 + y 2 + 4x − 6y − 3 = 0

c) x2 + y 2 = a2


Solution 5.5. a) Is a circle of radius r = 3, centered at c(1), −5.

b) We must first convert the given equation into the general form of the equation of the circle.

x2 + y 2 + 4x − 6y − 3 = 0 ⇐⇒ x2 + 4x + y 2 − 6y = 3

⇐⇒ (x + 2)2 − 22 + (y − 3)2 − 32 = 3

⇐⇒ (x + 2)2 + (y − 3)2 = 3 + 4 + 9

⇐⇒ (x + 2)2 + (y − 3)2 = 16 = 42

∴ This is an equation of a circle centered at c(−2, 3), with radius r = 4.

c) This is an equation of the circle centered at c(0, 0), with radius r = a.


75

5.3 TRIGONOMETRIC FUNCTIONS

To find the terminal point P (x, y) for a given real number θ, we move a distance θ along the circle
of radius r, starting at the point P0 (r, 0). We move in a counterclockwise direction if θ > 0 and in
a clockwise direction if θ < 0 as shown on Figure 5.5.

Figure 5.5: Radian measure of angle θ

We use the x and y coordinates of the point P (x, y) to determine several functions. By making use
of the right angle triangle 4OP P0 to determine the functions; sine (sin), cosine (cos), tangent
(tan), cosecant(csc), secant (sec) and cotangent (cot).

Definition 5.3.1. Trigonometric Functions


Let θ be any real number and let P (x, y) be the terminal point on the circle of radius r determined
by θ. We define

y x y
sin θ = , cos θ = , tan θ = (x 6= 0)
r r x
r r x
csc θ = , sec θ = , cot θ = (y 6= 0)
y x y

The above functions are called the basic trigonometric functions.

Below are the graphs of the three main basic trigonometric functions, sin, cos and tan.

(1.) y = sin θ
76

Figure 5.6: Graph of y = sin θ

• Domain = R

• Range = [−1, 1]
1
• sin(−θ) = − sin(θ) =⇒ y = sin θ is odd. So, csc θ = sin θ is also odd.

• P eriod = 2π

(2.) y = cos θ

Figure 5.7: Graph of y = cos θ

• Domain = R

• Range = [−1, 1]
1
• cos(−θ) = cos(θ) =⇒ y = cos θ is even. So, sec θ = cos θ is also even.

• P eriod = 2π

(3.) y = tan θ
77

Figure 5.8: Graph of y = tan θ

2n+1
• Domain = R|{±x|x = 2 ,n = 0, 1, . . . }

• Range = R
1
• tan(−θ) = − tan(θ) =⇒ y = tan θ is odd. So, tan θ = tan θ is also odd.

• P eriod = π

5.3.1 Special angles and special values of their trigonometric functions

Table 5.1 below shows some special angles and the values of their trigonometric functions.

θ sin θ sin θ tan θ csc θ sec θ cot θ


0 0 1 0 NA 1 NA
π 1

3

√1 2 √2 3
6 2 2 3 √ √3
π √1 √1
4 1 2 2 1
√2 2

π 3 1 √2 √1
3 2 2 3 3
2 3
π
2 1 0 NA 1 NA 0

Table 5.1: Special angles and values of their trigonometric function.

Table 5.1 can also be obtained from the following special triangles.
78

Figure 5.9: Special Triangles

5.3.2 Signs of trigonometric functions

The signs of a trigonometric function depends on the quadrant in which the terminal point lies as
shown on Figure 5.10 and Table 5.2 below.

Figure 5.10: Cast Diagram

Quadrant Positive Functions Negative Functions


I ALL NONE
II sin, csc cos, sec, tan, cot
III tan, cot sin, csc, cos, sec
IV cos, sec sin, csc, tan, cot

Table 5.2: Quadrants and the signs of the trigonometric functions.

NB: θR is called the reference angle and 0 < θR < π2 .


79

Example 5.6. Evaluate the following trigonometric functions.


 

a) cos 3
 
π
b) tan − 3
 
19π
c) sin 4
 
2π 2π π 2π 2π
Solution 5.6. a) θ = 3 and θR = π − θ = π − 3 = 3. and 3 is in Quadrant II, cos 3 < 0.
Thus,
 2π  π  1
cos(θ) = − cos(θR ) ⇐⇒ cos = − cos =−
3 3 2
 
b) θ = − π3 and θR = π
3 is in the Quadrant IV and tan − π
3 < 0. Thus,

 π π  √
tan θ = − tan θR ⇐⇒ tan − = − tan =− 3
3 3

19π 3π 19π 3π
c) Since 4 −4π = 4 , then the terminal point determined 4 and 4 are the same. θR = π − 2π
4 =
 
π 3π 3π
4 and the terminal point of 4 is in Quadrant II and sin 4 > 0. Thus,

 3π  π  1
sin(θ) = sin(θR ) ⇐⇒ sin = sin =√
4 4 2

5.3.3 Trigonometric Identities and Formulas

Trigonometric functions are related to each other through equations called trigonometric iden-
tities or formulas.

(1.) Pythagorean Identities

I. sin2 θ + cos2 θ = 1

II. tan2 θ + 1 = sec2 θ

III. 1 + cot2 θ = csc2 θ

(2.) Sum and Difference Formulas

I. sin(α + β) = sin α cos β + cos α sin β

II. sin(α − β) = sin α cos β − cos α sin β


80

III. cos(α + β) = cos α cos β − sin α sin β

IV. cos(α − β) = cos α cos β + sin α sin β


tan α+tan β
V. tan(α + β) = 1−tan α tan β

tan α−tan β
VI. tan(α − β) = 1+tan α tan β

(3.) Double Angle Formulas

I. sin(2θ) = 2 sin θ cos θ

II. cos(2θ) = cos2 θ − sin2 θ = 2 cos2 θ − 1 = 1 − 2 sin2 θ


2 tan θ
III. tan 2θ = 1−tan2 θ

(4.) Half Angle Formulas


q
I. sin 2θ = ± 1−cos θ
2
q
II. cos 2θ = ± 1+cos θ
2
q
III. tan 2θ = ± 1−cos θ
1+cos θ

Example 5.7. For the following questions

1. Prove that

a) sec2 θ = 1 + tan2 θ

b) csc2 θ = 1 + cot2 θ

2. Show that

a) cos(x − π2 ) = sin x

b) sin(x + π2 ) = cos x

3. Evaluate

a) sin 75◦

b) sin 7π
12
81

Solution 5.7. 1. a) From sin2 θ + cos2 θ = 1 we have that

sin2 θ cos2 θ 1
2 + 2 = =⇒ 1 + cot2 θ = csc2 θ
sin θ sin θ sin2 θ

∴ csc2 θ = 1 + cot2 θ

b) Similarly, from sin2 θ + cos2 θ = 1 we have that

sin2 θ cos2 θ 1
2
+ 2
= =⇒ tan2 θ + 1 = sec2 θ
cos θ cos θ cos2 θ

∴ sec2 θ = 1 + tan2 θ

2. a) By making use of the sum and difference formulas we have that

π π π
cos(x − ) = cos x cos + sin x sin
2 2 2
= cos x · 0 + sin x · 1

= sin x

b) Similarly, by making use of the sum and difference formulas we have that

π π π
sin(x + ) = sin x cos + cos x sin
2 2 2
= sin x · 0 + cos x · 1

= cos x

3. By making use of the sum formulas, we get

a) sin 75◦ = sin(45◦ + 30◦ )


82

sin(45◦ + 30◦ ) = sin 45◦ cos 30◦ + sin 30◦ cos 45◦

1 3 1 1
=√ · + ·√
2 2 2 2

1+ 3
= √
2 2

 
π π
b) sin 7π
12 = sin 4 + 3

π π π π π π
sin + = sin cos + sin cos
4 3 4 3 √3 4
1 1 1 3
=√ · +√ ·
2 2 2 2

1+ 3
= √
2 2

Exercise 5.2. Solve the following questions

1. Use the double angle formulas to find


π
a) cos2 8

π
b) sin2 8

tan A+tan B
2. show that tan(A + B) = 1−tan A tan B
83

5.4 EXPONENTIAL AND LOGARITHMIC FUNCTIONS

Exponential and logarithmic functions are important both in theory and practice. These functions
have numerous applications in engineering and other field of applied sciences. e.g. in the modeling
of population growth etc. In this section we will look at some graphs of exponential and logarithmic
functions and establish the relationship between the two types of functions.

(1.) EXPONENTIAL FUNCTIONS

Definition 5.4.1. Consider a function of the form

f (x) = ax , a>0 (5.4)

such a function is called an exponential function.

Lets consider three cases for the base a as follow:

(1.) If a = 1, then f (x) = 1x = 1. So, this just gives us a constant function f (x) = 1.

(2.) If 0 < a < 1, then the graph of f (x) = ax has the following properties:

I. f (x) does not intersect the x−axis however, it intersect the y−axis at y = f (0) = 1.

II. f (x) > 0 for all x ∈ R. Thus, Df = R and Rf = (0. + ∞).

III. As x → +∞, f (x) = ax → 0 and as x → −∞, f (x) = ax → +∞.

 x
1
Figure 5.11: Graph of f (x) = 2
84

 x
For example, let a = 12 , the graph of f (x) = 1
2 is shown on Figure 5.11 above.

(3.) If a > 1, then the graph of f (x) = ax has the following properties:

I. f (x) does not intersect the x−axis however, it intersect the y−axis at y = f (0) = 1.

II. f (x) > 0 for all x ∈ R. Since a > 1 then ax > 0 for all x ∈ Df . Thus, Df = R and
Rf = (0. + ∞).

III. As x → +∞, f (x) = ax → +∞ and as x → −∞, f (x) = ax → 0.

 x
1
Figure 5.12: Graph of f (x) = 2

For example, let a = 2, the graph of f (x) = 2x is shown on Figure 5.12 above.

 x
In general, f (x) = 1
a = a−x is a reflection of g(x) = ax on the y−axis.

Figure 5.13: Graph of f (x) = ex and g(x) = e−x


85

A particular example of an exponential function is when a = e = 2.718 . . . . The function


1
f (x) = ex is called the natural exponential function. Since e > 1 and e < 1, we can sketch
 x
the graphs of f (x) = ex and g(x) = 1
e = e−x on the same axis as show on Figure 5.13 above.

(2.) LOGARITHMIC FUNCTIONS

Definition 5.4.2. A logarithmic function is function of the form

f (x) = loga x (5.5)

We consider the following cases for the base a

(1.) If 0 < a < 1, then the graph of f (x) = loga x has the following properties:

I. f (x) does not intersect the y−axis however, it intersect the x−axis at x = 1. Thus,
f (x) = loga x = 0 =⇒ x = a0 = 1.

II. f (x) = loga x is only defined for x > 0. Thus Df = (0, +∞) and Rf = R.

For example, let a = 21 , the graph of f (x) = log 1 x is shown on Figure 5.14 below.
2

Figure 5.14: Graph of f (x) = log 1 x


2

(2.) If a > 1, then the graph of f (x) = loga x has the following properties:
86

I. f (x) does not intersect the y−axis however, it intersect the x−axis at x = 1. Thus,
f (x) = loga x = 0 =⇒ x = a0 = 1.

II. f (x) = loga x is only defined for x > 0. Thus Df = (0, +∞) and Rf = R.

For example, let a = 2, the graph of f (x) = log2 x is shown in Figure 5.15 below.

Figure 5.15: Graph of f (x) = log2 x

(3.) THE RELATIONSHIP BETWEEN EXPONENTIAL AND LOGARITH-


MIC FUNCTIONS

Let’s investigate the relationship between the exponential function f (x) = ex and logarithmic
functions g(x) = loge x = ln x by looking at their graphs a shown on Figure 5.16 below.

Figure 5.16: Graph of f (x) = ex and g(x) = loge x = ln x


87

 x
1
Similarly, for the exponential function f (x) = 2 and the logarithmic function g(x) = log 1 x
2

their graphs are shown on Figure 5.17 below.

 x
1
Figure 5.17: Graph of f (x) = 2 and g(x) = log 1 x
2

From Figures 5.16 and 5.17 we observe that the logarithmic g(x) = loga x is a reflection of the
exponential function f (x) = ax on the line y = x. ∴ The logarithmic function g(x) = loge x
is the inverse of the exponential function f (x) = ex i.e. g(x) = loge x = f −1 (x). Similarly
 x
g(x) = log 1 x is the inverse of f (x) = 1
2 i.e. g(x) = log 1 x = f −1 (x).
2 2

Exercise 5.3. Sketch the graphs of the following functions of the same axis

(a) f (x) = 3x
 x
1
(b) g(x) = 3

(c) h(x) = log3 x

(d) i(x) = log 1 x


3
88

5.5 HYPERBOLIC FUNCTIONS

In this section we will explorer the notion of hyperbolic functions and their algebraic relation to
trigonometric and exponential functions.

Definition 5.5.1. Hyperbolic functions, sinh x, cosh x, tanh x, etc. are certain even and odd com-
binations of exponential functions ex and e−x .

The six main hyperbolic functions are given below:

ex − e−x ex + e−x sinh x ex − e−x


sinh x = , cosh x = , tanh x = = x
2 2 cosh x e + e−x
1 2 1 2
csch x = = x , sech x = = x
sinh x e − e−x cosh x e + e−x
1 cosh x x
e +e −x
coth x = = = x
tanh x sinh x e − e−x

The notation of hyperbolic functions implies a close relation to trigonometric , sin x, cos x, tan x,
etc. However the relationship is algebraic rather than geometric. Figures 5.18, 5.19 and 5.20 shows
the graphs of sinh x, cosh x and tanh x.

Figure 5.18: sinh x Figure 5.19: cosh x Figure 5.20: tanh

The graphs of hyperbolic sine and cosine functions can be sketched using graphical addition as
shown in Figures 5.18 and 5.19.
89

Hyperbolic Identities

Similar trigonometric functions, hyperbolic functions satisfy a number of identities as shown below:

(1.) sinh(−x) = − sinh(x) sinh x is odd

(2.) cosh(−x) = cosh(x) cosh x is even

(3.) tanh(−x) = − tanh(x) tanh x is odd

(4.) cosh2 x − sinhx = 1

(5.) 1 − tanh2 x = sech2 x

(6.) coth2 x − 1 = csch2 x

(7.) sinh(x ± y) = sinh x cosh y ± sinh y cosh x

(8.) cosh(x ± y) = cosh x cosh y ± sinh x sinh y


tanh x ± tanh y
(9.) tanh(x ± y) =
1 ± tanh x tanh y

Some Applications of Hyperbolic Functions

Hyperbolic functions have numerous applications in science and engineering, here are some of the
most famous applications of cosine and tangent hyperbolic functions.

It can be proved that If a heavy flexible cable (such as a telephone or power line) is suspended
between two points at the same height, then it takes the shape of a curve with equation y =
 
x
c + a cosh a called a catenary (the Latin word “catena” means “chain.”) as shown on Figure 5.21
.

Figure 5.21: A catenary y = c + a cosh x Figure 5.22: Idealized ocean wave


90

Another application of hyperbolic functions occurs in the description of ocean waves. The velocity
of a water wave with length L moving across a body of water with depth d is modeled by the
function

v !
u
u gL 2πd
v=t tanh
2π L

where g is the acceleration due to gravity (see Figure 5.22).

Example 5.8. Solve the following exercises

(1.) Find the numerical value of each expression

i) sinh(ln 2)

ii) sech(0)

iii) sinh(1)

(2.) Prove that

i) sinh(−x) = − sinh(x)

ii) cosh x + sinh x = ex


tanh x+tanh y
(3.) Prove that tanh(x + y) = 1+tanh x tanh y

eln 2 −e− ln 2 2− 12 3
Solution 5.8. (1.) i) sinh(ln 2) = 2 = 2 = 4

1 2
ii) sech (0) = cosh(0) = e0 +e−0
=1
e1 −e−1 e2 −1
iii) sinh(1) = 2 = 2e

e−x −ex x −e−x


(2.) i) sinh(−x) = 2 = −e 2 = − sinh x
ex +e−x ex −e−x 2ex
ii) cosh x + sinh x = 2 + 2 = 2 = ex
91

sinh(x+y)
(3.) tanh(x + y) = cosh(x+y)

sinh(x + y) sinh x cosh y + cosh x sinh y


=
cosh(x + y) cosh x cosh y + sinh x sinh y
sinh x cosh y sinh y cosh x
cosh x cosh y + cosh x cosh y
= sinh x cosh y sinh x sinh y
cosh x cosh y + cosh x cosh y
tanh x + tanh y
=
1 + tanh x tanh y

tanh x+tanh y
∴ tanh(x + y) = 1+tanh x tanh y .

Exercise 5.4. Show that


x2 −1
a) tanh(ln x) = x2 +1

b) (cosh x + sinh x)n = cosh nx + sinh nx


92

5.6 INVERSES OF TRIGONOMETRIC AND HYPERBOLIC FUNCTIONS

In this section we will study the inverses of trigonometric and hyperbolic functions

5.6.1 INVERSE TRIGONOMETRIC FUNCTIONS

5.6.2 INVERSE HYPERBOLIC FUNCTIONS

We can see from Figures 5.18 and 5.20 that sinh and tanh are one-to-one (injective) functions and
so they are invertible and their inverse functions are denoted by sinh−1 and tanh−1 . Figure 5.19
shows that cosh is not one-to-one, but when it is restricted to a domain [0, ∞) it becomes one-to-
one. The inverse hyperbolic cosine function is defined as the inverse of this restricted function.

y = sinh−1 x ⇐⇒ sinh y = x

y = cosh−1 x ⇐⇒ cosh y = x and y≥0

y = tanh−1 x ⇐⇒ tanh y = x

Recall the function


ex − e−x
y = sinh x =
2

To find its inverse, we have to make x a subject of the formula and swap x and y. So,

ex − e−x
y= =⇒ ex − e−x = 2y
2
1
=⇒ ex − x = 2y
e
=⇒ e2x − 1 = 2yex

Thus, we have (ex )2 − 2yex − 1 = 0 which is quadratic in ex . Therefore

p
x 2y ± 4y 2 + 4 q
e = = y ± y2 + 1
2
93

p p
We get ex = y + y 2 + 1 or ex = y − y 2 + 1 (not valid since ex > 0). So,

q q
ex = y + y 2 + 1 =⇒ x = ln(y + y 2 + 1)

.

∴ By swapping x and y we get sinh−1 x = x = ln(x + x2 + 1)

cosh−1 x and tanh−1 x as well as their reciprocals can be derived in the similar manner and their
shown in the table below:

√ !
−1
p
−1 x2 + 1
1+
(1.) sinh x = ln(x + x2 + 1), x∈R (4.) csch x = ln , x ∈ R/{0}
x
√ !
p 1 + 1 − x2
(2.) cosh−1 x = ln(x + x2 − 1), x≥1 −1
(5.) sech x = ln , x≥0
x
! !
1 1+x 1 x+1
(3.) tanh−1 x = ln , −1 < x < 1 (6.) coth−1 x = ln , −1 < x < 1
2 1−x 2 x−1

The graphs of sinh−1 x, cos−1 x and tanh−1 x are show below.

Figure 5.23: sinh−1 x Figure 5.24: cosh−1 x Figure 5.25: tanh−1


domain= R, range= R domain= [1, ∞], range= [0, ∞] domain= (−1, 1), range= R

Exercise 5.5. Prove the formula for the above functions


94

5.7 RATIONAL FUNCTIONS

In this section we are going to study rational functions and explore the notion of partial fractions
decomposition.

Definition 5.7.1. A rational function is a function of the form

P (x)
r(x) = (5.6)
Q(x)

where P and Q are polynomials. We assume that P and Q have no factor in common. Partial
functions are also know as partial fraction.

Definition 5.7.2. Partial Fractions Decomposition


The process of taking a rational expression of the form (5.6) and decompose it into simpler rational
expressions that can be added or subtracted to get the original rational expression is called partial
fraction decomposition.
1 1 3·1+2·1 2(x−1)+3(x+1)
For example: 2 + 3 = 6 = 56 , 2
x+1 + 3
x−1 = (x+1)(x−1) = 5x+1
(x+1)(x−1) .

5.7.1 rules for partial fraction decomposition

There are five main rules used in partial fractions decomposition.

(1.) Denominator with Non-repeated Linear Factors


In this case, we are going to look at when the denominator has non-repeated linear factors;
meaning that when we factor the denominator, the factors will be linear and non of them is
A
repeated. For each linear factor of the form x − a of Q(x), include a fraction of the form x−a ,

A is a non-zero constant.

Example 5.9. Find the partial fraction decomposition for the following rational fractions
7x−4
a) x2 −2x−8

5x+1
b) (x−1)(x+1)
95

8
c) x2 +11x+28

Solution 5.9. a) The denominator is Q(x) = x2 − 2x − 8 = (x − 4)(x + 2). Thus

7x − 4 7x − 4
=
x2 − 2x − 8 (x − 4)(x + 2)
A B
= +
x−4 x+2

So,

7x − 4 A(x + 1) + B(x − 4)
=
x2 − 2x − 8 (x − 4)(x + 2)
(A + B)x + 2A − 4B
=
(x − 4)(x + 2)

Thus,

4(A + B = 7)

2A − 4B = −4

6A = 24

24
A=
6

A = 4 and B = 7 − 4 = 3.

7x−4 4 3
∴ x2 −2x−8
= x−4 + x+2 .
96

b) The denominator is Q(x) = (x − 1)(x + 1). Thus

5x + 1 A B
= +
(x − 1)(x + 1) x−1 x+1
A(x + 1) + B(x − 1)
=
(x − 1)(x + 1)
(A + B)x + A − B
=
(x − 1)(x + 1)

Thus

A+B =5

A−B =1

2A = 6

6
A=
2

A = 3 and B = 5 − 3 = 2.

5x+1 3 2
(x−1)(x+1) = x−1 + x+1 .

c) The denominator is Q(x) = x2 + 11x + 28 = (x + 4)(x + 7). Thus

8 8
=
x2 + 11x + 28 (x + 4)(x + 7)
A B
= +
x+4 x+7
A(x + 7) + B(x + 4)
=
(x + 4)(x + 7)
(A + B)x + 7A + 4B
=
(x + 4)(x + 7)

Thus,
97

−4(A + B = 0)

7A + 4B = 8

3A = 8

8 8
A= and B=−
3 3

8 8 8
∴ x2 +11x+28
= 3(x+4) − 3(x+7)

(2.) Denominator with repeated linear factors


In this case, we consider the denominator Q(x) to contain a repeated factor of the form (x−a)n .
The corresponding partial fractions decomposition is:

A1 A2 An
+ + ··· + (5.7)
x − a (x − a)2 (x − a)n

Example 5.10. a) Find the partial fractions decompositions of

7x2 + 5x + 7
(x − 2)(x2 + 2x + 1)

b) Set up the partial fractions decomposition for the following rational expressions
1
(i.) (x+1)2 (x−1)

5x+8
(ii.) (x−1)2 (x+3)2

Solution 5.10. a) Q(x) = (x − 2)(x2 + 2x + 1) = (x − 2)(x + 1)2 , x − 2, x + 1 are both linear


98

where (x + 1)2 implies x + 1 is repeated twice. Thus

7x2 + 5x + 7 7x2 + 5x + 7
=
(x − 2)(x2 + 2x + 1) (x − 2)(x + 1)2
A B C
= + +
x − 2 x + 1 (x + 1)2
A(x + 1)2 + B(x − 2)(x + 1) + C(x − 2)
=
(x − 2)(x + 1)2
A(x2 + 2x + 1) + B(x2 − x − 2) + C(x − 2)
=
(x − 2)(x + 1)2
(A + B)x2 + (2A − B + C)x + A − 2B − 2C
=
(x − 2)(x + 1)2

Thus,

A+B =7

2A − B + C = 5

A − 2B − 2C = 7

        
1 1 0 7 7 A  5 
        
5 = 5. So, AX = B, thus X = B  =  2 . ∴ A = 5, B = 2 and
2 −1 1        

        
1 −2 −2 7 7 C −3
| {z }|{z} |{z}
A X B
C = −3.

7x2 +5x+7 5 2 3
∴ (x−2)(x2 +2x+1)
= x−2 + x+1 − (x+1)2
.

b) (i.) Q(x) = (x + 1)2 (x − 1), x − 1, x + 1 are both linear where (x + 1)2 implies x + 1 is
repeated twice. Thus

1 A B C
2
= + 2
+
(x + 1) (x − 1) (x + 1) (x + 1) x−1
99

(ii.) Q(x) = (x − 1)2 (x + 3)2 , x − 1, x + 3 are both linear where (x − 1)2 implies x − 1 is
repeated twice and (x + 3)2 implies x + 3 is repeated twice. Thus

5x + 8 A B C D
= + + +
(x − 1)2 (x + 3)2 x − 1 (x − 1)2 x + 3 (x + 3)2

(3.) Denominator with irreducible quadratic factors


We consider the denominator Q(x) with a quadratic factor ax2 +bx+c such that O = b2 −4ac <
0. For each irreducible quadratic factor ax2 + bx + c of Q(x), include a fraction of the form

Ax + B
(5.8)
ax2 + bx + c

Example 5.11. Find the partial fractions decomposition of

2x − 5
(x + 1)(x2 + 4)

Solution 5.11. Q(x) = (x + 1)(x2 + 4), where x + 1 is linear and (x2 + 4) is irreducible since
O < 0. Thus

2x − 5 A Bx + C
2
= + 2
(x + 1)(x + 4) x+1 x +4
A(x2 + 4) + (Bx + C)(x + 1)
=
(x + 1)(x2 + 4)
Ax2 + 4A + Bx2 + (B + C)x + C
=
(x + 1)(x2 + 4)
(A + B)x2 + (B + C)x + 4A + C
=
(x + 1)(x2 + 4)
100

A + B = 0 =⇒ A = −B

B + C = 2 =⇒ −A + C = 2

4A + C = −5

−A + C = 2

4A + C = −5

−5A = 7

Thus A = − 75 , B = 7
5 and C = 2 − 7
5 = 35 .

2x−5 7 7x+3
∴ (x+1)(x2 +4)
= − 5(x+1) + 5(x 2 +4)

(4.) Denominator with repeated irreducible quadratic factors


We consider the denominator Q(x) to contain repeated irreducible quadratic factors of the
form (ax2 + bx + c)n . The corresponding partial fraction decomposition is

A1 x + B1 A2 x + B 2 A3 x + B3 An x + Bn
+ + + ··· + (5.9)
2 2
ax + bx + c (ax + bx + c) 2 2
(ax + bx + c) 3 (ax2 + bx + c)n

Example 5.12. Set up the form of partial fraction decomposition for the following rational
fractions
4x2 +2x−7
a) (x2 +3)2

3x2 −7x+9
b) (x2 +7)3
101

11x−6
c) (x2 −5x+10)2

Solution 5.12. a) Q(x) = (x2 + 3)2 where x2 + 3 is irreducible and repeated twice. Thus

4x2 + 2x − 7 A1 x + B1 A2 x + B2
2 2
= + 2
(x + 3) x2 + 3 (x + 3)2

b) Q(x) = (x2 + 7)3 where x2 + 7 is irreducible and repeated three times. Thus

3x2 − 7x + 9 A1 x + B1 A2 x + B2 A3 x + B3
2 3
= + 2 + 2
(x + 7) x2 + 7 (x + 7)2 (x + 7)3

c) Q(x) = (x2 − 5x + 10)2 where x2 − 5x + 10 is irreducible and repeated twice. Thus

11x − 6 A1 x + B 1 A2 x + B2
= 2 +
(x2 − 5x + 10)2 x − 5x + 10 (x2 − 5x + 10)2

(5.) When the degree of P (x) is greater than the degree of Q(x)

When the degree of the numerator P (x) is greater than the degree of the denominator, we first
apply long division to obtain the expression of the form

R(x)
f (x) = f1 (x) + (5.10)
Q(x)

where f1 (x) is the quotient and R(x) is the remainder. In Equation (5.10), the proper fraction
R(x)
of the form Q(x) , will then then be decomposed into simpler fraction.

Example 5.13. Find the partial fraction decomposition of the following rational fractions.
4x3 +10x+4
a) x(2x+1)
102

x3 +1
b) x2 +1

2x4 +3x2 +1
c) x2 +3x+2

Solution 5.13. a) P (x) = 4x3 + 10x + 4 is of degree 3 and Q(x) = x(2x + 1) = 2x2 + x of
degree 2. so, this is an improper rational expression. Thus

f1(x) → 2x − 1
Q(x) → 2x2 + x 4x3 + 10x + 4
−(4x3 + 2x2 )

−2x2 + 10x + 4
−(−2x2 − x)

R(x) → 11x + 4

So,
4x3 + 10x + 4 11x + 4
= 2x − 1 +
x(2x + 1) x(2x + 1)

and

11x + 4 A B
= +
x(2x + 1) x 2x + 1
A(2x + 1) + Bx
=
x(2x + 1)

Thus, 2A + B = 11 and A = 4, then B = 11 − 2(4) = 3.

Therefore,
11x + 4 4 3
= +
x(2x + 1) x 2x + 1

4x3 +10x+4 4 3
∴ x(2x+1) = 2x − 1 + x + 2x+1 .

b) P (x) = x3 + 1 is of degree 3 and Q(x) = x2 + 1 of degree 2. so, this is an improper rational


expression. Thus
103

 f1 (x) → x
Q(x) → x2 +1 x3 + 1
−(x3 + x)

R(x) → −x + 1

Thus

x3 + 1 x−1
2
=x− 2
x +1 x +1

c) P (x) = 2x4 + 3x2 + 1 is of degree 4 and Q(x) = x2 + 3x + 2 of degree 2. so, this is an


improper rational expression. Thus

f1 (x)
 → 2x2 − 6x + 17
2
Q(x) → x + 3x + 2 2x4 + 3x2 + 1
−(4x4 + 6x3 + 4x2 )

17x2 + 12x + 1
− (17x2 + 51x + 34)

−6x3 − x2 + 1
− (−6x3 − 18x2 − 12x)

R(x) → −39x − 33

So,

2x4 + 3x2 + 1 39x + 33


= 2x2 − 6x + 17 − 2
x2 + 3x + 2 x + 3x + 2
39x + 33
= 2x2 − 6x + 17 −
(x + 1)(x + 2)
104

and,

39x + 33 A B
= +
(x + 1)(x + 2) x+1 x+2
Ax + 2A + Bx + B
=
(x + 1)(x + 2)

A + B = 39

2A + B = 33

A = −6 and B = 39 − (−6) = 45

39x + 33 6 45
=− +
(x + 1)(x + 2) x+1 x+2

2x4 +3x2 +1 6 45
∴ x2 +3x+2
= 2x2 − 6x + 17 + x+1 − x+2

Exercise 5.6. Find the partial fractions decomposition of the following functions
3x+11
a) x2 −x−6

x2 +4
b) 3x3 +4x2 −4x

x2 −29x+5
c) (x−4)2 (x2 +3)

x
d) (x2 −x+1)2 (3x−2)
105

5.8 LIMITS AND CONTINUITY OF FUNCTIONS

In this section we will be exploring the notions of limits and continuity of functions. To introduce
the concept let’s investigate the behavior of the function defined by

x3 − 8
f (x) =
x2 − 4

for values of near 2. This function is not defined at x = 2. The following table gives values of f (x)
for values of x close to 2, but not equal to 2.

x 1.9 1.99 1.999 2 2.001 2.01 2.1


f (x) 2.926 2.993 2.999 * 3.001 3.008 3.076
limx→2 f (x) −→ * ←−

Table 5.3: Guessing limx→2 f (x).

From Table 5.3 we see that when x is close to 2 (on either side of 2), f (x) is close to 3. We express
x3 −8
this by saying “the limit of the function f (x) = x2 −4
as x approaches 2 is equal to 3.” Denoted by

x3 − 8
lim f (x) = lim =3
x→2 x→2 x2 − 4

Definition 5.8.1. Definition of Limit


We write
lim f (x) = L (5.11)
x→a

and say that “the limit of f (x), as x approaches a, equals to L.

We make the the values of f (x) arbitrary closer to L by taking x values that are sufficiently closer
to a on either side but not equals to a. The alternative notation for limx→a f (x) = L is

f (x) → L as x → a

Which read “f (x) approaches L as x approaches a”


106

The phrase “but x 6= a” means that in finding the limit of f (x) as x approaches a , we never
consider x = a . In fact, f (x) do not need to be defined when x = a . We are only interested in
how f (x) is defined near a.

Figure 5.26 shows the graphs of the three functions where in each case, limx→a f (x) = L.

Figure 5.26: limx→a f (x) = L in all three cases

From Figure 5.26 we observe that in (a), f (a) = L, in (b), f (a) 6= L and in (c), f (a) is not defined.
However, in each case it is true that limx→a f (x) = L.

Example 5.14. Guess the values of the following limits


x−1
a) limx→1 x2 −1

 x−1 x 6= 1


x2 −1
, if
b) limx→1 g(x) where g(x) =

2,

if x=1

t2 +9−3
c) limt→0 t2

x−1
Solution 5.14. a) The function f (x) = x2 −1
is not defined when x = 1, we will use the table to
compute values of f (x) for values of x close to 1, but not equal to 1 as shown below.
107

x 0.9 0.99 0.999 0.9999 1 1.0001 1.001 1.01 1.1


f (x) 0.526316 0.502513 0.500250 0.500025 * 0.499975 0.499750 0.497512 0.476190
−→ * ←−

Table 5.4: Guessing limx→1 f (x)

From Table 5.4, we observe that f (x) approaches 0.5 from either side as x approaches 1. So, on
the basis of the values in the table, we can make an educated guess that

x−1
lim = 0.5
x→1 x2 − 1

b) The function g(x) resulted from the function f (x) in (a) by assigning the value 2 when x = 1.
So, the limit of f (x) and g(x) as x approaches 1 are equal as shown on Figures 5.27 and 5.28

Figure 5.27 Figure 5.28


t2 +9−3
c) Table 5.5 below shows the list of values of the function f (t) = t2
for several values of t
near 0.
108

t -0.5 -0.1 -0.05 0.01 0 0.01 0.05 0.1 0.5


f (t) 0.16662 0.16662 0.16666 0.16667 * 0.16667 0.16666 0.16662 0.16662
−→ * ←−

Table 5.5: Guessing limt→0 f (t)

From Table 5.5 as t approaches 0, the values of the function seem to approach 0.1666666. . . and
so we make an educated guess that

t2 + 9 − 3 1
lim =
t→0 t2 6

Definition 5.8.2. One-side limits


A one-sided limit limit can be the left-hand limit which is the limit as f (x) approaches a from
the left written as
lim f (x) = L (5.12)
x→a−

or the right-hand limit which is the limit as f (x) approaches a from the right written as

lim f (x) = L (5.13)


x→a+

The symbol “x → a− ” means that we consider only x < a, while the symbol “x → a+ ” means that
we consider only x > a as illustrated in Figure 5.29 and 5.30 below.

Figure 5.29: limx→a− f (x) = L Figure 5.30: limx→a+ f (x) = L


109

Definition 5.8.3. Existence of Limits


For the function f (x) the limit exist and it is given by

lim f (x) = L
x→a

if and only if
lim f (x) = lim f (x) = L
x→a− x→a+

i.e. The function f (x) has a limit as x approaches a if and only if the right-hand and the left-hand
limits as x approaches a exist and are equal.

Example 5.15. Find limx→1 f (x) for the function



1 − 2x,

 x<1
f (x) =

x − 3,

x≥1

Solution 5.15. The graph of f (x) is shown on Figure 5.31 below.

Figure 5.31: Graph of f (x)

We have that
lim f (x) = lim (1 − 2x) = 1 − 2(1) = −1
x→a− x→a−
110

and
lim f (x) = lim (x − 3) = 1 − 3 = −2
x→a+ x→a−

Since
lim f (x) = −1 6= −2 = lim f (x)
x→a− x→a+

then limx→a f (x) does not exist.

Theorem 5.8.4. Limit Laws


Let f (x) and g(x) be functions such that

lim f (x) = L1 and lim g(x) = L2


x→a x→a

and c be a constant. Then

1. limx→a [f (x) + g(x)] = limx→a f (x) + limx→a g(x) = L1 + L2 SUM LAW

2. limx→a [f (x) − g(x)] = limx→a f (x) − limx→a g(x) = L1 − L2 DIFFERENCE LAW

3. limx→a [cf (x)] = c limx→a f (x) = cL1 CONSTANT MULTIPLE LAW

4. limx→a [f (x)g(x)] = limx→a f (x) · limx→a g(x) = L1 L2 PRODUCT LAW


f (x) limx→a f (x) L1
5. limx→a g(x) = limx→a g(x) = L2 , if limx→a g(x) = L2 6= 0 QUOTIENT LAW

6. limx→a [f (x)]n = [limx→a f (x)]n , where n is a positive integer

7. limx→a c = c LIMIT OF A CONSTANT FUNCTION

8. limx→a x = a

9. limx→a xn = an , where n is a positive integer



n

n
10. limx→a x= a, where n is a positive integer (If n is even, we assume that a > 0).
p
n
p
n
11. limx→a f (x) = limx→a f (x), where n is a positive integer (If n is even, we assume that
limx→a f (x) > 0).
111

Example 5.16. Find the following limits

a) limx→2 x2

b) limx→3 (x2 + 5)

c) limx→3 4x2

d) limx→5 (2x2 − 3x + 4)
x3 +2x2 −1
e) limx→−2 5−3x

Solution 5.16. a) limx→2 x2 = limx→2 c · c = limx→2 · limx→2 x = 2 · 2 = 4

b) limx→3 (x2 + 5) = limx→3 x2 + limx→3 5 = 32 + 5 = 14

c) limx→3 4x2 = 4 limx→3 x2 = 4 · 32 = 36

d)

lim (2x2 − 3x + 4) = lim (2x2 ) − lim (3x) + lim 4


x→5 x→5 x→5 x→5
2
= 2 lim x − 3 lim x + lim 4
x→5 x→5 x→5

= 2(52 ) − 3(5) + 4

= 39

e)

x3 + 2x2 − 1 limx→−2 (x3 + 2x2 − 1)


lim = (by Law 5)
x→−2 5 − 3x limx→−2 (5 − 3x)
limx→−2 x3 + 2 limx→−2 x2 − limx→−2 1
= (by Law 1, 2 and 3)
limx→−2 5 − 3 limx→−2 x
(−2) + 2(−2)2 − 1
3
= (by Law 9, 8 and 7)
5 − 3(−2)
1
=−
11

Theorem 5.8.5. If f (x) = an xn + an−1 xn−1 an−2 xn−2 + · · · + a0 is a polynomial function, and c
is any number, then limx→c f (x) = f (c) = an cn + an−1 cn−1 + an−2 cn−2 + · · · + a0 .
112

Theorem 5.8.6. If f (x) and g(x) are polynomials, and c is a constant, then

f (x) f (x)
lim = , provided that g(c) 6= 0
x→c g(x) g(c)

Example 5.17. Determine the following limits

a) limx→3 x2 (2 − x)
x3 +4x2 −3
b) limx→2 x2 +5

x2 −25
c) limx→−5 3(x+5)

Solution 5.17. a) Since f (x) = x2 (2 − x) = −x3 + 2x2 is a polynomial, then

lim f (x) = f (3) = −(3)3 + 2(32 ) = −9


x→3

b) We have that f (x) = x3 + 4x2 − 3 and g(x) = x2 + 5. So, f (2) = 23 + 4(22 ) − 3 = 21 and
g(2) = 22 + 5 = 9. Thus,

f (x) f (2) 21 7
lim = = =
x→2 g(x) g(2) 9 3

c) We have that f (x) = x2 − 25 and g(x) = 3(x + 5). So f (−5) = (−5)2 − 25 and g(−5) =
f (−5)
3(−5 + 5) = 0, then g(−5) is undefined. Hence

f (x) (x − 5)
(x+5)
 x−5 −10
lim = lim = lim =
x→−5 g(x) 3
(x+ 5) 3 3
x→−5  x→−5

Trigonometric Function and Square Roots

Consider the function


sin x
f (x) =
x

To guess
sin x
lim f (x) = lim
x→0 x→0 x
113

We consider the following table

x -0.01 -0.005 -0.001 0 0.001 0.05 0.01


f (x) 0.9999833 0.9999958 0.9999998 * 0.9999998 0.9999958 0.9999833
−→ * ←−

Table 5.6: Guessing limx→0 f (x)

From Table 5.5 as x approaches 0, the values of the function seem to approach 1. Therefore

sin x
lim =1 (5.14)
x→0 x

We use Equation 5.14 to find the limit as x approaches 0 of trigonometric functions of similar form.

Example 5.18. Determine the following limits


sin(3x)
a) limx→0 4x

b) limx→0 2x cot x
sin θ
c) limx→0 2θ

3x−tan(2x)
d) limx→0 2x

Solution 5.18. a) Since x → 0, then 3x → 0. Thus

sin(3x) sin(3x)
lim = lim
x→0 4x x→0 4x · 3
3
3 sin(3x)
= lim
x→0 4 3x
3 sin(3x)
= lim
4 x→0 3x
3
= ·1
4
3
=
4
114

cos x
b) Note that cot x = sin x and 2x → 0 as x → 0. Thus

cos x
lim 2x cot x = lim 2x ·
x→0 x→0 sin x
x
= 2 lim · cos x
x→0 sin x
1
= 2 lim sin x · cos x
x→0
x
limx→0 1
=2 · lim cos x
limx→0 sinx x x→0
1
=2· ·1
1
=2

sin θ
sin θ
c) limθ→0 2θ = limθ→0 θ
sin 2θ . Thus
θ

sin θ sin θ
θ limθ→0 θ
lim =
θ→0 sin 2θ limθ→0 sinθ2θ
θ
limθ→0 sinθ θ
=
limθ→0 sin 2θ
θ· 22
sin θ
limθ→0 θ
=
2 limθ→0 sin2θ2θ
1
=
2

sin θ 1
∴ limx→0 2θ = 2
 
3x−tan(2x) 3 sin 2x
d) limx→0 2x = limx→0 2 − 2x cos 2x

3 sin 2x  3 sin 2x 1 
lim − = lim − ·
x→0 2 2x cos 2x x→0 2 2x cos 2x
3 sin 2x 1
= lim − lim · lim
x→0 2 x→0 2x x→0 cos x
3 1
= −1·1 =
2 2
115


t2 +9−3
Note that for the function f (x) = t2
in Example 5.14 (c) we made an educated guess that


t2 + 9 − 3 1
lim 2
=
t→0 t 6

However, this limit can also be computed algebraically as follows


√ √ √
t2 + 9 − 3 t2 + 9 − 3 t2 + 9 + 3
lim = lim ·√
t→0 t2 t→0 t 2
t2 + 9 + 3
2
t +9−9
= lim √
t→0 t ( t2 + 9 + 3)
2

t2
= lim √
2 ( t2 + 9 + 3)
t→0 t
1
= lim √
t→0 2
t +9+3
1
=
6

Example 5.19. Compute the following limits


√ √
2x+7− 7
a) limx→0 x

3x+1−1
b) limx→0 x
√ √
c) limh→0 2+h−
h
2

Solution 5.19. a)
√ √ √ √ √ √
2x + 7 − 7 2x + 7 − 7 2x + 7 + 7
lim = lim ·√ √
x→0 x x→0 x 2x + 7 + 7
2x + 7 − 7
= lim √ √
x→0 x( 2x + 7 + 7)
x
2
= lim √ √
x( 2x + 7 + 7)
x→3 
2
= lim √ √
x→0 2x + 7 + 7
116

b)
√ √ √
3x + 1 − 1 3x + 1 − 1 3x + 1 + 1
lim = lim ·√
x→0 x x→0 x 3x + 1 + 1
3x + 1 − 1
= lim √
x→0 x( 3x + 1 + 1)
x
3
= lim √
x( 3x + 1 + 1)
x→3 
3
= lim √
x→0 3x + 1 + 1

c)
√ √ √ √ √ √
2+h− 2 2+h− 2 2+h+ 2
lim = lim ·√ √
h→0 h h→0 h 2x + 7 + 7
2+h−2
= lim √ √
h→0 h( 2 + h + 2)

h
= lim √ √

h→0 h( 2 + h + 2)
2
= lim √ √
h→0 2+h+ 2
2
= √
2 2
1
=√
2

5.8.1 Infinite Limits

Consider the function


1
f (x) =
x2

To find
1
lim f (x) = lim
x→0 x→0 x2

Let’s consider the following table of values of f (x) for values of x close to 0, but not equal to 0.
117

x -0.05 -0.01 -0.001 0 0.001 0.01 0.05


f (x) 400 10,000 1,000,000 * 1,000,000 10,000 400
−→ * ←−

Table 5.7: Guessing limx→0 f (x)

As shown in Figure 5.32 and Table 5.7 the values of f (x) can be made arbitrarily large by taking x
1
values close enough to 0. The values of f (x) do not approach a number, so limx→0 x2
does not exist.

1
Figure 5.32: limx→0 x2

To indicate this kind of behavior, we use the notation

1
lim =∞
x→0 x2

This does not mean that we are regarding ∞ as a number. Nor does it mean that the limit exists.
It simply expresses the particular way in which the limit does not exist: can be made as large as
we like by taking x close enough to 0. This bring us to the following definition

Definition 5.8.7. Let f (x) be a function defined on both sides of a, except possibly at a itself.
Then
lim f (x) = ∞ (5.15)
x→a

This means the values of f (x) can be made arbitrarily large by taking x sufficiently close to a, but
not equal to a.
118

Definition 5.8.8. Let f (x) be a function defined on both sides of a, except possibly at a itself.
Then
lim f (x) = −∞ (5.16)
x→a

This means the values of f (x) can be made arbitrarily large negative by taking x sufficiently close
to a, but not equal to a.

Similar definitions can be given for the one-sided infinite limits

lim f (x) = ∞, lim f (x) = ∞


x→a− x→a+

lim f (x) = −∞, lim f (x) = −∞


x→a− x→a+

as demonstrated on Figure 5.33 below

Figure 5.33: One-sided infinite limits

Definition 5.8.9. The line x = a is called a vertical asymptote of the curve y = f (x) if at least
one of the following statements is true:

lim f (x) = ∞, lim f (x) = ∞, lim f (x) = ∞


x→a x→a− x→a+

lim f (x) = −∞ lim f (x) = −∞, lim f (x) = −∞


x→a x→a− x→a+

Example 5.20. Determine the following limits


1
a) f (x) = x
119

i) limx→0− f (x)

ii) limx→0+ f (x)


2x
b) g(x) = x−3

i) limx→3− g(x)

ii) limx→3+ g(x)

1
Solution 5.20. a) i) f (x) = x increases in a negative direction as x approaches 0 from the left.
Thus
lim f (x) = −∞
x→0−

1
ii) f (x) = x increases as x approaches 1 from the right. Thus

lim f (x) = +∞
x→0−

2x
b) i) g(x) = x−3 increases in a negative direction as x approaches 3 from the left. Thus

lim f (x) = −∞
x→0−

2x
ii) g(x) = x−3 increases as x approaches 3 from the right. Thus

lim f (x) = +∞
x→0−

5.8.2 Limits to Infinity

In this section, we want to look at the limit of the function f (x) as x → ∞ or x → −∞.

Consider the function


1
f (x) =
x

As shown in Figure 5.34 below.


120

1
Figure 5.34: limx→±∞ x

This function is defined for all real numbers except at x = 0. Thus


1
a) When x → ∞, x → 0+ .

b) When x → −∞, 1
x → 0− .

Note. If f (x) is a constant function. i.e. f (x) = k, then limx→∞ f (x) = k and limx→−∞ f (x) = k

Theorem 5.8.10. Laws of limits to infinity


Let f (x) and g(x) be functions such that

lim f (x) = L1 and lim g(x) = L2 , where L1 , L2 ∈ R


x→∞ x→∞

and c be a constant. Then

1. limx→∞ [f (x)+g(x)] = limx→∞ f (x)+limx→∞ g(x) = L1 +L2 SUM LAW

2. limx→∞ [f (x) − g(x)] = limx→∞ f (x) − limx→∞ g(x) = L1 − L2 DIFFERENCE LAW

3. limx→∞ [cf (x)] = c limx→∞ f (x) = cL1 CONSTANT MULTIPLE LAW

4. limx→∞ [f (x)g(x)] = limx→∞ f (x) · limx→∞ g(x) = L1 L2 PRODUCT LAW


f (x) limx→∞ f (x) L1
5. limx→∞ g(x) = limx→∞ g(x) = L2 , if limx→∞ g(x) = L2 6= 0 QUOTIENT LAW
121

Example 5.21. Determine the following limits


 
1
a) limx→∞ 5 + x

π 3
b) limx→−∞ x2

 
1 1
Solution 5.21. a) limx→∞ 5 + x = limx→∞ 5 + limx→∞ x = 5 + 0 = 5.

π 3
√ 1

b) limx→−∞ x2
= π 3 limx→−∞ x2
= π 3 · 0 = 0.

5.8.3 Limits of Rational Functions as x → ±∞

Consider the function


P (x)
r(x) =
Q(x)

To find

lim r(x)
x→±∞

we divide P (x) and Q(x) by the leading (highest) power of x in Q(x).

Here are three cases for limits of rational functions as x → ±∞:

(1.) If deg(P (x)) < deg(Q(x)), limx→±∞ r(x) = 0.


an
(2.) If deg(P (x)) = deg(Q(x)), limx→±∞ r(x) = bn , where an and bn are the leading coefficients of
P (x) and Q(x) respectively.

(3.) If deg(P (x)) > deg(Q(x)), limx→±∞ r(x) = ±∞.

Example 5.22. Determine the following limits


11x+2
a) limx→∞ 2x3 −1
122

15x
b) limx→−∞ − 7x+4
5x2 +8x−3
c) limx→∞ 3x2 +2

−4x3 +7x
d) limx→∞ 2x2 −3x−10

Solution 5.22. a) P (x) = 11x + 2 and Q(x) = 2x3 − 1. Thus deg(P (x)) = 1 < 3 = deg(Q(x)). So,

11
11x + 2 x2
+ x23 11 2 1
lim = lim , , , → 0 as x → ∞
x→∞ 2x3 − 1 x→∞ 2 − 13 x2 x3 x3
x
0
=
2
=0

b) P (x) = 15x and Q(x) = 7x + 4. Thus deg(P (x)) = 1 = deg(Q(x)). So,

15x 15 4
lim − = − lim , → 0 as x → −∞
x→−∞ 7x + 4 x→−∞ 7 + x4 x
15
=−
7

c) P (x) = 5x2 + 8x − 3 and Q(x) = 3x2 + 2. Thus, deg(P (x)) = 2 = deg(Q(x)). So,

5x2 + 8x − 3 5 + x8 − x32 8 3 2
lim = lim , , , → 0 as x → ∞
x→∞ 3x2 + 2 x→∞ 3 + x22 x x2 x2
5
=
3

d) P (x) = −4x3 + 7x and Q(x) = 2x2 − 3x − 10. Thus deg(P (x)) = 3 > 2 = deg(Q(x)).So,

−4x3 + 7x −4x + x7
lim = lim
x→∞ 2x2 − 3x − 10 x→∞ 2 − 3 − 102
x x
−4x
= lim
x→∞ 2

= −2 lim x
x→∞

= −2 · ∞ = −∞
123

Theorem 5.8.11. The Sandwich (Squeeze) Theorem


Let f (x), g(x) and h(x) be functions such that f (x) ≤ h(x) ≤ g(x) for all x in the interval
containing a (except possibly at a). If

lim f (x) = L = lim g(x)


x→a x→a

then
lim h(x) = L
x→a

Example 5.23. Determine the following limits


 
1
a) limx→0 x2 sin x2
 
sin x
b) limx→−∞ 2 + x

Solution 5.23. a) Since


−1 ≤ sin x ≤ 1

then
1
−1 ≤ sin ≤1
x2

Thus,
1
−x2 ≤ x2 sin ≤ x2
x2

and
lim (−x2 ) = 0 = lim x2
x→0 x→0

Therefore,

1
lim x2 sin
x→0 x2

b) We have that

 sin x  sin x sin x


lim 2+ = lim 2 + lim = 2 + lim
x→−∞ x x→−∞ x→−∞ x x→−∞ x
124

Since
−1 ≤ sin x ≤ 1

then
1 sin x 1
− ≤ ≤
x x x

So,
1 1
lim − = 0 = lim
x→−∞ x x→−∞ x

Therefore
sin x
lim =0
x→−∞ x

 
sin x
∴ limx→−∞ 2 + x =2+0=2

5.8.4 Continuous Functions

Sometimes the limit of a function as approaches a can often be found simply by calculating the
value of the function at x = a. Functions with this property are called continuous at at x = a.

Definition 5.8.12. Continuity at a point


A function y = f (x) is continuous at a point a on its domain if

lim f (x) = f (a) (5.17)


x→a

Definition 5.8.12 implicitly requires three things if f (x) is continuous at a:

1. f (a) is defined

2. limx→a f (x) exists

3. limx→a f (x) = f (a)

Definition 5.8.13. Continuity at an end-point


125

A function is continuous from the right at x = a if

lim f (x) = f (a) (5.18)


x→a+

and is continuous from the left at x = a if

lim f (x) = f (a) (5.19)


x→a−

Definition 5.8.14. Continuity of a Function


A function f (x) is continuous if it is continuous at each point on its domain.

Definition 5.8.15. Discontinuity at a point


If a function f is not continuous at a point x = a, we say that f (x) is discontinuous at x = a and
we call x = a a point of discontinuity of f (x).

Example 5.24. a) Determine whether the function



1 − x2 , 0<x≤1


f (x) =

x,

x>1

is continuous at x = 1

b) Show that the function f (x) = 1 − 1 − x2 is continuous on the interval [−1, 1].

Solution 5.24. a) We have that

lim f (x) = lim (1 − x2 ) = 1 − 12 = 0


x→1− x→1

and
lim f (x) = lim x = 1
x→1+ x→1
126

f (1) = 1 − (1)2 = 0

Since limx→1− f (x) = 0 6= 1 = limx→1+ f (x), then f (x) is not continuous at x = 1.

b) If −1 < a < 1, then

 p 
lim f (x) = lim 1 − 1 − x2
x→a x→a
p
= 1 − lim 1 − x2
x→a
p
=1− 1 − a2

= f (a)

Therefore f (x) is continuous at x = a if −1 < a < 1.

Furthermore,

 p 
lim f (x) = lim 1− 1 − x2
x→−1+ x→−1
p
= 1 − lim 1 − x2
x→−1
q
=1− 1 − (−1)2

= f (−1)

and

 p 
lim f (x) = lim 1 − 1 − x2
x→1− x→1
p
= 1 − lim 1 − x2
x→1
p
=1− 1 − 12

= f (1)


∴ f (x) = 1 − 1 − x2 is continuous on the interval [−1, 1].
127

Exercise 5.7. Consider the following

1. Consider the function 


1 − x 2 , x 6= 1


f (x) =

2,

x=1

a) Graph the function f (x).

b) Find limx→1− f (x) and limx→1+ f (x).

c) Does limx→1 f (x) exist? If so, what is it. If not, why not?

2. Determine the following limits

a) limx→0 1+√33x+1
sin 2x
b) limx→ x

10x5 +x4 +31


c) limx→±∞ x6

d) limx→∞ 2+√x
2− x

3
e) limx→2 x−2

2x2 2x2 +5
f) limx→∞ f (x) if x2 +1
< f (x) < x2

3. Find a value a for which the function



2
 x −1 , x 6= 1


x+1
f (x) =

a,

x = −1
128

CHAPTER 6

DIFFERENTIATION

In this chapter we will explorer the concept of differentiation. We first look at the derivative of the
function, where we consider basic applications such as gradients, speed and velocity. Furthermore,
we will study the basic rules for differentiation and some practical applications of differentiation.

6.1 DERIVATIVES AND RATES OF CHANGE

Finding the tangent line to a curve y = f (x) or the velocity of an object both involve finding the
same type of limit. This special type of limit is called a derivative and in this section we will see
how that can be interpreted as a rate of change in any of the sciences or engineering.

6.1.1 TANGENTS

Suppose we want to find the tangent line to the curve C : y = f (x) at the point P (a, f (a)). We
consider a nearby point Q(x, f (x)), and compute the slope of the secant line P Q (see Figure 6.1):

f (x) − f (a)
mP Q = (6.1)
x−a

We let Q approach P along the curve C by letting x approach a. If mP Q approaches a number m,


we can then define the tangent line L to the curve y = f (x) at point P with the slope m as shown
on Figure 6.2.

Definition 6.1.1. The Tangent line to the curve y = f (x) at the point P (a, f (a)) is the line L
through P with slope
f (x) − f (a)
m = lim (6.2)
x→a x−a

Provided that the limit exist.


129

Figure 6.1: Slope of the secant line P Q. Figure 6.2: Tangent to the curve C : y = f (x).

Example 6.1. Find the equation of the tangent line to the parabola y = x2 at the point P (1, 1).

Solution 6.1. Here we have a = 1 and f (x) = x2 , so the slope of the tangent line is given by

f (x) − f (1) x2 − 1
m = lim = lim
x→1 x−1 x→1 x − 1
(x + 1)(x− 1)
= lim
x→1 x−
  1
= lim (x + 1)
x→1

=1+1=2

Therefore, the equation of the tangent line at P (1, 1) is given by

y − yP = m(x − xP ) =⇒ y − 1 = 2(x − 1)

=⇒ y = 2x − 1

We can obtain another expression for the slope of the tangent line by letting h = x − a. Thus,
x = a + h and the slope of the secant line P Q becomes

f (a + h) − f (a)
mP Q = (6.3)
h
130

As shown on Figure 6.3 where h > 0 implies that Q is to the right of P . If h < 0, then Q will be
to the left of P .

As x approaches a, h approaches 0 and so the expression in Definition 6.1.1 become

f (a + h) − f (a)
m = lim (6.4)
h→0 h

Figure 6.3: Tangent to the curve C : y = f (x) where h = x − a

3
Example 6.2. Find an equation of the tangent line to the hyperbola y = x at the point P (3, 1).

Solution 6.2. Let f (x) = x3 . Then the slope of the tangent line at P (3, 1) is

3 3
f (3 + h) − f (3) 3+h− 3
m = lim = lim
h→0 h h→0 h
3−3−h
3+h
= lim
h→0 h
−h
= lim
h→0 h(3 + h)
−1
= lim
h→0 3 + h
1
=−
3

Therefore, the equation of the tangent line at the point P (3, 1) is


131

1
y − yP = m(x − xp ) =⇒ y − 1 = − (x − 3)
3
1
=⇒ y = − x + 2
3

As shown in Figure 6.4

3
Figure 6.4: Tangent to the curve y = x at the point P (3, 1).

6.1.2 VELOCITIES

Suppose an object moves along a straight line according to an equation of motion s = f (t), where
S is the displacement (directed distance) of the object from the origin at time t. The function
s = f (t) is called the position function of the object. In the time interval from t = a to t = a + h
the change in position is f (a + h) = f (a) as shown on Figure 6.5.

The average velocity over this time interval is

displacement f (a + h) − f (a)
average velocity = = (6.5)
time h

which is the same as the slope of the secant line P Q as shown in Figure 6.6.
132

f (a+h)−f (a)
Figure 6.5: Position change from f (a) to f (a + h). Figure 6.6: mP Q = h = average velocity

To compute the average velocities over shorter and shorter time intervals [a, a + h], we let h ap-
proaches 0. So, we define the velocity (or instantaneous velocity) v(a) at time t = a to be the
limit of these average velocities:

f (a + h) − f (a)
v(a) = lim (6.6)
h→0 h

Therefore, the velocity at time t = a is equal to the slope of the tangent line at P .

Example 6.3. Suppose that a ball is dropped from the upper observation deck of the CN Tower
in Toronto, 450 m above the ground.

(a) What is the velocity of the ball after 5 seconds?

(b) How fast is the ball traveling when it hits the ground?

Solution 6.3. We will need to find the velocity both when t = 5 seconds and when the ball hits the
ground, so it’s efficient to start by finding the velocity at a general time t = a.

Through experiments carried out four centuries ago, Galileo discovered that the distance fallen by
any freely falling body is proportional to the square of the time it has been falling. (This model
133

for free fall neglects air resistance.) If the distance fallen after t seconds is denoted by s(t) and
measured in meters, then Galileo’s law is expressed by the equation

s(t) = 4.9t2

Using the equation of motion s(t) = 4.9t2 , we have

f (a + h) − f (a) 4.9(a + h)2 − 4.9a2


v(a) = lim = lim
h→0 h h→0 h
4.9a2 + 9.8ah + 4.9h2 − 4.9a2
= lim
h→0 h
h(9.8a + 4.9h)

= lim
h→0 h

= lim (9.8a + 4.9h)
h→0

= 9.8a

a) The velocity after 5 seconds in v(5) = 9.8(5) = 49 m/s.

b) Since the observation deck is 450 m above the ground, the ball will hit the ground at the time
tG when s(tG ) = 450 m, that is,
4.9t2G = 450

This gives r
450 450
t2G = =⇒ tG = ≈ 9.6 s
4.9 4.9

Therefore, the velocity of the ball as it hits the ground is


!
450
v(tG ) = 4.9 ≈ 94 m/s
4.9

6.1.3 DERIVATIVES

The limits of the form


f (a + h) − f (a)
lim
h→0 h
134

arise whenever we calculate a rate of change in any of the applied sciences or engineering, such as
a rate of reaction in chemistry or a marginal cost in economics. Since this type of limit occurs so
widely, it is given a special name and notation.

Definition 6.1.2. The derivative of a function at a number , denoted by f 0 (a), is

f (a + h) − f (a)
f 0 (a) = lim (6.7)
h→0 h

if this limit exists.

Letting x = a + h, then we have h = x − a and h → 0 if and only if x → a. Therefore an equivalent


way of stating the definition of the derivative, as we saw in finding tangent lines, is f 0 (a), which is
given by:
f (x) − f (a)
f 0 (a) = lim (6.8)
x→a x−a

Equation 6.7 or 6.8 is called the Derivative of the function f from the first principle, and
f 0 (a) " is read “f prime of a.”

Example 6.4. Find the derivative of the function f (x) = x2 −8x+9 at x = 2 from the first principle.

Solution 6.4. From Definition 6.1.2 we have that

f (a + h) − f (a)
f 0 (2) = lim
h→0 h
[(2 + h)2 − 8(2 + h) + 9] − [a2 − 8a + 9]
= lim
→0 h
2
2ah + h − 8h
= lim
→0 h
h(2a + h − 8)

= lim
→0 h

= lim(2a + h − 8)
→0

= 2a − 8
135

∴ f 0 (2) = 2(2) − 8 = −4

We defined the tangent line to the curve y = f (a) at the point P (a, f (a)) to be the line that passes
through P (a, f (a)) and has a slope m given by Equation 6.2 or 6.4. Since, by Definition 6.1.2, this
is the same as the derivative f 0 (a), we can now say the following.

 The tangent line to y = f (x) at P (a, f (a)) is the line through P (a, f (a)) whose slope is equal to
f 0 (a), the derivative of f at a.

Using the point-slope form of the equation of a line, we can write an equation of the tangent line
to the curve y = f (a) at the point P (a, f (a)) as:

y − f (a) = f 0 (a)(x − a)

Example 6.5. Find an equation of the tangent line to the parabola y = x2 − 8x + 9 at the point
P (3, −6).

Solution 6.5. From Example 6.4 we found that the derivative of the function f (x) = x2 − 8x + 9 at
a number a is f 0 (a) = 2a − 8. Therefore, the slope of the tangent line at P (3, −6) is given by

f 0 (3) = −2

Thus, the equation of the tangent line at P (3, −6) (as shown in Figure 6.7), is

y + 6 = −2(x − 3) =⇒ y = −2x
136

Figure 6.7: Tangent to the curve y = x2 − 8x + 9 at the point P (3, −6).

6.1.4 RATES OF CHANGE

Let y be a quantity that depends on another quantity x. Thus y is a function of x and we write
y = f (x). If x changes from x1 to x2 , then the change in x (also called the increment of x) is

∆x = x2 − x1

and the corresponding change in y is

∆y = f (x2 ) − f (x1 )

The difference quotient


∆y f (x2 ) − f (x1 )
=
∆x x2 − x1

is called the average rate of change of y with respect to x over the interval [x1 , x2 ] and can
be interpreted as the slope of the secant line P Q shown in Figure 6.8 below.

We consider the average rate of change over smaller and smaller intervals by letting x2 → x1 and
therefore letting ∆x → 0 . The limit of these average rates of change is called the (instantaneous)
rate of change of y with respect to x at x = x1 , which is interpreted as the slope of the tangent
137

to the curve y = f (x) at P (x1 , f (x1 )) :

∆y f (x2 ) − f (x1 )
instantaneous rate of change = lim = lim = f 0 (x1 ) (6.9)
∆x→0 ∆x x2 →x1 x2 − x1

Figure 6.8: average rate of change mP Q


instantaneous rate of change = slope of tangent at P .

Note. The derivative f 0 (a) is the instantaneous rate of change of y = f (x) with respect to x when
x = a.

Example 6.6. A manufacturer produces bolts of a fabric with a fixed width. The cost of producing
x yards of this fabric is C = f (x) dollars.

(a) What is the meaning of the derivative f 0 (x)? What are its units?

(b) In practical terms, what does it mean to say that f 0 (1000) = 9 ?

(c) Which do you think is greater, f 0 (50) or f 0 (500)? What about f 0 (5000)?

Solution 6.6. (a) The derivative f 0 (x) is the instantaneous rate of change of C with respect to x
i.e. the rate of change of the production cost with respect to the number of yards produced.
138

Since
∆C
f 0 (x) = lim
∆x→0 ∆x

then the units for f 0 (x) are dollars per yard.

(b) f 0 (1000) = 9 means that, after 1000 yards of fabric have been manufactured, the rate at which
the production cost is increasing is $9/yard. (When x = 1000, C is increasing 9 times as fast
as x.)

Since ∆x = 1 is small compared with x = 1000, we could use the approximation

∆C ∆C
f 0 (1000) ≈ = = ∆C
∆x 1

and say that the cost of manufacturing the 1000th yard (or the 1001st ) is about $9.

(c) The rate at which the production cost is increasing (per yard) is probably lower when x = 500
than when x = 50 because of economies of scale

f 0 (50) > f 0 (500)

But, as production expands, the resulting large-scale operation might become inefficient and
there might be overtime costs. Thus it is possible that the rate of increase of costs will eventually
start to rise. So it may happen that

f 0 (5000) > f 0 (500)

OTHER NOTATIONS

If we use the traditional notation y = f (x) to indicate that the independent variable is x and the
dependent variable is y , then some common alternative notations for the derivative are as follows:

(1.) f 0 (x) = y 0
dy df d
(2.) dx = dx = dx f (x)
139

(3.) Df (x) = Dx f (x)

d
The D symbols and dx are called differentiation operators because they indicate the operation
dy
of differentiation. The symbol dx , which was introduced by Leibniz 1 , should not be regarded as
a ratio; it is simply a synonym for f 0 (x).

Referring to Equation 6.9, we can rewrite the definition of derivative in Leibniz notation as

dy ∆y
= lim
dx ∆x→0 ∆x

dy
If we want to indicate the value of a derivative dx in Leibniz notation at a specific number a, we
use the notation #
dy dy
or
dx dx
x=a x=a

which is a synonym for f 0 (a).

Definition 6.1.3. A function f is differentiable at a if f 0 (a) exists. It is differentiable on an


open interval (a, b) [or (a, ∞) or (−∞, a)or (−∞, ∞)] if it is differentiable at every number in
that interval.

Example 6.7. Where is the function y = |x| differentiable?


1
Gottfried Wilhelm Leibniz was born in Leipzig in 1646 and studied law, theology, philosophy, and mathematics
at the university there, graduating with a bachelor’s degree at age 17. After earning his doctorate in law at age
20, Leibniz entered the diplomatic service and spent most of his life traveling to the capitals of Europe on political
missions. In particular, he worked to avert a French military threat against Germany and attempted to reconcile the
Catholic and Protestant churches.
His serious study of mathematics did not begin until 1672 while he was on a diplomatic mission in Paris. There he
built a calculating machine and met scientists, like Huygens, who directed his attention to the latest developments
in mathematics and science. Leibniz sought to develop a symbolic logic and system of notation that would simplify
logical reasoning. In particular, the version of calculus that he published in 1684 established the notation and the
rules for finding derivatives that we use today.
Unfortunately, a dreadful priority dispute arose in the 1690s between the followers of Newton and those of Leibniz
as to who had invented calculus first. Leibniz was even accused of plagiarism by members of the Royal Society in
England. The truth is that each man invented calculus independently. Newton arrived at his version of calculus first
but, because of his fear of controversy, did not publish it immediately. So Leibniz’s 1684 account of calculus was the
first to be published.
140

Solution 6.7. Recall that 


−x,

 if x < 0
f (x) = |x| =

x,

if x > 0

as defined in Chapter 5.

If x < 0, then |x| = −x and we can choose h small enough that x + h < 0 and so |x + h| = −(x + h).
Therefore, for x < 0 we have that

|x + h| − |x|
f 0 (x) = lim
h→0 h
−(x + h) + x
= lim
h→0 x
−h
= lim
h→0 h

= lim (−1) = −1
h→0

∴ f is differentiable for any x < 0.

Similarly, for x > 0 we have |x| = x and we can choose h small enough that x + h > 0 and so
|x + h| = x + h. Therefore, for x > 0 we have that

|x + h| − |x|
f 0 (x) = lim
h→0 h
(x + h) − x
= lim
h→0 x
h
= lim
h→0 h

= lim 1 = 1
h→0

∴ f is differentiable for any x > 0.


141

For x = 0, let’s investigate

f (0 + h) − f (0)
f 0 (0) = lim
h→0 h
|0 + h| − |0|
= lim if it exists
h→0 h

We will compute the limit as h approaches 0 from the left and the right.

|0 + h| − |0| |h| −h
lim = lim = lim = lim (−1) = −1
h→0− h h→0 − h h→0 h h→0

and
|0 + h| − |0| |h| h
lim = lim = lim = lim 1 = 1
h→0+ h h→0− h h→0 h h→0

|0+h|−|0| |0+h|−|0|
Since limh→0− h 6= limh→0+ h , then f 0 (0) does not exist.

Therefore, f is differentiable at all except at x = 0.

Thus f 0 is given by 
−1,

 if x < 0
f 0 (x) =

1,

if x > 0

as shown in Figure 6.10. f 0 (0) does not exist because the curve y = |x| does not have a tangent
line at P (0, 0) as shown in Figure 6.9.

Figure 6.9: Graph of f (x) = |x|. Figure 6.10: Graph of f 0 (x).


142

Continuity and differentiability are desirable properties for a function to have. The following
theorem shows the relationship between continuity and differentiability.

Theorem 6.1.4. If the function y = f (x) is differentiable at x = a , then it is continuous at x = a.

However, not all continuous functions are differentiable. For example, the function f (x) = |x| is
continuous at x = 0 since
lim f (x) = lim |x| = 0 = f (0)
x→0 x→0

But in Example 6.7 we showed that f (x) is not differentiable at x = 0.

HOW CAN A FUNCTION FAIL TO BE DIFFERENTIABLE?

The function y = |x| in Example 6.7 is not differentiable at x = 0. Its graph changes direction
abruptly when x = 0 as shown in Figure 6.9. In general, there are three possibilities for which the
function y = f (x) will not be differentiable at a point x = a.

(1.) If the graph of a function f (x) has a “corner” or “kink” at x = a, then the graph of f has no
tangent at that particular point and it is not differentiable there.

(2.) If the function y = f (x) is not continuous at x = a then is not differentiable at x = a.

(3.) If the curve y = f (x) has a vertical tangent line when x = a; that is, f is continuous at
x = a and
lim f (x) = ∞
x→a

This means that the tangent lines become steeper and steeper as x → a. The above three
possibilities are demonstrated on the figures below.

Figure 6.11: Three ways for y = f (x) not to be differentiable at x = a.


143

6.1.5 HIGHER DERIVATIVES

If f is a differentiable function, then its derivative f 0 is also a function and it may have a derivative
of its own, denoted by (f 0 )0 = f 00 . This new function is called the second derivative of f because it
is the derivative of the derivative of f . Using Leibniz notation, we write the second derivative of
y = f (x) as
!
d dy d2 y
=
dx dx dx2

Example 6.8. If f (x) = x3 − x, find and interpret f 00 (x).

Solution 6.8. The first derivative is given by

f (x + h) − f (x)
f 0 (x) = lim
h→0 h
(x + h) − (x + h) − x3 + x
3
= lim
h→0 h
x + 3x h + 3xh2 + h3 − x − h − x3 + x
3 2
= lim
h→0 h
2 2
3x h + 3xh + h − h3
= lim
h→0 h
h(3x2 + 3xh + h2 − h)

= lim
h→0 h

= lim (3x2 + 3xh + h2 − 1)
h→0

= 3x2 − 1

So, the second derivative

f 0 (x + h) − f 0 (x)
f 00 (x) = lim
h→0 h
3(x + h)2 − 1 − 3x2 + 1
= lim
h→0 h
3x2 + 6xh + 3h2 − 3x2
= lim
h→0 h
h(6x + 3h)

= lim
h→0 h

= lim (6x + 3h) = 6x
h→0
144

We can interpret f 00 (x) as the slope of the curve f 0 at the point P (x, f 0 (x)).

In general, we can interpret a second derivative as a rate of change of a rate of change.

Consider s = s(t) as the position function of an object that moves in a straight line, we know that
its first derivative represents the velocity v(t) of the object as a function of time:

ds
v(t) = s0 (t) =
dt

The instantaneous rate of change of velocity with respect to time is called the acceleration of
the object. So, acceleration function is the derivative of the velocity function and is therefore the
second derivative of the position function:

a(t) = v 0 (t) = s00 (t)

or, in Leibniz notation,


dv d2 s
a(t) = = 2
dt dt

The third derivative f 000 is the derivative of the second derivative (f 00 )0 : f 000 = (f 00 )0 . So f 000 (x)
can be interpreted as the slope of the curve y = f 00 (x) or as the rate of change of f 00 (x). If y = f (x),
then alternative notations for the third derivative are:
!
000 000 d d2 y d3 y
y = f (x) = =
dx dx2 dx3

The process can be continued. The fourth derivative is f 0000 usually denoted by f (4) . In general,
the nth derivative of f is denoted by f (n) and is obtained from f by differentiating f n times. If
y = f (x), we write
dn y
y (n) = f (n) (x) =
dxn

Example 6.9. If f (x) = x3 − x, find f 000 and f (4) (x).


145

Solution 6.9. In example 6.8 we found that f 00 (x) = 6x. The graph of the second derivative has
equation y = 6x and so it is a straight line with slope 6. Since the derivative f 000 (x) is the slope of
f 00 (x), we have that
f 000 (x) = 6

for all values of x. So f 000 is a constant function and its graph is a horizontal line. Therefore, for
all values of x,
f (4) = 0

Exercise 6.1. 1. Find the derivative of the following functions from the first principle.

(a) f (x) = −x2 + 3

(b) f (x) = 34 x3 − 3

(c) f (x) = 6x2 − 10x − 5x2

2. For each of the function S = f (t) of the body moving along a coordinate line during a time
interval a ≤ t ≤ b, with S in meters and t in seconds. Find the body’s speed and acceleration
at the beginning and at the end of the interval.

(a) S = 0.8t2 , 0 ≤ t ≤ 10 (free fall on the moon)



(b) S = 2t − 4 t, 1 ≤ t ≤ 4.

3. A rock thrown vertically upward from the surface of the moon at a velocity of 24 m/s reaches
a height of S = 24t − 0.8t2 meters in t seconds.

(a) Find the rock’s velocity and acceleration at time t.

(b) How long does it take the rock to reach its highest point?

(c) How high does the rock go?

(d) How long does it take the rock to reach half its maximum height?

(e) How long is the rock a loft?


146

6.2 DIFFERENTIATION RULES

In the previous section, we have seen how to interpret derivatives as slopes and rates of change and,
used the definition of a derivative to calculate the derivatives of functions defined by formulas. But
using the definition is not always easy, so in this section we will study rules for finding derivatives
without having to use the definition directly. These differentiation rules enable us to easily calculate
the derivatives of polynomials, rational functions, algebraic functions, exponential and logarithmic
functions, and trigonometric and inverse trigonometric functions. We then use these rules to solve
problems involving rates of change.

6.2.1 DERIVATIVES OF POLYNOMIALS AND EXPONENTIAL FUNCTIONS

Here we will learn how to differentiate constant functions, power functions, polynomials, and ex-
ponential functions.

(1.) DERIVATIVE OF A CONSTANT: Let f (x) = c where c is a constant, then

d
(c) = 0 (6.10)
dx

For example, if f (x) = 6, then f 0 (x) = 0.

(2.) THE POWER RULE: Let f (x) = xn where where n is any real number, then

d n
(x ) = nxn−1 (6.11)
dx

Example 6.10. Find f 0 (x) where,

a) f (x) = x1000
1
b) f (x) = x2

3
c) f (x) = x2
147

Solution 6.10. a) f 0 (x) = 1000x1000−1 = 1000x999

b) Since f (x) = 1
x2
= x−2 , then

2
f 0 (x) = −2x−2−1 = −2x−3 = −
x3


3
 1
3 2
c) Since f (x) = x2 = x2 = x 3 , then

2 2 2 1 2
f 0 (x) = x 3 −1 = x− 3 = √
3 3 33x

(3.) THE CONSTANT MULTIPLE RULE: Let f be a differentiable and c be a constant,


then
d d
[cf (x)] = c f (x) (6.12)
dx dx

For example,
d 4 d
a) dx (3x ) = 3 dx (x4 ) = 3(4x3 ) = 12x3
d d
b) dx (−x) = − dx (x) = −1(1) = −1

(4.) THE SUM RULE: Let f and g be differentiable functions, then

d d d
[f (x) + g(x)] = f (x) + g(x) (6.13)
dx dx dx

(5.) THE DIFFERENCE RULE: Let f and g be differentiable functions, then

d d d
[f (x) − g(x)] = f (x) − g(x) (6.14)
dx dx dx

dy
Example 6.11. a) Find dx where y = x8 + 12x5 − 4x4 + 10x3 − 6x + 5.

b) Find the points on the curve y = x4 − 6x2 + 4 where the tangent line is horizontal.
148

Solution 6.11. a)

dy d 8
= (x + 12x5 − 4x5 10x3 − 6x + 5)
dx dx
d 8 d d d d d
= (x ) + (12x5 ) − (4x4 ) + (10x3 ) − (6x) + (5)
dx dx dx dx dx dx
= 8x7 + 60x4 − 16x3 + 30x2 − 6

b) The points for which the tangent line is horizontal the gradient is equals to 0. Thus,

dy d 4
m= = 0 =⇒ (x − 6x2 + 4) = 0
dx dx
d 4 d d
=⇒ (x ) − (6x2 ) + (4) = 0
dx dx dx
=⇒ 4x3 − 12x = 0

=⇒ 4x(x2 − 3) = 0
√ √
=⇒ 4x(x − 3)(x + 3) = 0

√ √ √
∴ x = 0 and x = ± 3. The corresponding points are (− 3, −5), (0, 4) and ( 3, −5).

EXPONENTIAL FUNCTIONS

Let’s consider computing the derivative of the exponential function f (x) = ax from the first
principle.

f (x + h) − f (x)
f 0 (x) = lim
h→0 h
ax+h − ax
= lim
h→0 h
ax ah − ax
= lim
h→0 h
ax (ah − 1)
= lim
h→0 h
h
a −1
= ax lim
h→0 h

ah −1
Notice that f 0 (0) = limh→0 h
149

Since f (x) = ax is differentiable at x = 0, then it is differentiable everywhere and,

f 0 (x) = f 0 (0)ax (6.15)

The equation above shows that the rate of change of any exponential function is proportional
to the function itself.

For example, let’s find f 0 (x) for a = 2 and a = 3 using the table below.

2h −1 3h −1
h h h
±0.1 0.7177 1.1612
±0.01 0.6956 1.1047
±0.001 0.6934 1.0992
±0.0001 0.6932 1.0987
ah −1
Table 6.1: f 0 (0) = limh→0 h .

From Table 6.1, we observe that the limits exist and

2h − 1
for a = 2, f 0 (0) = lim ≈ 0.69
h→0 h
3h − 1
for a = 3, f 0 (0) = lim ≈ 1.10
h→0 h

Therefore, from Equation 6.15 we have that

d x
(2 ) ≈ (0.69)2x
dx
d x
(3 ) ≈ (1.10)3x
dx

The simplest differential formula occurs when f 0 (0) = 1. Thus there exist a number a such
that 2 < a < 3 for which f 0 (0) = 1. traditionally, we denote this value by e.

Definition 6.2.1. The Definition of the number e


150

e is the number such that


eh − 1
lim =1 (6.16)
h→0 h

Geometrically, this means that of all the possible exponential functions, the function f (x) = ex
is the one whose tangent line at (0, 1) has a slope f 0 (0) = 1 as shown in Figure 6.12 and Figure
6.13 below.

Figure 6.12 Figure 6.13

(6.) DERIVATIVE OF THE NATURAL EXPONENTIAL FUNCTION: Let f (x) = ex ,


then
d x
(e ) = ex (6.17)
dx

d x d
Note. Make no mistake! If a 6= e then dx (a ) 6= ax . However, x
dx (a ) = (ln a)ax . e.g.
d x
dx (2 ) = (ln 2)2x . Why? Hold that thought! We will revisit this when we talk about chain
rules for differentiation.
151

Example 6.12. If f (x) = ex − x, find f 0 and f 00 .

Solution 6.12. Using the differentiation rules, we have that

d x d x d
f 0 (x) = (e − x) = (e ) − (x) = ex − 1
dx dx dx

and
d x d x d
f 00 (x) = (e − 1) = (e ) − (1) = ex
dx dx dx

(7.) THE PRODUCT RULE: Let f and g be differentiable function, then

d d d
[f (x)g(x)] = g(x) [f (x)] + f (x) [g(x)] (6.18)
dx dx dx

Example 6.13. For the function f (x) = xex find,

a) f 0 (x), and

b) The nth derivative.

Solution 6.13. a) By the Product Rule, we have

d d d
f 0 (x) = (xex ) = ex (x) + x (ex )
dx dx dx
= xex + ex

= (x + 1)ex

b) Using the Product Rule for the second time we get,


152

d
f 00 (x) = ((x + 1)ex )
dx
d d
= (x + 1) (ex ) + ex (x + 1)
dx dx
= xex + 2ex

= (x + 2)ex

Using the Product Rule for the third time we will get f 000 (x) = (x + 3)ex . Therefore, the
nth derivative is f (n) (x) = (x + n)ex .

(8.) THE QUOTIENT RULE: Let f and g be differentiable function, then

d d
" #
d f (x) g(x) dx [f (x)] − f (x) dx [g(x)]
= i2 (6.19)
dx g(x)
h
g(x)

Example 6.14. Find,


dy x2 +x−2
a) dx where y = x3 +6
.
ex
b) An equation of the tangent line to the curve y = 1+x2
at the point P (1, 2e )

Solution 6.14. a) Using the Quotient Rule, we have

d d
dy (x3 + 6) dx (x2 + x − 2) − (x2 + x − 2) dx (x3 + 6)
=
dx (x3 + 6)2
(x3 + 6)(2x + 1) − (x2 + x − 2)(3x2 )
=
(x3 + 6)2
2x4 + x3 + 12x + 6 − 3x4 − 3x3 + 6x2
=
(x3 + 6)2
−x4 − 2x3 + 6x2 + 12x + 6
=
(x3 + 6)2
153

b) From the Quotient Rule, we have that

d d
dy (1 + x2 ) dx (ex ) − ex dx (1 + x2 )
=
dx (1 + x2 )2
(1 + x )e − ex (2x)
2 x
=
(1 + x2 )2
x2 ex + ex − 2xex
=
(1 + x2 )2
(x2 − 2x + 1)ex
=
(1 + x2 )2
(x − 1)2 ex
=
(1 + x2 )2

The slope of the tangent line at the point P (1, 2e ) is given by

dy
m= =0
dx
x=1

∴ The tangent line at the point P (1, 2e ) is horizontal, and it is y = e


2

(9.) THE RECIPROCAL RULE: Let f be a differentiable function, then

d
!
d 1 [f (x)]
= − dx (6.20)
dx f (x) [f (x)]2

1
Example 6.15. Find the derivative of the function f (x) = x2 +6

Solution 6.15. By the Reciprocal Rule, we have

d
(x2 + 6)
f 0 (x) = − dx 2
(x + 6)2
2x
=− 2
(x + 6)2
154

The derivative rules we have learned can be summarized as follow.

d d n d x
(1.) (c) = 0 (2.) (x ) = nxn−1 (3.) (e ) = ex
dx dx dx
(4.) (cf )0 = cf 0 (5.) (f + g)0 = f 0 + g 0 (6.) (f − g)0 = f 0 − g 0
!0 !0
f gf 0 − f g 0 1 f0
(7.) (f g)0 = gf 0 + f g 0 (8.) = (9.) =−
g g2 f f2

dy d2 y
Exercise 6.2. 1. For each of the following functions find dx and dx2
.

a) y = 6x2 − 10x − 5x3


  
1 1
b) y = x + x x− x

(x+1)(x+2)
c) y = (x−1)(x−2)

1
d) y = (x2 −1)(x2 +x+1)

e) y = √x
2 x−7

2. Find an equation for the tangent line to the given curve at the given value of x.
1
a) y = x − 2x , x = − 12
x 1
b) y = 2 + 2x−4 , x=3

3. If the gas in a cylinder is maintained at a constant temperature T , the pressure P is related to


nRT 2
the volume by a formula of the form P = V −nb − Van62 in which a, b, n and R are constants. Find
dP
dV .

6.2.2 DERIVATIVES OF TRIGONOMETRIC FUNCTIONS

In Section 5.3 we have studied trigonometric functions. In this section we will be studying the
derivatives of the six basic trigonometric functions: sin, cos, tan, csc, sec and cot. With some basic
knowledge on the derivative of the six basic trigonometric functions and the help of differentiation
rules we have studied in Section 6.2.1, the can extend the idea to even complex trigonometric func-
tions.
155

The derivatives of the six main trigonometric functions are as follow:

d
(1.) (sin x) = cos x
dx
d
(2.) (cos x) = − sin x
dx
d
(3.) (tan x) = sec2 x
dx
d
(4.) (csc x) = − csc x cot x
dx
d
(5.) (sec x) = sec x tan x
dx
d
(6.) (cot x) = − csc2 x
dx

dy
Example 6.16. 1. Find dx when y = x2 sin x.
sec x
2. Let f (x) = 1+tan x ,

a) Find f 0 (x).

b) For what values of x does the graph of f has a horizontal tangent line?

3. An object at the end of a vertical spring is stretched 4 cm beyond its rest position and released
at time t = 0. Its position at time t is given by

s = f (t) = 4 cos t

Find the velocity and acceleration at time t.

Solution 6.16. 1. Using the Product Rule for differentiation, we have

dy d d
= sin x (x2 ) + x2 (sin x)
dx dx dx
= 2x sin x + x2 cos x
156

2. a) Using the Quotient Rule, we have

d d
(1 + tan x) dx (sec x) − sec x dx (1 + tan x)
f 0 (x) = 2
(1 + tan x)
(1 + tan x) sec x tan x − sec x(sec2 x)
=
(1 + tan x)2
sec x tan x + sec x tan2 x − sec3 x
=
(1 + tan x)2
sec x(tan x + tan2 x − sec2 x)
=
(1 + tan x)2
sec x(tan x − 1)
=
(1 + tan x)2

b) The graph of f has a horizontal tangent line for all x such that f 0 (x) = 0. Thus

sec x(tan x − 1) = 0 =⇒ sec x = 0 or tan x = 1

π
Since sec x is never 0, then this can only occur when tan x = 1 =⇒ x = 4 + nπ, where
n ∈ Z.

3. The velocity and acceleration at time time are given by

ds d
v(t) = = (4 cos t) = −4 sin t
dt dt

and
dv d2 s d
a(t) = = 2 = (−4 sin t) = −4 cos t
dt dt dt

dy
Exercise 6.3. 1. Find dx where,

a) y = −10x + 3 cos x

b) y = x sin x + cos x

c) (sec x + tan x)(sec x − tan x)

d) y = x2 cos x − 2x sin x − 2 cos x


157

ds
2. Find dt for
sin t
a) s = 1−cos t

b) s = t2 − sec t + 1
dr
3. Find dθ for

a) r = sec θ csc θ

b) r = θ sin θ + cos θ

c) r = sin θ(1 + sec θ)

6.2.3 THE CHAIN RULE

Suppose we want to differentiate the function

p
F (x) = x2 + 1

The differentiation formulas we have learned earlier do not enable us to calculate F 0 (x). We notice

that F is a composite function. Thus, if we let y = f (u) = u and u = g(x) = x2 + 1. So, we write

y = F (x) = f (g(x))

Thus, F = f ◦ g and we know how to differentiate both f and g. So, it would be helpful to have a
rule for finding the derivative of F = f ◦ g.

du dy
Let dx be the rate of change of u with respect to x, du be the rate of change of y with respect to
dy
u, and dx be the rate of change of y with respect to x. We expect that

dy dy du
=
dx du dx

(1.) THE CHAIN RULE: Let g be differentiable at x and f be differentiable at g(x), then
F = f ◦ g defined by F (x) = f (g(x)) is differentiable at x and F 0 is given by the product
158

F 0 (x) = f 0 (g(x)) · g 0 (x) (6.21)

In Leibniz notation, if y = f (u) and u = g(x) are both differentiable functions, then

dy dy du
= (6.22)
dx du dx

Example 6.17. Find F 0 (x) for the following functions



a) F (x) = x2 + 1

b) F (x) = sin(x2 )

c) F (x) = sin2 x

√ √ dy 1
Solution 6.17. a) Using The Chain Rule, let y = u and u = x2 + 1. So, du = √
2 u
and
du
dx = 2x, then

dy du
F 0 (x) =
du dx
1
= √ (2x)
2 u
1
= √ (2x)
2 x2 + 1
x
=√
x2 + 1

b) Similarly, using Chain Rule, let g(x) = x2 and f (g(x)) = sin(g(x)). So, g 0 (x) = 2x and
f 0 (g(x)) = cos(g(x)), then

F 0 (x) = f 0 (g(x))g 0 (x)

= cos(g(x))(2x)

= 2x cos(x2 )
159

dy du
c) Note that sin2 x = (sin x)2 . Let y = u2 and u = sin x. So, du = 2u and dx = cos x, then

dy du
F 0 (x) =
du dx
= 2u cos x

= 2 sin x cos x

(2.) THE POWER RULE COMBINED WITH THE CHAIN RULE: Let n be any real
number and u = g(x) be a differentiable function, then

d n du
(u ) = nun−1 (6.23)
dx dx

Alternatively,
d d
[g(x)]n = n[g(x)]n−1 · [g(x)] (6.24)
dx dx

Example 6.18. Differentiate the following functions:

a) y = (x3 − 1)100
1
b) f (x) = √
x2 +x+1
!9
t−2
c) g(x) = 2t+1

Solution 6.18. (a) Let u = g(x) = x3 − 1 and n = 100. So, g 0 (x) = 3x2 , then

dy d 3
= (x − 1)100
dx dx
= 100(x3 − 1)99 g 0 (x)

= 100(x3 − 1)(3x2 ) = 100x2 (x3 − 1)

1
(b) Let’s rewrite f as f (x) = (x2 +x+1)− 2 . Let u = x2 +x+1 and n = − 12 . So, g 0 (x) = 2x+1,
160

then

d 2 1
f 0 (x) = (x + x + 1)− 2
dx
1 3
= − [g(x)]− 2 g 0 (x)
2
1 2 3
= − (x + x + 1)− 2 (2x + 1)
2

(c) By combining the Power Rule, Chain Rule and Quotient Rule, we get

!8 !
0 t−2 d t−2
g (x) = 9
2t + 1 dx 2t + 1
!8
t−2 (2t + 1) − 2(t − 2)
=9
2t + 1 (2t + 1)2
!8
t−2 5
=9
2t + 1 (2t + 1)2
45(t − 2)8
=
(2t + 1)3

6.2.4 DERIVATIVES OF HYPERBOLIC FUNCTIONS

Similar to trigonometric functions, derivatives of hyperbolic functions are given in the table
below:

d
(1.) (sinh x) = cosh x
dx
d
(2.) (cosh x) = sinh x
dx
d
(3.) (tanh x) = sech2 x
dx
d
(4.) (cschx) = −cschx coth x
dx
d
(5.) (sechx) = −sechx tanh x
dx
d
(6.) (coth x) = −csch2 x
dx
161

Exercise 6.4. Prove the above derivatives using formulas for hyperbolic functions we learned
in Section 5.5.

dy
Exercise 6.5. 1. Find dx for the following functions at the given points:

a) y = u5 + 1, u = x at x = 1
 2
u−1 1
b) y = u+1 ,u= x2
− 1 at x = −1
dy
2. Find dx for

a) y = tan(2x − x3 )

b) y = cot(π − x1 )

c) y = sec(x2 + 2)

3. Find f 0 (x) for the following functions


 −7
x
a) f (x) = 1 − 7
 5
x 1
b) f (x) = 5 + 5x
162

6.3 IMPLICIT DIFFERENTIATION

The functions that we have met so far can be described by expressing one variable explicitly in
terms of another variable (say y = f (x)). Some functions, however, are defined implicitly by a
relation between x and y such as

x2 + y 2 = 25 and x3 + y 3 = 6xy

It’s not easy to solve these equations for y explicitly as a function of x by hand. Fortunately, we
don’t need to solve an equation as y = f (x) in order to find the derivative of y. Instead, we can use
the method of implicit differentiation. This consists of differentiating both sides of the equation
with respect to x and then solving the resulting equation for y 0 .

dy
Example 6.19. a) If x2 + y 2 = 25, find dx .

b) Find an equation of the tangent line to the circle x2 + y 2 = 25 at the point P (3, 4).

Solution 6.19. a) Let’s differentiate both sides of the equation x2 + y 2 = 25 with respect to x.

d 2 d
(x + y 2 ) = (25)
dx dx
d 2 d 2
(x ) + (y ) = 0
dx dx

Remembering that y = f (x) and using the Chain Rule, we have

d 2 d dy
(x ) + (y 2 ) =0
dx dy dx
dy
2x + 2y =0
dx
dy x
=−
dx y

So, at the point P (3, 4) we have that the gradient of the tangent line is

dy 3
m= =−
dx (3,4) 4
163

Therefore, the equation of the tangent line at the point P (3, 4) is given by

3 3 25
y − 4 = − (x − 3) =⇒ y = − x +
4 4 4

Exercise 6.6. 1. For the folium of Descartes x3 + y 3 = 6xy,

a) Find y 0 .

b) Find the tangent to the folium of Descartes at the point P (3, 3).

c) At what points in the first quadrant is the tangent line horizontal?

2. Find y 0 if sin(x + y) = y 2 cos x.

6.3.1 DERIVATIVES OF INVERSE TRIGONOMETRIC FUNCTIONS

The inverse trigonometric functions were reviewed in Section 5.6.1. In this section we will use
implicit differentiation to find the derivatives of inverse trigonometric functions.

Recall the definition of arcsine function:

π π
y = sin−1 x =⇒ sin y = x and − ≤y≤
2 2

By differentiation sin y = x implicitly with respect to x, we get

dy dy 1
cos y =⇒ =
dx dx cos y

Since − π2 ≤ y ≤ π2 , then 0 ≤ cos y < 1, so

q q p
cos y = 1 − sin2 y = 1 − (sin(sin−1 y))2 = 1 − x2

Therefore,
dy d 1
= (sin−1 x) = √ (6.25)
dx dx 1 − x2
164

dy 1
=√
dx 1 − x2

Similarly, for the derivative of arctangent function; if y = tan−1 x then x = tan y . Differentiating
this latter equation implicitly with respect to x, we get

dy dy 1
sec2 y = 1 =⇒ =
dx dx sec2 y
1
=
1 + tan2 y
1
=
1 + x2

Therefore
d 1
(tan−1 x) = (6.26)
dx 1 + x2

Example 6.20. Differentiate


1
a) y = sin−1 x

b) f (x) = x tan−1 x

Solution 6.20. a) By using the Reciprocal Rule, we have

dy d
(sin−1 x)
= − dx −1 2
dx (sin x)
√ 1
1−x2
= −
(sin−1 x)2
1
= −√
1− x2 (sin−1 x)2

b) By using the Product Rule and Chain Rule, we have


165

√ d d √ d √
f 0 (x) = tan−1x (x) + x (tan−1 x) ( x)
dx dx dx
√ x 1
= tan−1 x + √
1+x2 x

√ x
= tan−1 x +
2(1 + x)

The derivatives of the remaining four inverse trigonometric functions are given in the table below.

DERIVATIVES OF INVERSE TRIGONOMETRIC FUNCTIONS

d 1 d 1
(1.) (sin−1 x) = √ (4.) (csc−1 x) = − √
dx 1 − x2 dx x 1 − x2
d 1 d 1
(2.) (cos−1 x) = − √ (5.) (sec−1 x) = √
dx 1 − x2 dx x 1 − x2
d 1 d 1
(3.) (tan−1 x) = (6.) (cot−1 x) = −
dx 1 + x2 dx 1 + x2

Exercise 6.7. Find the derivatives of the functions below. Simplify where possible

a) f (x) = tan−1 x

b) g(x) = sec−1 (2x + 1)


 
c) h(x) = cot−1 x + cot−1 1
x

d) i(x) = csc−1 (e2x )

6.3.2 DERIVATIVES OF LOGARITHMIC FUNCTIONS

In this section we use implicit differentiation to find the derivatives of the logarithmic functions
y = loga xand, in particular, the natural logarithmic function y = ln x. It can be proved that
logarithmic functions are differentiable; this is certainly plausible from their graphs in Section 5.4.
166

Let y = loga x, then


d 1
(loga x) = (6.27)
dx x ln a

Proof. Let y = loga x, then


ay = x

By applying implicit differentiation with respect to x, we have

d y dy d dy
(a ) = (x) =⇒ (ln a)ay =1
dx dx dx dx
dy 1 1
=⇒ = y =
dx a ln a x ln a

Let a = e, then loge x = ln x and ln e = 1

d 1
(ln x) = (6.28)
dx x

Now, lets come back to the exponential equation y = ax . We stated that

dy d x
= (a ) = (ln a)ay
dx dx

here is the reason why.

By applying ln of both sides of the equation y = ax , we get

ln y = ln ax =⇒ ln y = x ln a
167

By differentiation implicitly with respect to x, we get

d d d dy d
(ln y) = (x ln a) =⇒ (ln y) = ln a (x)
dx dx dy dx dx
1 dy
=⇒ = ln a
y dx
dy
=⇒ = (ln a)y
dx

dy
∴ dx = (ln a)ax .

CHAIN RULE FOR LOGARITHMIC FUNCTIONS: Let F (x) = f (u) = ln u where u =


g(x) such that f and g are differentiable at u and x respectively. Then,

d 1 du d g 0 (x)
(ln u) = or [ln g(x)] = (6.29)
dx u dx dx g(x)

Example 6.21. Differentiate the following functions:

a) f (x) = ln(x3 + 1)

b) g(x) = ln(sin x)

Solution 6.21. a) By using the Chain Rule for logarithmic functions, let u = ln u and u = x3 + 1,
then

dy du
f 0 (x) =
du dx
1
= (3x2 )
u
3x2
= 3
x +1
168

b) Similarly, let y = ln u and u = sin x, then

dy dy du
=
dx du dx
1
= (cos x)
u
cos x
= = cot x
sin x

DERIVATIVES OF INVERSE HYPERBOLIC FUNCTIONS


The inverse hyperbolic functions were reviewed in Section 5.6.2. We observe that inverse hyperbolic
functions are given as logarithmic functions. We apply the Chain Rule for logarithmic to obtain
expressions for inverse hyperbolic functions given below:

d 1 d 1
(1.) (sinh−1 x) = √ (4.) (csch−1 x) = − √
dx 1 + x2 dx |x| x2 + 1
d 1 d 1
(2.) (cosh−1 x) = √ (5.) (sech−1 x) = − √
dx 2
x −1 dx x 1 − x2
d 1 d 1
(3.) (tanh−1 x) = (6.) (coth−1 x) =
dx 1 − x2 dx 1 − x2

dy
Exercise 6.8. 1. For the following functions, find dx .

a) y = log2 (1 − 3x)

b) y = ln(x + x2 − 1)

c) y = ln(sec x + tan x)

2. Find an equation of the tangent line to the curve at the given point.
 2

a) y = ln xex , at P (1, 1)

b) y = ln(x3 − 7), at P (2, 1)

3. Prove the formulas for derivatives of inverse hyperbolic functions.


169

6.4 APPLICATIONS OF DIFFERENTIATION

In this section we will look at some applications of differentiation where we first look at some
theories behind the first and second derivatives; such as finding extreme values of functions as well
minimizing or maximizing some functions. Furthermore, we will look at some practical applications
of differentiation involving related rates of changes.

6.4.1 EXTREME VALUES AND FIRST AND SECOND DERIVATIVE TESTS

Some of the most important applications of differential calculus are optimization problems, in which
we are required to find the optimal (best) way of doing something.

MAXIMUM AND MINIMUM VALUES

Definition 6.4.1. A function f has an absolute maximum (or global maximum) at c if


f (c) ≥ f (x) for all x in Df where Df is the domain of f . The number f (c) is called the maxi-
mum value of f on Df . Similarly, f has an absolute minimum at c if f (c) ≤ f (x) for all x in
Df and the number is called the minimum value of on Df . The maximum and minimum values
of are called the extreme values of f .

Definition 6.4.2. A function has a local maximum (or relative maximum) at c if f (c) ≥ f (x)
when x is near c. Similarly, f has a local minimum at c if f (c) ≥ f (x) when is x near c.

For example

1.) If f (x) = x2 , then f (x) ≥ f (0) because x2 ≥ 0 for all x ∈ R. Therefore, f (0) = 0 is the absolute
(and local) minimum value of f . This corresponds to the fact that (0, 0) is the lowest point
on the parabola y = x2 (See Figure 6.14). However, there is no highest point on the parabola
y = x2 and so this function has no maximum value.

2.) From the graph of the function shown in Figure 6.15, we see that this function has neither
an absolute maximum value nor an absolute minimum value. In fact, it has no local extreme
values either.
170

Figure 6.14: Minimum value 0, Figure 6.15: No minimum,


no maximum. no maximum.

Theorem 6.4.3. THE EXTREME VALUE THEOREM


If f is continuous on a closed interval [a, b], then f attains an absolute maximum value f (c) and
an absolute minimum value f (d) at some numbers c and d in (a, b) (as illustrated in Figure 6.16)

Figure 6.16: Illustration of the Extreme Value Theorem

Definition 6.4.4. A critical number c of a function f is a number in the domain of f such that
either f 0 (c) = 0 or f 0 (c) does not exist.
3
Example 6.22. Find the critical numbers of f (x) = x 5 (4 − x).

Solution 6.22. From Definition 6.4.4 we have that critical values of f occurs when f 0 (x) = 0 or
171

f 0 (x) does not exists. Thus,

d 3 3 d
f 0 (x) = 0 =⇒ (4 − x) (x 5 ) + x 5 (4 − x) = 0
dx dx
3 2 3
=⇒ (4 − x)x− 5 − x 5 = 0
5
3(4 − x) 3
=⇒ 2 − x5 = 0
x5
12 − 8x
=⇒ 2
5x 5
2
=⇒ 12 − 8x = 0 or 5x 5 = 0
3
=⇒ x = or x = 0
2

3
∴ x = 0 and x = 2 are the critical numbers of f .

THE CLOSED INTERVAL METHOD


Here are the steps for finding the absolute maximum and minimum values of a continuous function
f on a closed interval [a, b]:

1.) Find the values of f at the critical numbers of f in (a, b) .

2.) Find the values of f at the endpoints of the interval.

3.) The largest of the values from Steps 1.) and 2.) is the absolute maximum value; the smallest
of these values is the absolute minimum value.

Example 6.23. Find the absolute maximum and minimum values of the function

1
f (x) = x3 − 3x2 + 1, − ≤x≤4
2

Solution 6.23. Since f is continuous on [− 12 , 4], then we can use the Closed Interval Method to find
its absolute maximum and minimum.
172

f 0 (x) = 0 =⇒ 3x2 − 6x = 0

=⇒ 3x(x − 2) = 0

=⇒ 3x = 0 or x − 2 = 0

Therefore, x = 0 and x = 2 are the critical values. So,

 1 1
f − = , f (0) = 1, f (2) = −3, f (4) = 17
2 8

∴ Comparing these four values we see that the absolute maximum value is f (4) = 17 and the
absolute minimum value is f (2) = −3.

Theorem 6.4.5. THE MEAN VALUE THEOREM


Let f be a function that satisfies the following conditions:

1.) f is continuous on the closed interval [a, b].

2.) f is differentiable on the open interval (a, b).

Then there is a number c in (a, b) such that

f (b) − f (a)
f 0 (c) = (6.30)
b−a

or, equivalently,

f (b) − f (a) = f 0 (c)(b − a) (6.31)

We observe that this is the slope of the secant line AB from A(a, f (a)) to B(b, f (b)) given by

f (b) − f (a)
mAB =
b−a
173

Example 6.24. Show that the function

f (x) = x3 − x, 0≤x≤2

satisfies the Mean Value Theorem

Solution 6.24. Since f is a polynomial, then f is continuous on [0, 2]. Furthermore, f is differentiable
on (0, 2). By the Mean Value Theorem, there is a number c ∈ (0, 2) such that

f (2) − f (0) = f 0 (c)(2 − 0) =⇒ 6 = (3c2 − 1)2

=⇒ 8 = 6c2
4
=⇒ c2 =
3

Therefore c = ± √23 . Since c ∈ (0, 2), then c = √2 .


3

HOW DERIVATIVES AFFECT THE SHAPE OF A GRAPH

Many of the applications of Engineering Mathematics depend on our ability to deduce facts about
functions from information concerning their derivatives. Since f 0 (x) represents the slope of the
curve y = f (x) at the point (x, f (x)), it tells us the direction in which the curve proceeds at each
point (x, f (x)). Therefore f 0 (x) provides us with the information about the behaviors of f (x).

INCREASING/ DECREASING TEST

a) If f (0 x) > 0 on an interval, then f is increasing on that interval.

b) If f 0 (x) < 0 on an interval, then f is increasing on that interval.

The increasing or decreasing test is demonstrated in Figure 6.17 below.


174

Figure 6.17: Illustration of the Increasing/Decreasing Test

Example 6.25. Find where the function f (x) = 3x4 − 4x3 − 12x2 + 5 is increasing and where it
is decreasing?

Solution 6.25. We have that

f 0 (x) = 12x3 − 12x2 − 24x = 12x(x2 − x − 2) = 12x(x + 1)(x − 2)

So, f 0 (x) = 0 =⇒ x = −1, x = 0 and x = 2 are the critical values.

Interval 12x x+1 x−2 f 0 (x) f (x)


x < −1 - - - - Decreasing
−1 < x < 0 - + - + Increasing
0<x<2 + + - - Decreasing
x>2 + + + + Increasing

Table 6.2: Behavior of f (x).

We can see from Table 6.2 that f (0) is a local maximum of f because f increases on (−1, 0) and
decreases on (0, 2) and f (−1) is a local minimum of f because f decreases on (−∞, −1) and in-
creases on (−1, 0).

FIRST DERIVATIVE TEST


Suppose that c si a critical number of a continuous function f .
175

a) If f 0 changes from positive to negative at c, then f has a local maximum at c.

b) If f changes from negative to positive at c, then f has a local minimum at c.

c) If f 0 does not change sign at c, then f has no local maximum or minimum at c.

The First Derivative Test can be visualized in the following diagram.

Figure 6.18: Illustration of the First Derivative Test

Example 6.26. Find the local maximum and minimum values of the function f in Example 6.25.

Solution 6.26. From Table 6.2 f 0 (x) changes from negative to positive at −1, so f (−1) = 0 is a local
minimum value by the First Derivative Test. Similarly, f 0 (x) changes from negative to positive at
2, so f (2) = −27 is also a local minimum value. As previously noted, f (0) = 5 is a local maximum
value because f 0 (x) changes from positive to negative at 0.

Definition 6.4.6. If the graph of f lies above all of its tangents on an interval I, then it is called
concave upward on I. If the graph of f lies below all of its tangents on I, then it is called
concave downward on I.

Figure 6.19 shows the graph of the function that is concave upward and Figure 6.20 shows the
graphs of the function that is concave downward.
176

Figure 6.19: concave upward Figure 6.20: concave downward

CONCAVITY TEST

a) If f 00 (x) > 0 for all x ∈ I, then the graph of f is concave upward on I.

b) If f 00 (x) < 0 for all x ∈ I, then the graph of f is concave downward on I.

Definition 6.4.7. A point P on a curve y = f (x) is called an inflection point if f is continuous


at P and the curve changes from concave upward to concave downward or from concave downward
to concave upward at P .

SECOND DERIVATIVE TEST

a) If f 0 (c) = 0 and f 00 (c) > 0, then f has a local minimum at c.

b) If f 0 (c) = 0 and f 00 (c) < 0, then f has a local maximum at c.

Example 6.27. Discuss the curve y = x4 − 4x3 with respect to concavity, points of inflection, and
local maxima and minima. Use this information to sketch the curve.

Solution 6.27. We have that,

f 0 (x) = 4x3 − 12x2 = 4x2 (x − 3)

f 00 (x) = 12x2 − 24x = 12x(x − 2)


177

Letting f 0 (x) = 0 we get x = 0 and x = 3 as the critical numbers. In order to use the second
Derivative Test we evaluate f 00 at the critical numbers. So,

f 00 (0) = 0 f 00 (3) = 36 > 0

Since f (0) = 0 and f (3) = −27 > 0, f 0 (0) = −27 is a local minimum. Since f 00 (0) = 0, then
the Second Derivative Test does tell us anything about the critical number 0. Lets consider the
following table for more information.

Interval 4x2 x−3 f 0 (x) f (x)


x<0 + - - Decreasing
0<x<3 + - - Decreasing
x>3 + + + Increasing

Table 6.3: Fist Derivative Test.

From 6.3 we have that f 0 (x) < 0 for x < 0 and 0 < x < 3, the First Derivative Test tells us that f
does not have a local minimum or maximum at 0.

Since f 00 (0) = 0 when x = 0 or 2, then we can consider the following table.

Interval 12x x−2 f 00 (x) Concavity


x<0 - - + upward
0<x<2 + - - downward
x>2 + + + upward

Table 6.4: Concavity Test.

From 6.4 we can deduce that the points (0, 0) and (2, −16) are inflection points since the curve
changes from concave upward to concave downward, and from concave downward to concave up-
ward respectively.

Using the local minimum, the intervals of concavity, and the inflection points, we sketch the curve
as in Figure 6.21.
178

Figure 6.21: Sketch of the curve y = x4 − 4x3

Exercise 6.9. 1. For the curve y = x4 (x − 1)3 ,

a) Find the critical numbers.

b) What does the First Derivative Test tell you about the behavior of f at those critical numbers?

c) What does the Second Derivative Test tell you about the behavior of f at those critical
numbers?

2. Consider the following functions

a) f (x) = 2x3 + 3x2 − 36x

b) g(x) = x4 − 2x2 + 3

c) h(x) = √x
ln
x

For the functions above,

i) Find the intervals on which f is increasing or decreasing.

ii) Find the local maximum and minimum values of f .

iii) Find the intervals of concavity and the inflection points.


179

6.4.2 INDETERMINATE FORMS AND L’HOSPITAL’S RULE

In this section we will apply differentiation to compute limits of functions.

Suppose we want to analyze the behavior of the function

ln x
f (x) =
x−1

In computing this limit we can’t apply the quotient law of limits because the limit of the denomina-
tor is 0. Although the limit exists, its value is not obvious because both numerator and denominator
0
approach 0 and 0 is not defined.

In general, if we have a limit of the form

f (x)
lim
x→a g(x)

In this case both f (x) → 0 and g(x) → 0, then this limit may or may not exist and is called an
0
indeterminate form of type 0. We met some similar limits of this type in Chapter 5. For
example

√ √
3h + 3 − 3
lim
h→0 h

In this particular case we can rationalize. Thus,

√ √ √ √ √ √
3h + 3 − 3 3h + 3 − 3 3h + 3 + 3
lim = lim √ √
h→0 h h→0 h 3h + 3 + 3
3h + 3 − 3 h
3
= lim √ √ = lim √ √
h→0 h( 3h + 3 + 3) h( 3h + 3 + 3)
h→0 
3
= lim √ √
→0 3h + 3 + 3

3 3
= √ =
2 3 2
180

Definition 6.4.8. L’HOSPITAL’S RULE2


Suppose f and g are differentiable and g 0 (x) 6= 0 on an open interval that contains a (except possibly
at a). Suppose that
lim f (x) = 0 and lim g(x) = 0
x→a x→a

or
lim f (x) = ±∞ and lim g(x) = 0
x→a x→a

0 ∞
(i.e. we have an indeterminate form of type o or ∞ .) Then

f (x) f 0 (x)
lim = lim 0
x→a g(x) x→a g (x)

Example 6.28. Compute the following limits


ln x
a) limx→1 x−1

ex
b) limx→∞ x2

Solution 6.28. a) Since limx→1 ln x = ln 1 = 0 and limx→1 (x − 1) = 0, then we have an indetermi-


nate form of type 00 . So, we can apply L’Hospital’s Rule. Thus,

d
ln x dx (ln x)
lim = lim d
x→1 x − 1
dx (x − 1)
x→1
1
x
= lim
1
x→1
1
= lim
x→1 x

=1

ln x
∴ limx→1 x−1 = 1.


b) Since limx→∞ ex = ∞ and limx→∞ x2 = ∞, then we have an indeterminate form of type ∞. So,
2
L’Hospital’s Rule is named after a French nobleman, the Marquis de l’Hospital (1661–1704), but was discovered
by a Swiss mathematician, John Bernoulli (1667–1748). You might sometimes see l’Hospital spelled as l’Hôpital, but
he spelled his own name l’Hospital, as was common in the 17th century.
181

we can apply L’Hospital’s Rule. Thus,

d x
ex dx (e )
lim = lim d
x→∞ x2 x→∞ 2
dx (x )
ex
= lim
x→∞ 2x

Furthermore, limx→∞ ex = ∞ and limx→∞ 2x = ∞, then we have an indeterminate form of type



∞. So, we can apply L’Hospital’s Rule again. Thus,

d x
ex dx (e )
lim = lim d
x→∞ 2x x→∞
dx (2x)
ex
= lim
x→∞ 2
=∞

INDETERMINATE PRODUCTS

If limx→a f (x) = 0 and lim→a g(x) = ±∞, then it is not clear what the value of lim→a f (x)g(x)
will be. This kind of limit is called an indeterminate form of type 0 · ∞. We will deal with this
kind of limit by writing the product f g as a quotient:

f g
fg =   or fg =  
1 1
g f

0 ∞
This converts the given limit into an indeterminate form of type 0 or ∞ so that we can use
L’Hospital’s Rule.

Example 6.29. Evaluate limx→0+ x ln x

Solution 6.29. Since limx→0+ x = 0 and limx→0+ ln x = −∞, then we have an indeterminate form
182

of type 0 · ∞. So, we can apply L’Hospital’s Rule. Thus,

d 1
ln x dx (ln x) x
lim x ln x = lim 1 = lim = lim = lim (−x) = 0
− x12
 
x→0+ x→0+ x→0+ d 1 x→0+ x→0+
x dx x

∴ limx→0+ x ln x = 0

INDETERMINATE DIFFERENCES

If limx→a f (x) = ∞ and limx→a g(x) = ∞, then the limit

lim [f (x) − g(x)]


x→a

is called an indeterminate form of type ∞ − ∞. To compute such limits, we try to convert the
difference into a quotient by using a common denominator, or rationalization, or factoring out a
0 ∞
common factor so that we have an indeterminate form of type 0 or ∞.

Example 6.30. Compute limx→ π − (sec x − tan x).


2

Solution 6.30. Since limx→ π − sec x = ∞ and limx→ π − sec x = ∞, then we have an indeterminate
2 2

form of type ∞ − ∞. In this particular case we use a common denominator:

 1 sin x 
lim (sec x − tan x) = lim −
x→ π2 − cos x
x→ π2 − cos x
1 − sin x
= lim
π−
x→ 2 cos x
d
dx (1 − sin x)
= lim d
x→ π2 − dx (cos x)
cos x
= lim =0
x→ π2 − sin x
183

INDETERMINATE POWERS

Below are indeterminate forms arising from the limit

lim [f (x)]g(x)
x→a

(1.) lim f (x) = 0 and lim g(x) = 0 type 00


x→a x→a

(2.) lim f (x) = ∞ and lim g(x) = 0 type ∞0


x→a x→a

(3.) lim f (x) = 1 and lim g(x) = ±∞ type 1∞


x→a x→a

(4.) lim f (x) = ∞ and lim g(x) = ∞ type ∞∞


x→a x→a

Each of these four cases can be treated either by taking the natural logarithm:

Let y = [f (x)]g(x) , then ln y = g(x) ln f (x)

or by writing the function as an exponential:

[f (x)]g(x) = eg(x) ln f (x)

In either method we are led to the indeterminate product g(x) ln f (x), which is of type 0 · ∞.

Therefore,
lim [f (x)]g(x) = elimx→a g(x) ln f (x)
x→a

Example 6.31. Evaluate the following limits.

a) limx→0+ (1 + sin 4x)cot x

b) limx→0+ xx

Solution 6.31. a) Since limx→0+ (1 + sin 4x) = 1 and limx→0+ cot x = ∞, then we have an indeter-
184

minate form of type 1∞ . Let


y = (1 + sin 4x)cot x

Then
ln(1 + sin 4x)
ln y = ln(1 + sin 4x)cot x = cot x ln(1 + sin 4x) =
tan x

So, By L’Hospital’s Rule we have

ln(1 + sin 4x)


lim ln y = lim
x→0+ x→0+ tan x
d
dx (ln(1 + sin 4x))
= lim d
dx (tan x)
x→0+
4 cos 4x
1+sin 4x
= lim
x→0+ sec2 x
4
= =4
1

cot x
∴ limx→0+ (1 + sin 4x)cot x = elimx→0+ ln(1+sin 4x) = e4 .

b) Since limx→0+ x = 0, then we have an indeterminate form of type 00 . Thus

x
lim xx = elimx→0+ ln x = elimx→0+ x ln x
x→0+

From Example 6.29, have that


lim x ln x = 0
x→0+

Therefore,

lim xx = e0 = 1
x→0+

Exercise 6.10. Evaluate the following limits


1−sin θ
a) limθ→ π2 csc θ

b) limx→0 cot 2x sin 6x


185

c) limx→ π4 (1 − tan x) sec x


 x
2
d) limx→∞ 1 + x

6.4.3 PRACTICAL APPLICATIONS OF DIFFERENTIATION

dy
In Section 6.1 we have established that if y = f (x), then the derivative dx can be interpreted
as the rate of change of y with respect to x. In this section we will examine a few examples of
practical applications of the derivative in physics, chemistry, biology, economics, and other sciences.

PART A: RELATED RATES OF CHANGE

Example 6.32. How rapidly will the fluid level inside a vertical cylindrical tank drops if we pump
the fluid out at the rate of 3m3 /minutes?

Solution 6.32. In order to solve the problem above, consider the figure below.

Figure 6.22: A cylindrical tank with fluid.

Let V , r and h be the volume of the fluid in the tank, the radius of the tank, and the level of the
fluid in the tank at time t, where t is measured in minutes.

We start by identifying two things:


dV
1.) Given information: The rate of change of the volume of the fluid in the tank is dt = −3 m3 /minutes.
186

dh
2.) Unknown: The rate of change of the level of the fluid in the tank dt .

dr dV dh
We know that the radius of the tank remains constant i.e. dt = 0. In order to connect dt and dt ,

we first relate V , h and r by the formula for the volume of a cylinder:

V = πr2 h

In order to apply the given information, we differentiate each side of this equation with respect to
t. Thus,
dV dh
= πr2
dt dt

Now, we solve for the unknown quantity:

dV
dh 3
= dt2 = − 2 m/minutes
dt πr πr

dh
∴ The rate at which the fluid level is changing is: dt = − πr3 2 m/minutes.

Example 6.33. Air is being pumped into a spherical balloon so that its volume increases at a rate
of 100 cm3 /s. How fast is the radius of the balloon is increasing when the diameter is 50 cm?

Solution 6.33. In order to solve the problem above, consider the figure below.

Figure 6.23: A spherical balloon, with radius r.

Let V and r be the volume of the balloon and its radius at time t, where t is measured in seconds.
187

We start by identifying two things:


dV
1.) Given information: The rate of increase of the volume of the air is dt = 100 cm3 /s.
dr
2.) Unknown: The rate of increase of the radius dt when the diameter is 50 cm.

dV dr
In order to connect dt and dt , we first relate V and r by the formula for the volume of a sphere:

4
V = πr3
3

In order to apply the given information, we differentiate each side of this equation with respect to
t. Thus,
dV dr
= 4πr2
dt dt

Now, we solve for the unknown quantity:

dV
dr
= dt 2
dt 4πr

50 dV
If we substitute r = 2 = 25cm and dt = 100 cm3 /s in the equation above, we get

dr 100 1
= 2
= cm/s
dt 4π(25) 25π

∴ The rate at which the radius of the balloon is increasing when its diameter is 50 cm is given by:
dr 1
dt = 25π cm/s.

Example 6.34. A hot air balloon rising straight up from a level field is tracked by a range finder
π
500 ft from the lift off point. At the moment, the range finder’s angle of elevation is 4, the angle
is increasing at the rate of 0.14 rad/min. How fast is the balloon rising?

Solution 6.34. In order to analyze the problem, consider the figure below.
188

Figure 6.24

Let R, y, and θ be the distance between the range finder and the air balloon, altitude gained by
the air balloon, and range the finder’s angle of elevation at time t, where t is measured in minutes.

We start by identifying two things:



1.) Given information: The rate of increase of the range finder’s angle of elevation is dt =
0.14 rad/min.
dy
2.) Unknown: The rate of increase of the altitude dt , of the air balloon when the the range finder’s
π
angle of elevation is 4.

dy dθ
In order to connect dt and dt , we first relate y and θ by the following formula:

y
tan θ = =⇒ y = 500 tan θ
500

In order to apply the given information, we differentiate each side of the equation above with respect
to t. Thus,
dy dθ 500 dθ
= 500 sec2 θ =
dt dt cos2 θ dt

π
If we substitute θ = 4 rad in the equation above, we get

dy 500(0.14)
=   = 140 ft/min
dt cos2 π4
189

π
∴ The rate at which the air balloon is rising when the range finder’s angle of elevation is 4 is given
dy
by: dt = 70 ft/min.

Example 6.35. A ladder 10 ft long rests against a vertical wall. If the bottom of the ladder slides
away from the wall at a rate of 1 ft/s, how fast is the top of the ladder slides down the wall when
the bottom of the ladder is 6 ft from the wall?

Solution 6.35. In order to analyze the problem, consider the figure below.

Figure 6.25: A sliding ladder problem

Let x be the distance from the bottom of the ladder to the wall and y the distance from the top of
the ladder to the ground. Both x and y are functions of time.

We start by identifying two things:

1.) Given information: The rate at which the bottom of the ladder is sliding away from the wall is
dx
dt = 1 ft/s.

2.) Unknown: The rate at which the top of the ladder is sliding down the wall when the bottom
of the ladder is 6 ft from the wall.

dx dy
In order to connect dt and dt , we notice that x is related to y by Pythagoras’s theorem.

x2 + y 2 = 100
190

Differentiating each side with respect to t using the Chain Rule, we have,

dx dy
2x + 2y =0
dt dt

Now, we solve for the unknown quantity:

dy x dx
=−
dt y dt


When x = 6, by Pythagorean Theorem we get y = 100 − 36 = 8 and so, substituting these values
dx
and dt = 1, we have
dy 6 3
= − (1) = − ft/s
dx 8 4

dy
∴ The rate at which the top of ladder is sliding down the wall is dx = − 34 ft/s.

Example 6.36. A water tank has the shape of an inverted circular cone with base radius 2m and
height 4 m. If water is being pumped into the tank at a rate of 2 m3 /min, find the rate at which
the water level is rising when the water is 3 m deep.

Solution 6.36. In order to analyze the problem, consider the figure below.

Figure 6.26: An inverted circular cone shaped water tank.

Let V , r, and h be the volume of the water, the radius of the surface, and the height of the water
at time t, where t is measured in minutes.
191

We start by identifying two things:


dV
1.) Given information: The rate of at which the water is being pumped into the tank dt is
2 m3 /min.
dh
2.) Unknown: The rate at which the water level is rising dt when the water level is 3 m deep.

dV dh
In order to connect dt and dt , we first relate V , h and r by the formula for the volume of a cone:

1
V = πr2 h
3

Figure 6.27: Similar triangles

However, it is very useful to express V as a function of h alone. In order to eliminate r, we use the
concept of similarity of triangle as shown in Figure 6.27 to write,

r h h
= =⇒ r =
2 4 2

Therefore, the expression for V becomes

1  h 2 1
V = π h = πh3
3 2 12

In order to apply the given information, we differentiate each side of this equation with respect to
t. Thus,
dV 1 dh
= πh2
dt 4 dt
192

Solving for the unknown quantity we will get,

dh 4 dV
= dt2
dt πh

dV
By substituting h = 3 m and dt = 2m3 /min, we have

dh 4(2) 8
= 2
= m/min
dt π(3) 9π

dh 8
∴ The rate at which the fluid level is changing is: dt = 9π ≈ 0.28 m/minutes.

Example 6.37. Car A is traveling west at 50 km/h and Car B is traveling north at 60 km/h.
Both cars are heading for the intersection of the two roads. At what rate are the cars approaching
each other when car A is 0.3 km and car B is 0.4 km from the intersection?

Solution 6.37. Let C be the intersection of the two roads. At a given time t, let x be the distance
from car A to C, let y be the distance from car B to C, and let z be the distance between the cars,
where x, y, and z are measured in kilometers as demonstrated in Figure 6.28 below.

Figure 6.28: Two cars approaching an intersection.

We start by identifying two things:

1.) Given information: The rate at which the distance between Car A and the intersection C is
dx
decreasing is dt = −50 km/h, and the rate at which the distance between Car B and the
193

dx
intersection C is decreasing is dt = −60 km/h.
dz
2.) Unknown: The rate at which the distance between cars A and B given by dt is decreasing when
cars A and B are 0.3 km and 0.4 km away from the intersection C, respectively.

dx dy dz
In order to connect dt , dt , and dt we notice that the equation that relates x, y, and z is given by
the Pythagorean Theorem:
x2 + y 2 = z 2

Differentiating each side with respect to t using the Chain Rule, we have,

dx dy dz
2x + 2y = 2z
dt dt dt

Now, we solve for the unknown quantity:


!
dx 1 dx dy
= +
dt z dt dt


When x = 0.3 km and y = 0.4 km, by Pythagorean Theorem we get y = 0.32 + 0.42 = 0.5 km
dx dy
and substituting these values and, dt = −50 and dt = −60, we get

dz
= 2(0.3(−5) + 0.4(−60)) = −78 hm/h
dx

∴ The cars are approaching each other at a rate of 78 km/h.

PART B: OPTIMIZATION PROBLEMS

The methods we have learned in Section 6.4.1 for finding extreme values have practical applications
in many areas of life. For example a businessperson wants to minimize costs and maximize prof-
its. A traveler wants to minimize transportation time. In this section we solve problems involving
maximizing areas, volumes, and profits and minimizing distances, times, and costs.

In solving such practical problems the greatest challenge is often to convert the word problem
194

into a mathematical optimization problem by setting up the function that is to be maximized or


minimized. Below are some useful steps in solving optimization problems:

(1.) Understand the Problem: The first step is to read the problem carefully until it is clearly
understood. Ask yourself: What is the unknown? What are the given quantities? What are
the given conditions?

(2.) Draw a Diagram: In most problems it is useful to draw a diagram and identify the given
and required quantities on the diagram.

(3.) Introduce Notation: Assign a symbol to the quantity that is to be maximized or minimized.
Also select appropriate symbols for other unknown quantities and label the diagram with these
symbols.

(4.) Express the quantity to be minimized or maximized in terms of some of the other symbols
(say Q = f (x1 , x2 , . . . , xn )) from (3.).

(5.) After expressing the quantity to be minimized or maximized as a function of more than one
variable in (4.), use the given information to find relationships among these variables. Then
use these equations to eliminate all but one of the variables in the expression.

(6.) Use the methods of Section 6.4.1 to find the absolute maximum or minimum value of f .

Example 6.38. A farmer has 2400 ft of fencing and wants to fence off a rectangular field that
borders a straight river. He needs no fence along the river. What are the dimensions of the field
that has the largest area?

Solution 6.38. As illustrated on the figure below, we wish to maximize the area A of the rectangle.
195

Figure 6.29: Area of a rectangular field with dimensions x and y.

Let x and y be the depth and width of the rectangle in feet. We express A in terms of x and y:

A = xy

In order to express A as a function of just one variable (say A = f (x)), we use the fact that the
total length of the fencing is 2400 ft. Thus

2x + y = 2400 =⇒ y = 2400 − 2x

Substituting the expression for y in the expression for area we get

A(x) = x(2400 − 2x) = 2x(1200 − x)

Note that 0 ≤ x ≤ 1200 so that A > 0 (otherwise A < 0). So, we wish to maximize the function

A(x) = 2400x − 2x2

Since
A0 (x) = 2400 − 4x,

then to find the critical numbers we solve the equation

A0 (x) = 0 =⇒ 2400 − 4x = 0
196

which gives x = 600. The maximum value of A must occur either at this critical number or at an
endpoint of the interval. Thus

A(0) = 0, A(600) = 720000, and A(1200) = 0

The Closed Interval Method gives the maximum value as A(600) = 720000 ft2 . Thus,

x = 600 and y = 2400 − 2(600) = 1200

Therefore, the rectangular field with the largest area should be 600 ft long and 1200 ft wide.

Example 6.39. A cylindrical can is to be made to hold 1 L of oil. Find the dimensions that will
minimize the cost of the metal to manufacture the can.

Solution 6.39. Figure 6.30 illustrate the cylindrical can, where r and h are the radius and height of
the can in centimeters. In order to minimize the cost of the metal, we minimize the total surface
area of the cylinder. From Figure 6.31 we see that the total surface area is composed of two circular
plates with radius r and the sides made from a rectangular sheet with dimensions 2πr and h. So
the surface area is
A = 2πr2 + 2πrh

Figure 6.31: Total Surface Area of a


Figure 6.30: A cylindrical can.
cylindrical can.
197

To eliminate h we use the fact the volume should be given by V = 1 L = 1000 cm3 . Thus,

1000
πr2 h = 1000 =⇒ h =
πr2

Substituting the expression for h into the formula for the total surface area A we get
!
2 1000 2000 2πr3 + 2000
A(r) = 2πr + 2πr = 2πr2 + =
πr2 r r

Note that r > 0, so we wish to minimize the function

2πr3 + 2000
A(r) =
r

Since
2000 4πr3 − 2000
A0 (r) = 4πr − =
r2 r2

To find the critical numbers we solve the equation

4πr2 − 2000
A0 (r) = 0 =⇒ = 0 and A0 (r) is undefined
r2
q q
3 500 3 500
We get r = π and r = 0. Since r > 0, then the only critical number is r = π . Furthermore,
q q
we observe that A0 (r) < 0 for r < 3 500
π and A0 (r) > 0 for r > 3 500
π . So, A is decreasing for all
!
q q q
3 500 3 500 3 500
r< π and increasing for all r > π . Therefore A π is the local minimum.

q
500
The value of h corresponding to r = π is

r
1000 3 500
h =  2 = 2 = 2r
3 π
π 500
π

q
3 500
∴ To minimize the cost of metal to manufacture the can, the radius should be π cm and the
q
500
height should be 2 3 π cm which is equals to the diameter.

Example 6.40. A man launches his boat from point A on a bank of a straight river, 3 km wide,
198

and wants to reach point B, 8 km downstream on the opposite bank, as quickly as possible (see
Figure 6.32). He could row his boat directly across the river to point C and then run to B, or he
could row directly to B, or he could row to some point D between C and B and then run to B. If
he can row at 6 km/h and run at 8 km/h, where should he land to reach as soon as possible? (We
assume that the speed of the water is negligible compared with the speed at which the man rows.)

Figure 6.32

Solution 6.40. Let x be the distance from C to D, then the running distance is |DB| = 8 − x and

by Pythagorean Theorem the rowing distance is |AD| = x2 + 9. We use the equation

Distance
Time =
Rate

x2 +9 8−x
Then the rowing time is 6 and the running time is 8 , so the total time T as a function of x
is √
x2 + 9 8 − x
T (x) = + , 0≤x≤8
6 8

If x = 0, he rows to C and if x = 8, he rows directly to B. The derivative of T is

x 1
T 0 (x) = √ −
6 x +9 8
2
199

Since x ≥ 0, we have that

x 1
T 0 (x) = 0 ⇐⇒ √ =
6 x2 + 9 8
p
⇐⇒ 4x = 3 x2 + 9

⇐⇒ 16x2 = 9x2 + 81

⇐⇒ 7x2 = 81
9
⇐⇒ x = √
7

The only critical number is x = √9 . To see whether the minimum occurs at this critical number
7
or at an endpoint of the domain [0, 8], we evaluate T at all three points:

 9 
√ √
7 73
T (0) = 1.5, T √ =1+ ≈ 1.33, T (8) = ≈ 1.42
7 8 8

Since the smallest of these values of T occurs when x = √9 , the absolute minimum value of T must
7
9
√ ≈ 3.4 km downstream from his starting
occur there. ∴ The man should land his boat at a point 7
point.

Example 6.41. A 1125 m3 open-top rectangular tank with a square base x m on the side and y
m deep is to be built with its top flush with the ground to catch runoff water. The cost associated
with the tank involves not only the material from which the tank is made but also an excavation
charge proportional to the product xy. If the total cost is

C(x, y) = 5(x2 + 4xy) + 10xy,

what values of x and y will minimize the cost and determine that minimum cost?

Solution 6.41. The rectangular tank to be build is shown in Figure 6.33 below.
200

Figure 6.33: A 1125 m3 rectangular tank

Let x and y be the breadth/length and height of the tank in m. The total cost associated with the
tank is given by:
C(x, y) = 5(x2 + 4xy) + 10xy

In order to express C as a function of just one variable (say C = f (x)), we use the fact that the
volume of the tank must be 1125 m3 . Thus

1125
x2 y = 1125 =⇒ y =
x2

Substituting the expression for y into the formula for the cost function we get
!
2 1125 33750
C(x) = 5x + 30x = 5x2 +
x2 x

Note that x > 0, so we wish to minimize the cost function

33750
C(x) = 5x2 +
x

Since
33750
C 0 (x) = 10x −
x2

To find the critical numbers we solve the equation

33750 10x3 − 33750


C 0 (x) = 0 =⇒ 10x − = = 0 or C 0 (x) is undefined
x2 x2

We get x = 15 and x = 0. Since x > 0, then the only critical number is x = 15. Furthermore, we
201

observe that C 0 (x) < 0 for x < 15 and C 0 (x) > 0 for x > 15. So, C is decreasing for all x < 15 and
increasing for all x > 15. Therefore C(15) is the local minimum.

The value of y corresponding to x = 15 is

1125
y= =5m
152

∴ To minimize the total cost for building the tank, the square base side should be 15 m and the
height should be 5 m, and total cost will be

C(15, 5) = 5(15)2 + 30(15)(5) = N $2750

Exercise 6.11. Solve the following problems

1. A spherical balloon is being inflated. Find the volume of the balloon at the instant when the
rate of increase of the surface area is eight times the rate of increase of the its radius.

2. A woman 5 feet tall is walking away from a streetlight hung 20 feet from the ground at the rate
of 6 ft/sec. How fast is her shadow lengthening?

3. Two cars leave an intersection at the same time. The first car is going due east at the rate of
40 km/h and the second is going due south at the rate of 30 km/h. How fast is the distance
between the two cars increasing when the first car is 120 km from the intersection?

4. An open box is to be made using a piece of cardboard 8 cm by 15 cm by cutting a square from


each corner and folding the sides up. Find the length of a side of the square being cut so that
the box will have a maximum volume.

5. A rocket is sent vertically up in the air with the position function s = 100t2 where s is measured
in meters and t in seconds. A camera 3000 m away is recording the rocket. Find the rate of
change of the angle of elevation of the camera 5 seconds after the rocket went up.

6. Given the cost function C(x) = 2500 + 0.02x + 0.004x2 , find the production level such that the
average cost per unit is a minimum.

7. Find the maximum area of a rectangle inscribed in an ellipse whose equation is 4x2 +25y 2 = 100.
202

REFERENCES
203

APPENDIX A

QUESTIONNAIRE

This is how an appendix is given. It has a heading just like a chapter and title describing the ap-
pendix. Whenever it is referred to in the main section of the report, it is referred to as Appendix A.

This might include your questionnaire, feedback from respondents, maps, etc. If you have any
figures or tables here, number the according to the appendix, e.g. Figure A.1 would be the first
figure in Appendix A.
204

APPENDIX B

USB STYLE SHEETS

You might also like